Sei sulla pagina 1di 158

CVS-System

Wise 1700-by Sush and Team. 2016


Susmita, Asad, Manu, Saima, Zohaib, Savia, Shanu, Mona, Manisha, Sitara, Samreena, Sami and Komal


Dear Plabber,


This first ever System Wise 1700 document was created thanks to 3 months of daily

hard work by the PLAB Skype group Unity which was brought together by Dr Susmita

Chowdhury.


The team members were:


& Susmita (Lead/most ignorant as she is working full time in public health for 13 years)

& Asad (Invaluable in IT and all types of support/the heart of the group)

& Manu (Volunteered to solve more questions/pathologist/amazing genuine person)

& Saima (Most concise clear notes/ photographic memory)

& Zohaib (Great research/a surgeon)

& Savia (Great research/multi-tasker with two little ones)

& Shanu (Very helpful after her March exam for those appearing in June)

& Mona (Great contributor in discussions)

& Manisha (Gyne/great discussion contributor)

& Sitara (Good discussion contributor)

& Samreena (Stayed a shorter time but great)

& Sami (Contributed the most early on but too brilliant for the group/still great friends)

& Komal (Knowledgeable sweet supportive girl)



The main purpose was to break down the 1700 Q Bank System wise.


We did our own reliable research for the options (OHCM/Patient info etc.) and

concluded these keys below on skype. This can save you 100s of hours of research. But I

suggest you also do your own.


90% of the document consists of Unity research. We also added information from other

circulating documents and they are referenced as Dr Khalid/Dr Rabia (and her Team).


However, several keys may be incorrect and so please use your own judgment as we

take no responsibility. I suggest cross checking with Dr Khalids latest keys (a few of

which are still debatable). Finally decide on your own key.


Sorry if some members failed to make their answers thorough. The highlights are mostly

as per what the team members wanted to highlight. Blank tables to be ignored.


Note that some 1700 Questions are missing from here (when members did not do their

share). Questions may not be in order due to merging of documents and there is excess

information than required. Read as much as needed.


This has been circulated by our team as a generous contribution to the Plabbers success

and must not be sold.

Good luck and best wishes: Sush and Team

1
CVS-System Wise 1700-by Sush and Team. 2016
Susmita, Asad, Manu, Saima, Zohaib, Savia, Shanu, Mona, Manisha, Sitara, Samreena, Sami and Komal

Q:561 A 48yo presents with severe chest pain since the last 40mins. In the ED he is
given oxygen, GTN, morphine. ECG=ST elevation. Bloods=increased troponin
levels. What is the next step of management?
a. Beta blockers
b. Percutaneous angiography
c. Anticoagulant & heparin
d. Clopidogrel
e. Aspirin

Clincher(s) Last 40 mins pt suffering from jSevere chest pain,st elevation on ecg and
increased troponin levels
A No role
B Stemi on ECG and PCI available within 120 mins
C After fibrinolysis
D Can be given initially
E 300 mg of aspirin should be given initially by GP or paramedics
KEY E
Additional If they asked for more specific then B should be the answer but here they
Information asked the next step so aspirin E is the right answer.
Reference OHCM 809
Dr Khalid/Rabia Technically speaking trops are raised which means 2-3 hours have passed. PCI
is indicated within 120 mins.
It should be E
PCI is more appropriate as the clincher here is the time 40 mins that's why i
went for B. (Dr Rabia)
In case of ST elevation MI.. mx include aspirin at first usually given by GP or
paramedic then morphine with Metochlopramide . GTN not used routinely
except in case of HTN or severe LVF. Next step is PCI if available
within 120mins of hospital contact. If not available .fibrinolysis done if no CI
and later proceed for PCI.
As here time frame of 40 mins mentioned..i guess its PCI..B
Its B. ST elevation MI means that the thrombus clot has already been stabilized
and occluded the vessel...that is why we give t-PA (thrombolytic) in ST
elevation MI only... aim is to break down the thrombus, not stop it from
forming...aspirin only stops it from forming it doesn't break it down... smile
emoticon also raised troponin levels mean that the patient has a previous
infarct at most 10 days before (since it raised in only 40 minutes, not 4 hours),
having previous infarct means has 2 or 3 vessel disease and not 1 vessel
disease, he is high risk patient and should have angioplasty as soon as
possible...answer is B, angiography is done right before angioplasty

Dr khalid :

The key is B. Percutaneous angiography. This is a wrong key. Correct key is E.
Aspirin. [Next step of
management is aspirin].

2
CVS-System Wise 1700-by Sush and Team. 2016
Susmita, Asad, Manu, Saima, Zohaib, Savia, Shanu, Mona, Manisha, Sitara, Samreena, Sami and Komal



Q:579 A 65yo woman is undergoing coronary angiography. What measure will
protect her kidneys from contrast?
a. Furosemide
b. Dextrose
c. 0.45% saline
d. 0.9% saline

Clincher(s) Coronary angiography
A
B
C In paediatrics used (hypotonic)
D IV access ideally in the left hand. Catheter is placed either in femoral or radial
artery.
E
KEY D
Additional
Information


Reference OHCM page 104
Dr Khalid/Rabia Post contrast nephropathy due to contrast induced or cholesterol
embolisation.
adequately hydrated pt prior to procedure reduces the complications


Q:603 A 53yo had a dental extraction after which he recently had a mitral valve
prolapse, high temp of 39C, cardiac failure and new cardiac murmur. What is
the single most likely dx?
a. Atheroma

3
CVS-System Wise 1700-by Sush and Team. 2016
Susmita, Asad, Manu, Saima, Zohaib, Savia, Shanu, Mona, Manisha, Sitara, Samreena, Sami and Komal

b. Congenital
c. Regeneration
d. Infection
e. Neoplastic

Clincher(s) Dental extraction, cardiac failure and cardiac murmur, high temp, mitral
valve prolapse after dental procedure.
A
B
C
D
E
KEY D
Additional Fever plus new murmur = (Bacterial) Endocarditis until proven otherwise.
Information (Srept viridian)
OHCM page 144
Reference
Dr Khalid/Rabia Post MI and endocarditis =MR
Antibodies on strep > vegetation on corda tendinae>M- prolapse>MR> LHF>
Pul edema (orthopmea, PND, Frothy sputum> Cardiac wheeze, > Fluffy shadow

Pul HTN>RHF

LHF+RHF> CHF

Pul: oedema> sit up, give O2 (learn Rx)
Loop diuretic
Chronic>
BP>100 dias> nitrate

Samson




Q:607 A 46yo African-Caribbean man is found to have BP=160/90mmHg on 3
separate occasions. What is the best initial tx?
a. ACEi
b. Beta-blockers
c. ARBs
d. None
e. CCB


Clincher(s) African Caribbean,BP 160/90 at 3 diff occasions.
A Younger than 55 yrs then ACEi is the first line

4
CVS-System Wise 1700-by Sush and Team. 2016
Susmita, Asad, Manu, Saima, Zohaib, Savia, Shanu, Mona, Manisha, Sitara, Samreena, Sami and Komal

B It's not normally the first line hypertensive unless if ACEi is contraindicated or
female with childbearing age.
C If dry cough is the issue then switch to ARB
D X
E 55yr or above and in black pts of any age.
KEY E
Additional If > 55 yes and in black pts of any age, first choice is a calcium channel blocker
Information or thiazides. If < 55 then Ace-I is the first choice.
B blockers co snider in younger pts and females of child baring age.
OHCM page 134
Reference
Dr Khalid/Rabia If age less than 55 years but Afro-Caribbean origin then CCB].


Q:615 A 45yo man had recently started taking anti-HTN therapy. 6m later his
RBS=14mmol/l. Which single drug is most likely to have caused this?
a. Amlodipine
b. Bendroflumethiazide
c. Doxazosin
d. Lorsartan
e. Ramipril


Clincher(s) Random blood sugar (RBS) high
A SE: gingival hyper trophy, oedema non responsive to diuretics, headaches, LV
function decreased
B SE: hyperuricemia (gout), Ca inc, impotence,glucose level increase
HyperGLUC ( glucose, lipidemia,uricemia, calcemia)
C SE: dizziness, fatigue, vertigo, hypertension, symptomatic orthostatic
hypotension, malaise
D SE: hypotension, hyper kalemia
E SE: dry cough, hypotension, hyper kalemia, renal impairment
KEY B
Additional OHCM page 108
Information
Reference
Dr Khalid/Rabia High blood sugar is a well known side effect of Thiazide].
Hyper GLUC

Q:630 A 43yo lady is admitted with pyrexia, arthropathy, breathlessness and syncope.
She was recently dx with pulmonary emboli. There is an early diastolic sound
and a mid-diastolic rumble. Her JVP is elevated with prominent a-waves. What
is the most likely cause?
a. Mitral regurgitation
b. Ventricular ectopics
c. Pulmonary regurgitation

5
CVS-System Wise 1700-by Sush and Team. 2016
Susmita, Asad, Manu, Saima, Zohaib, Savia, Shanu, Mona, Manisha, Sitara, Samreena, Sami and Komal

d. Atrial myxoma
e. Complete heart block

A wave- pulmonary and tricuspid problem?
Clincher(s) Pyrexia, arthropathy, SOB, syncope, P emboli (systemic features)

Early diastolic sound and raised JVP with prominent a waves (mean
right side is involved now) both the signs are present in either
pulmonary regurgitation or tricuspid stenosis

Mid diastolic rumble ( Mitral stenosis)

A Signs : pan systolic murmur at apex radiating to Avila,RV heave, soft S1, split S2
and loud P2(Pulmonary hypertension)
B Ventricular ectopics are commonest post MI but can occur in normal healthy
person. Palpitation is the common presenting symptom.

Extrasystoles usually occur after a normal heartbeat and are followed by a
pause until the normal heart rhythm returns. Therefore, they may be felt as
'missed' or 'skipped' beats or 'feeling the heart has stopped'.
C Signs: early diastole at the left eternal edge (the graham steel murmur if assc
with mitral stenosis and pulmonary hypertension)
D Benign tumor of atria. It mimic infective endocarditis( fever,clubbing, wtloss,
inc ESR) or mitral stenosis (left atria obstruction and systemic emboli)
E Symptoms : bradycardia, feeling dizzy, SOB, chest pain
KEY D
Additional Three quarters of Atrial myxoma is found in left atria and tend to grow on
Information septal wall. Symptoms worse on lying and relieve on sitting forward.
They are more common in females.2:1
Usually sporadic but 10% can be familial and present in early age
Symptoms can be systemic, or valvular as described above.
For more info pls check British heart foundation.
Diagnosis: Echo

Tx: Excision

Raised JVP=rt heart side problem
Reference British heart foundation. Ohcm page 146
Dr Khalid/Rabia

Q:

Clincher(s)
A
B
C

6
CVS-System Wise 1700-by Sush and Team. 2016
Susmita, Asad, Manu, Saima, Zohaib, Savia, Shanu, Mona, Manisha, Sitara, Samreena, Sami and Komal

D
E
KEY
Additional
Information
Reference
Dr Khalid/Rabia tumour above but when they open the Myxoma falls on the valve producing
early diastolic plop.On top we
have extra cardiac symptoms

depending upon location,if myxoma obstructs the valve, then its diastolic
rumble...if leaflets r damaged, then
its a systolic rumble(due to regurgitation)
Jugular venous pressure may be elevated, and a prominent A wave may be
present.
A loud S1 is caused by a delay in mitral valve closure due to the
prolapse of the tumor into the mitral valve orifice (mimicking mitral
stenosis).
P2 may be delayed. Its intensity may be normal or increased,
depending on the presence of pulmonary hypertension.
In many cases, an early diastolic sound, called a tumor plop, is heard.
This sound is produced by the impact of the tumor against the
endocardial wall or when its excursion is halted.
An S3 or S4 may be audible.
A diastolic atrial rumble may be heard if the tumor is obstructing the
mitral valve.
If there is valve damage from the tumor, mitral regurgitation may cause
a systolic murmur at the apex.
A right atrial tumor may cause a diastolic rumble or holosystolic
murmur due to tricuspid regurgitation.
General examination may reveal fever, cyanosis, digital clubbing, rash,
or petechiae.



Q:636 A 60yo lady has severe chest pain. ECG shows changes of inferior wall MI. ECG
also shows progressive prolongation of PR interval until a QRS complex is
dropped. What is the most probable dx?
a. Atrial fibrillation
b. VT
c. SVT
d. Mobitz type I 2nd degree heart block
e. Mobitz type II 2nd degree heart block

AF- irr irrg pulse, P wave absent

7
CVS-System Wise 1700-by Sush and Team. 2016
Susmita, Asad, Manu, Saima, Zohaib, Savia, Shanu, Mona, Manisha, Sitara, Samreena, Sami and Komal

P-R prolong- AV node involvement- (2nd degree AV block) Mobitz1 or M2

In both QRS dropped

p-r interval lengthens progressively until vent beat is dropped- M 1
p-r internal lengthening is constant throughout - M2 ( 2: 1 =p:qrs)

in type 2 though pr lengthening occurs but its constant- not increasing
progressively

Inf MI have strong link with M1

Clincher(s) Inferior wall MI, prolongation of PR interval until QRS is dropped
A P wave absent and irregularly irregular rhythm
B

C SVT is narrow complex tachycardia and vassalva manoeuvres and IV


Adenosine block the AV mode to unmask the atrial cause from AV.
D Progressive prolongation of the PR interval culminating in a non-
conducted P wave
The PR interval is longest immediately before the dropped beat
The PR interval is shortest immediately after the dropped beat

E Intermittent non-conducted P waves without progressive


prolongation of the PR interval.
The PR interval in the conducted beats remains constant.
The P waves march through at a constant rate.
The RR interval surrounding the dropped beat(s) is an exact multiple
of the preceding RR interval (e.g. double the preceding RR interval for
a single dropped beat, treble for two dropped beats, etc)

8
CVS-System Wise 1700-by Sush and Team. 2016
Susmita, Asad, Manu, Saima, Zohaib, Savia, Shanu, Mona, Manisha, Sitara, Samreena, Sami and Komal

KEY D
Additional
Information
Reference
Dr Khalid/Rabia First degree heart block :PR interval > 0.2 seconds

Second degree heart block : type 1 (Mobitz I, Wenckebach): progressive
prolongation of the PR interval until a dropped beat occurs

type 2 (Mobitz II): PR interval is constant but the P wave is often not followed
by a QRS complex

Third degree (complete) heart block : there is no association between the P
waves and QRS complexes

From Mona M:

Mona M: Mobitz I heart block is characterized by progressive prolongation of
the PR interval on the electrocardiogram (ECG) on consecutive beats followed
by a blocked P wave (i.e., a 'dropped' QRS complex). After the dropped QRS
complex, the PR interval resets and the cycle repeats.
[
Mobitz II heart block is characterized on a surface ECG by intermittently
nonconducted P waves not preceded by PR prolongation and not followed by
PR shortening. There is usually a fixed number of non-conducted P waves for
every successfully conducted QRS complex, and this ratio is often specified in
describing Mobitz II blocks. For example, Mobitz II block in which there are two
P waves for every one QRS complex may be referred to as "2:1 Mobitz II block



Q:645 A 56yo man was recently put on anti-HTN meds and recent biochemistry on 2
occasions showed: Na+=132, K+=7.6, Urea=11.3, Creat=112. Which of the
following drugs is responsible for this result?
a. Amlodipine
b. Bendroflumethiazide
c. Doxazosin
d. Atenolol

9
CVS-System Wise 1700-by Sush and Team. 2016
Susmita, Asad, Manu, Saima, Zohaib, Savia, Shanu, Mona, Manisha, Sitara, Samreena, Sami and Komal

e. Ramipril

Clincher(s) Hyperkalemia, mild Hyponatremia, hyperuricemia
A
B
C
D
E Typical side effect is hyper kalemia and Hyponatremia
KEY E- ACEi
Additional OHCM page : 109
Information
Reference
Dr Khalid/Rabia


Q:660 A 63yo man continues to experience chest pain and has a temp of 37.8C 2 days
after an acute MI. His ECG shows widespread ST elevation with upward
concavity. What is the single most likely explanation for the abnormal inv?
a. Acute pericarditis
b. Cardiac tamponade
c. Atrial thrombus
d. Left ventricular aneurysm
e. Dressler syndrome


Clincher(s) Infective features 2 days after MI,ST elevation with upward concavity
A ECG: ST elevation but saddle shape ohcm page 92
B Present with Becks triad: falling BP, raised JVP and muffled heart sounds.
C ECG : ST elevation
D Again Post MI complication but presents with LVF, angina, recurrent VT
ECG: ST elevation.
E Complication of post MI after 4-6 weeks. Presents with chest pain, fever , SOB
because of pericarditis and pleural effusion
KEY A
Additional
Information
Reference
Dr Khalid/Rabia Pericarditis in the first 48 hours following a transmural MI is common

Features :
chest pain: may be pleuritic. Is often relieved by sitting forwards other
symptoms include non-productive cough, dyspnoea and flu-like symptoms
pericardial rub
tachypnoea
tachycardia

10
CVS-System Wise 1700-by Sush and Team. 2016
Susmita, Asad, Manu, Saima, Zohaib, Savia, Shanu, Mona, Manisha, Sitara, Samreena, Sami and Komal

Causes
viral infections (Coxsackie)
tuberculosis
uraemia (causes 'fibrinous' pericarditis)
trauma
post-myocardial infarction, Dressler's syndrome
connective tissue disease
hypothyroidism
ECG changes
widespread 'saddle-shaped' ST elevation
PR depression: most specific ECG marker for pericarditis

left ven aneurysm: ST elevation is leads V1-V4



Q:434 A pt had TIA which he recovered from. He has a hx of stroke and exam shows
HR in sinus rhythm. He is already on aspirin 75mg and anti-HTN drugs. What
other action should be taken?
a. Add clopidogrel only
b. Increase dose of aspirin to 300mg
c. Add warfarin
d. Add clopidogrel and statin
e. Add statin only

Clincher(s)
A Add clopidogrel only- NICE issued updated guidelines relating to stroke and
transient ischaemic attack (TIA) in 2008.
B Increased dose of aspirin to 300 mg
C Add warfarin
D Add clopidogrel and statin
E Add statin only
KEY A (Sush- I think D)
Additional Transient ischaemic attack
Information

NICE issued updated guidelines relating to stroke and transient ischaemic
attack (TIA) in 2008. They advocated the use of the ABCD2 prognostic score for
risk stratifying patients who've had a suspected TIA:

Criteria Points

A Age >= 60 years 1
B Blood pressure >= 140/90 mmHg 1
Clinical features

- Unilateral weakness
C 2
- Speech disturbance, no
1
weakness

11
CVS-System Wise 1700-by Sush and Team. 2016
Susmita, Asad, Manu, Saima, Zohaib, Savia, Shanu, Mona, Manisha, Sitara, Samreena, Sami and Komal

Duration of symptoms
D - > 60 minutes 2
- 10-59 minutes 1
Patient has diabetes 1


This gives a total score ranging from 0 to 7. People who have had a suspected
TIA who are at a higher risk of stroke (that is, with an ABCD2 score of 4 or
above) should have:

aspirin (300 mg daily) started immediately


specialist assessment and investigation within 24 hours of onset of
symptoms
measures for secondary prevention introduced as soon as the diagnosis is
confirmed, including discussion of individual risk factors


If the ABCD2 risk score is 3 or below:

specialist assessment within 1 week of symptom onset, including decision


on brain imaging
if vascular territory or pathology is uncertain, refer for brain imaging


People with crescendo TIAs (two or more episodes in a week) should be
treated as being at high risk of stroke, even though they may have an ABCD2
score of 3 or below.

Antithrombotic therapy

clopidogrel is recommended first-line (as for patients who've had a


stroke)
aspirin + dipyridamole should be given to patients who cannot tolerate
clopidogrel
statin?
these recommendations follow the 2012 Royal College of Physicians
National clinical guideline for stroke. Please see the link for more details
(section 5.5)
these guidelines may change following the CHANCE study (NEJM
2013;369:11). This study looked at giving high-risk TIA patients aspirin +
clopidogrel for the first 90 days compared to aspirin alone. 11.7% of
aspirin only patients had a stroke over 90 days compared to 8.2% of dual
antiplatelet patients


With regards to carotid artery endarterectomy:

recommend if patient has suffered stroke or TIA in the carotid territory

12
CVS-System Wise 1700-by Sush and Team. 2016
Susmita, Asad, Manu, Saima, Zohaib, Savia, Shanu, Mona, Manisha, Sitara, Samreena, Sami and Komal

and are not severely disabled
should only be considered if carotid stenosis > 70% according ECST*
criteria or > 50% according to NASCET** criteria


*European Carotid Surgery Trialists' Collaborative Group
**North American Symptomatic Carotid Endarterectomy Trial

Reference
Dr Khalid/Rabia KEY- D
TIA Prophylaxis: Aspirin, clopidogrel and statin.
TIA ttt: Aspirin and dypiridamol


Q: 435. A 40yo woman suddenly collapsed and died. At the post-mortem autopsy,
it was found that there a bleed from a berry aneurysm from the circle of Willis.
In which space did the bleeding
occur?
a. Subarachnoid
b. Subdural
c. Extradural
d. Subparietal
e. Brain ventricles

Clincher(s)
A A
B
C
D
E
KEY A
Additional
Information
Reference
Dr Khalid/Rabia KEY- A.
Berry (or saccular) aneurysms are found in the circle of willis which is found in
the subarachnoid space. They are the most common form of cerebral
aneurysms. They present with sudden severe headache and gold standard for
diagnosis is CT. Gold standard for treatment is surgical clipping, done after
restoration of respiration and reduction of ICP. Berry aneurysms are often
associated with APCKD.

13
CVS-System Wise 1700-by Sush and Team. 2016
Susmita, Asad, Manu, Saima, Zohaib, Savia, Shanu, Mona, Manisha, Sitara, Samreena, Sami and Komal





Q: 439. A 72yo man is found to be not breathing in the CCU with the following
rhythm. What is the most likely dx?
a. SVT
b. VT
c. VF
d. Atrial fib
e. Atrial flutter
Post MI> VF
Clincher(s)
A
B
C
D
E
KEY C
Additional
Information
Reference
Dr Khalid/Rabia VFib- Chaotic depolarisation of ventricles. Atrial rate 60-100. Ventricular rate
400-600. Irregular. Ttt by immediate defibrillation
VTach- Sequence of 3 or more ventricular beats. Atrial rate 60-100. Ventricular
rate 110-250. Regular. Can progress to VFib and cardiac arrest. Ttt if pulse
present, cardioversion. If pulseless, defibrillation

14
CVS-System Wise 1700-by Sush and Team. 2016
Susmita, Asad, Manu, Saima, Zohaib, Savia, Shanu, Mona, Manisha, Sitara, Samreena, Sami and Komal



Q: 454. A 65yo male presents with dyspnea and palpitations. Exam:
pulse=170bpm, BP=120/80mmHg.
Carotid massage has been done as first instance. What is the next step of the
management?
a. Adenosine
b. Amilodipine
c. DC cardioversion
d. Lidocaine
e. Beta blocker

Clincher(s)
A A
B
C
D
E
KEY A (>150 bpm = SVT)
Additional Supraventricular tachycardia
Information

Whilst strictly speaking the term supraventricular tachycardia (SVT) refers to
any tachycardia that is not ventricular in origin the term is generally used in
the context of paroxysmal SVT. Episodes are characterised by the sudden
onset of a narrow complex tachycardia, typically an atrioventricular nodal re-
entry tachycardia (AVNRT). Other causes include atrioventricular re-entry
tachycardias (AVRT) and junctional tachycardias.

Acute management

vagal manoeuvres: e.g. Valsalva manoeuvre


intravenous adenosine 6mg (if not treated) 12mg 12mg:
contraindicated in asthmatics - verapamil is a preferable option
electrical cardioversion


Prevention of episodes

beta-blockers
radio-frequency ablation


Reference
Dr Khalid/Rabia

15
CVS-System Wise 1700-by Sush and Team. 2016
Susmita, Asad, Manu, Saima, Zohaib, Savia, Shanu, Mona, Manisha, Sitara, Samreena, Sami and Komal


Q: 461. A 55yo woman was found collapsed at home, paramedics revived her but
in the ambulance she had a Cardiac arrest and couldnt be saved. The
paramedics report tells that the woman was immobile lately due to hip pain
and that they found ulcers on the medial side of ankle. She had DM and was
on anti-diabetics. What is the cause of her death?
a. Acute MI
b. DKA
c. Pulmonary embolism
d. Acute pericarditis
e. Cardiac tamponade

Clincher(s)
A
B
C
D
E
KEY C
Additional thrombolysis is now recommended as the first-line treatment for
Information massive PE where there is circulatory failure (e.g. hypotension). Other
invasive approaches should be considered where appropriate facilities
exist


Reference
Dr Khalid/Rabia KEY- C
This is a case of collapse due to PE following DVT caused by the patients
immobilization due to hip pain.
Cardiac tamponade- Triad of hypotension, distended engorged neck veins, and
muffled JVP.
Pericarditis- Chest pain worse with inspiration and lying down, relieved by lying
forward.
No history supporting DKA or MI.



Q: 498. An ECG of an elderly lady who collapsed in the ED shows rapid ventricular
rate of 220bpm,QRS=140ms. What is the most probable dx?
a. Atrial fibrillation
b. VT
c. SVT
d. Mobitz type1 2nd degree heart block
e. Sinus tachycardia

Clincher(s)

16
CVS-System Wise 1700-by Sush and Team. 2016
Susmita, Asad, Manu, Saima, Zohaib, Savia, Shanu, Mona, Manisha, Sitara, Samreena, Sami and Komal

A
B
C
D
E
KEY B (QRS- n is 120) all others will have narrower QRS
Additional
Information
Reference
Dr Khalid/Rabia


Q: 504. A 58yo T1DM on anti-HTN therapy for 13yrs developed central chest pain
for 45 mins while driving a/w cold sweating and dyspnea. What is the single
most appropriate dx?
a. MI
b. Pericarditis
c. Pulmonary embolism
d. Costochondritis
e. Pneumothorax

Clincher(s)
A
B
C
D
E
KEY A
Additional
Information
Reference
Dr Khalid/Rabia characteristic central or epigastric chest pain radiating to the arms,
shoulders, neck, or jaw.
The pain is described as substernal pressure, squeezing, aching,
burning, or even sharp pain.
Radiation to the left arm or neck is common.
Chest pain may be associated with sweating, nausea, vomiting,
dyspnoea, fatigue, and/or palpitations.

Pericarditis chest pain: may be pleuritic. Is often relieved by sitting forwards


Pulmonary embolism pleuritic chest pain, dyspnoea and haemoptysis
Costochondritis Chest wall pain with a history of repeated minor trauma or
unaccustomed activity (eg, painting, moving furniture) is common

17
CVS-System Wise 1700-by Sush and Team. 2016
Susmita, Asad, Manu, Saima, Zohaib, Savia, Shanu, Mona, Manisha, Sitara, Samreena, Sami and Komal



Q: 505. A man was brought to the ED from a shopping mall after collapsing there.
He is conscious and answering questions now. His ECG shows irregular rhythm.
Your choice of inv:
a. CT
b. MRI
c. 24h ECG
d. Echo

Clincher(s)
A
B
C
D
E
KEY C
Additional
Information
Reference
Dr Khalid/Rabia The man had a syncopial attack ..the most valvular cause for it is aortic
stenosis which needs an Echo to diagnose it or if there is any other valvular
lesion or ventricular dysfunction go for echo just to exclude any structural
abnormalities.
holter- ecg is already said to be irregular. we already know that there is an
rhythm problem so no use to holter.
If in history something indicated towards TIA or stroke then CT or MRI would
be considered.



Q: 510. A 45yo manual worker presented with a 2h hx of chest pain radiating to
his left arm. His ECG is normal. What is the single most appropriate inv?
a. Cardiac enzymes
b. CXR
c. CT
d. ECG
e. V/Q scan

Clincher(s)
A
B
C
D
E
KEY A

18
CVS-System Wise 1700-by Sush and Team. 2016
Susmita, Asad, Manu, Saima, Zohaib, Savia, Shanu, Mona, Manisha, Sitara, Samreena, Sami and Komal

Additional
Information
Reference Pass Med
Dr Khalid/Rabia a. Cardiac enzymes
to rule out NSTEMI.

Non-ST-elevation ACS (NSTE-ACS): patients present with acute chest pain but
without persistent ST-segment elevation. The ECG shows persistent or
transient ST-segment depression or T-wave inversion, flat T waves, pseudo-
normalisation of T waves, or no ECG changes at presentation.

Management:
All patients should receive
aspirin 300mg
nitrates or morphine to relieve chest pain if required

Antithrombin treatment. Fondaparinux should be offered to patients who are


not at a high risk of bleeding and who are not having angiography within the
next 24 hours. If angiography is likely within 24 hours or a patients creatinine is
> 265 mol/l unfractionated heparin should be given.

Clopidogrel 300mg should be given to all patients and continued for 12
months.

Intravenous glycoprotein IIb/IIIa receptor antagonists (eptifibatide or
tirofiban) should be given to patients who have an intermediate or higher risk
of adverse cardiovascular events (predicted 6-month mortality above 3.0%),
and who are scheduled to undergo angiography within 96 hours of hospital
admission.

Coronary angiography should be considered within 96 hours of first admission
to hospital to patients who have a predicted 6-month mortality above 3.0%. It
should also be performed as soon as possible in patients who are clinically
unstable.



Q: 520. A 50yo man with DM suddenly develops persistent crushing central chest
pain radiating to the neck. What is the single most appropriate dx?
a. Angina
b Costochondritis (tietzs disease)
c. Dissecting aneurysm
d. MI
e. Pulmonary embolism

19
CVS-System Wise 1700-by Sush and Team. 2016
Susmita, Asad, Manu, Saima, Zohaib, Savia, Shanu, Mona, Manisha, Sitara, Samreena, Sami and Komal

Clincher(s)
A
B
C
D
E
KEY C
Additional More info should be given
Information
Reference
Dr Khalid/Rabia c. Dissecting aneurysm
Pain can radiate to back (classically described in questions) or to the neck as
well. MI is an important differential but usually MI in diabetics is silent one.
Angina does not radiate, costochondritis mostly have localised pain
In aortic dissection, pain is abrupt in onset and maximal at the time of onset. In
contrast, the pain associated with acute myocardial infarction starts slowly and
gains in intensity with time. It is usually more oppressive and dull.
Although tearing is the classical description, the pain is described as sharp
more often than tearing, ripping, or stabbing.

Investigations
Often the first problem is to distinguish aortic dissection from myocardial
infarction. Both conditions may exist if the dissection involves the coronary
ostium. For this reason, the electrocardiogram (ECG) is very important.
Best: MRI



Q: 1079. A 65yo man presents with dyspnea 3d after an MI. On auscultation he
has a pansystolic murmur at the apex radiating to the axilla. What is the most
likely dx?
a. Ruptured papillary muscle
b. Ventricular aneurysm
c. Pericarditis
d. Pericardial effusion
e. VSD

Clincher(s)
A
B
C
D
E
KEY A

20
CVS-System Wise 1700-by Sush and Team. 2016
Susmita, Asad, Manu, Saima, Zohaib, Savia, Shanu, Mona, Manisha, Sitara, Samreena, Sami and Komal

Additional Myocardial infarction: complications
Information

Patients are at risk of a number of immediate, early and late complications
following a myocardial infarction (MI).

Cardiac arrest

This most commonly occurs due to patients developing ventricular fibrillation
and is the most common cause of death following a MI. Patients are managed
as per the ALS protocol with defibrillation.

Cardiogenic shock

If a large part of the ventricular myocardium is damaged in the infarction the
ejection fraction of the heart may decrease to the point that the patient
develops cardiogenic shock. This is difficult to treat. Other causes of
cardiogenic shock include the 'mechanical' complications such as left
ventricular free wall rupture as listed below. Patients may require inotropic
support and/or an intra-aortic balloon pump.

Chronic heart failure

As described above, if the patient survives the acute phase their ventricular
myocardium may be dysfunctional resulting in chronic heart failure. Loop
diuretics such as furosemide will decrease fluid overload. Both ACE-inhibitors
and beta-blockers have been shown to improve the long-term prognosis of
patients with chronic heart failure.

Tachyarrhythmias

Ventricular fibrillation, as mentioned above, is the most common cause of
death following a MI. Other common arrhythmias including ventricular
tachycardia.

Bradyarrhythmias

Atrioventricular block is more common following inferior myocardial
infarctions.

Pericarditis

Pericarditis in the first 48 hours following a transmural MI is common (c. 10%
of patients). The pain is typical for pericarditis (worse on lying flat etc), a
pericardial rub may be heard and a pericardial effusion may be demonstrated
with an echocardiogram.

21
CVS-System Wise 1700-by Sush and Team. 2016
Susmita, Asad, Manu, Saima, Zohaib, Savia, Shanu, Mona, Manisha, Sitara, Samreena, Sami and Komal

Dressler's syndrome tends to occur around 2-6 weeks following a MI. The
underlying pathophysiology is thought to be an autoimmune reaction against
antigenic proteins formed as the myocardium recovers. It is characterised by a
combination of fever, pleuritic pain, pericardial effusion and a raised ESR. It is
treated with NSAIDs.

Left ventricular aneurysm

The ischaemic damage sustained may weaken the myocardium resulting in
aneurysm formation. This is typically associated with persistent ST elevation
and left ventricular failure. Thrombus may form within the aneurysm
increasing the risk of stroke. Patients are therefore anticoagulated.

Left ventricular free wall rupture

This is seen in around 3% of MIs and occurs around 1-2 weeks afterwards.
Patients present with acute heart failure secondary to cardiac tamponade
(raised JVP, pulsus paradoxus, diminished heart sounds). Urgent
pericardiocentesis and thoracotomy are required.

Ventricular septal defect

Rupture of the interventricular septum usually occurs in the first week and is
seen in around 1-2% of patients. Features: acute heart failure associated with a
pan-systolic murmur. An echocardiogram is diagnostic and will exclude acute
mitral regurgitation which presents in a similar fashion. Urgent surgical
correction is needed.

Acute mitral regurgitation

More common with infero-posterior infarction and may be due to ischaemia or
rupture of the papillary muscle. An early-to-mid systolic murmur is typically
heard. Patients are treated with vasodilator therapy but often require
emergency surgical repair.

Reference
Dr Khalid/Rabia Complications post MI
Cardiac arrest, unstable angina, bradycardia or heart block, tachyarrhythmias,
right ventricular failure, pericarditis, DVT and PE, systemic embolism, cardiac
tamponade, mitral regurgitation, VSD, late malignant vent arrhythmia,
dresslers syndrome, left ventricular aneurysm.

In this scenario, patient has presented to us post MI with complains of dysnea
(pulmonary edema). Auscultation reveals a pansystolic murmur at apex
radiating to axilla characteristic of MR murmur. MR post MI occurs as a result
of papillary muscle dysfunction (mild MR) or papillary muscle or chordal

22
CVS-System Wise 1700-by Sush and Team. 2016
Susmita, Asad, Manu, Saima, Zohaib, Savia, Shanu, Mona, Manisha, Sitara, Samreena, Sami and Komal

rupture or ischemia (severe MR).
Vsd will also present with pansystolic murmur but at lower left sternal edge.
Pericarditis and pericardial effusion with muffled heart sounds and pericardial
pain relieved by sitting forward.
Ventricular aneurysm occurs late after 4-6 weeks. Presents with LVF, angina,
recurrent VT or systemic embolization. Persistent ST segment elevation.
Treatment is excise and coagulate.

Q: 1087. A 70yo hypertensive white british man on thiazide diuretics needs a 2nd
drug to control his BP.
Which one of the following is the best choice for him?
a. Amlodipine (CCB)
b. Enapril (ACEi)
c. Propranolol (BB)
d. Increase dose of diuretic
e. Prazocin (Alpha blocker)

Clincher(s)
A
B
C
D
E
KEY B
Additional Hypertension: management
Information
This man is over 55yr, Ca channel or thiazide so he is on thiazide. Step 1 is
always ACEinh

NICE published updated guidelines for the management of hypertension in
2011. Some of the key changes include:

classifying hypertension into stages


recommending the use of ambulatory blood pressure monitoring
(ABPM) and home blood pressure monitoring (HBPM)
calcium channel blockers are now considered superior to thiazides
bendroflumethiazide is no longer the thiazide of choice


Blood pressure classification

This becomes relevant later in some of the management decisions that NICE
advocate.

Stage Criteria
Stage 1 Clinic BP >= 140/90 mmHg and subsequent ABPM daytime average o

23
CVS-System Wise 1700-by Sush and Team. 2016
Susmita, Asad, Manu, Saima, Zohaib, Savia, Shanu, Mona, Manisha, Sitara, Samreena, Sami and Komal

hypertension average BP >= 135/85 mmHg
Stage 2 Clinic BP >= 160/100 mmHg and subsequent ABPM daytime average
hypertension HBPM average BP >= 150/95 mmHg
Severe
Clinic systolic BP >= 180 mmHg, or clinic diastolic BP >= 110 mmHg
hypertension

Managing hypertension

Lifestyle advice should not be forgotten and is frequently tested in exams:

a low salt diet is recommended, aiming for less than 6g/day, ideally
3g/day. The average adult in the UK consumes around 8-12g/day of
salt. A recent BMJ paper* showed that lowering salt intake can have a
significant effect on blood pressure. For example, reducing salt intake
by 6g/day can lower systolic blood pressure by 10mmHg
caffeine intake should be reduced
the other general bits of advice remain: stop smoking, drink less
alcohol, eat a balanced diet rich in fruit and vegetables, exercise more,
lose weight


ABPM/HBPM >= 135/85 mmHg (i.e. stage 1 hypertension)

treat if < 80 years of age AND any of the following apply; target organ
damage, established cardiovascular disease, renal disease, diabetes or
a 10-year cardiovascular risk equivalent to 20% or greater


ABPM/HBPM >= 150/95 mmHg (i.e. stage 2 hypertension)

offer drug treatment regardless of age


For patients < 40 years consider specialist referral to exclude secondary
causes.

Step 1 treatment

patients < 55-years-old: ACE inhibitor (A)


patients > 55-years-old or of Afro-Caribbean origin: calcium channel
blocker (C)


Step 2 treatment

ACE inhibitor + calcium channel blocker (A + C)

24
CVS-System Wise 1700-by Sush and Team. 2016
Susmita, Asad, Manu, Saima, Zohaib, Savia, Shanu, Mona, Manisha, Sitara, Samreena, Sami and Komal


Step 3 treatment

add a thiazide diuretic (D, i.e. A + C + D)


NICE now advocate using either chlorthalidone (12.5-25.0 mg once
daily) or indapamide (1.5 mg modified-release once daily or 2.5 mg
once daily) in preference to a conventional thiazide diuretic such as
bendroflumethiazide


NICE define a clinic BP >= 140/90 mmHg after step 3 treatment with optimal or
best tolerated doses as resistant hypertension. They suggest step 4 treatment
or seeking expert advice

Step 4 treatment A+C+D+B

consider further diuretic treatment


if potassium < 4.5 mmol/l add spironolactone 25mg od
if potassium > 4.5 mmol/l add higher-dose thiazide-like diuretic
treatment
if further diuretic therapy is not tolerated, or is contraindicated or
ineffective, consider an alpha- or beta-blocker


Patients who fail to respond to step 4 measures should be referred to a
specialist. NICE recommend:

If blood pressure remains uncontrolled with the optimal or maximum tolerated
doses of four drugs, seek expert advice if it has not yet been obtained.

Blood pressure targets

ABPM /
Clinic BP
HBPM
140/90
Age < 80 years 135/85 mmHg
mmHg
150/90
Age > 80 years 145/85 mmHg
mmHg

New drugs

Direct renin inhibitors

e.g. Aliskiren (branded as Rasilez)


by inhibiting renin blocks the conversion of angiotensinogen to
angiotensin I
no trials have looked at mortality data yet. Trials have only investigated

25
CVS-System Wise 1700-by Sush and Team. 2016
Susmita, Asad, Manu, Saima, Zohaib, Savia, Shanu, Mona, Manisha, Sitara, Samreena, Sami and Komal

fall in blood pressure. Initial trials suggest aliskiren reduces blood
pressure to a similar extent as angiotensin converting enzyme (ACE)
inhibitors or angiotensin-II receptor antagonists
adverse effects were uncommon in trials although diarrhoea was
occasionally seen
only current role would seem to be in patients who are intolerant of
more established antihypertensive drugs


Reference
Dr Khalid/Rabia Hypertension management
If age is greater than 55, or black patient of any age: ca channel blocker or
thiazide (C/D)
If age is less than 55: ace inhibitor(A)
B blocker to be considerd in young, pregnancy or if increased sympathetic
drive
Combination therapy
Ace inhibitor plus ca channel blocker or thiazide(A+C/D)
3 drugs needed: ace inhibitor plus ca channel blocker plus diuretic (A+C+D)
4th drug needed. Add higher dose diuretic or b blocker or alpha
blocker(A+C+D+B)


Q: 1089. A 62yo man has slow palpitations and the following ECG. What is the
most likely dx?
a. Sinus bradycardia
b. 1st degree heart block
c. Mobitz type 1 block
d. Mobitz type 2 block
e. Complete heart block

Clincher(s)
A
B
C
D
E
KEY C
Additional
Information
Reference
Dr Khalid/Rabia Progressive prolongation of PR interval followed by missed beat is indicative of
MOBITZ type 1 block.
Bradycardia is to be treated with atropine. If not controlled, then temporary
pacing may be required.

26
CVS-System Wise 1700-by Sush and Team. 2016
Susmita, Asad, Manu, Saima, Zohaib, Savia, Shanu, Mona, Manisha, Sitara, Samreena, Sami and Komal

Q: 1177. A white English man with a past hx of MI is a known HTN and DM. He is
currently on aspirin, statin and metformin. What would you add to the tx?
a. ACEiz
b. Diuretic
c. Insulin
d. Beta blocker
e. CCB

Clincher(s)
A
B
C
D
E
KEY A
Additional
Information
Reference
Dr Khalid/Rabia a. ACEi (cardioprotective)
ACEI should even be added in patients who are non hypertensive as it reduces
morbidity and mortality post MI.
ACE inhibitors are also used to treat diabetic nephropathy and have a role in
secondary prevention of ischaemic heart disease.

Q: 1494. A 72yo woman who is taking loop diuretics for left ventricular failure.
She now is suffering from
palpitations and muscle weakness. What is the electrolyte imbalance found?
a. Na+=130mmol/L, K+=2.5mmol/L
b. Na+=130mmol/L, K+=5.5mmol/L
c. Na+=140mmol/L, K+=4.5mmol/L
d. Na+=150mmol/L, K+=3.5mmol/L
e. None

Clincher(s)
A
B
C
D
E
KEY A (hyponatrimia and hypokalemia)
Additional 135-145 N for Na
Information
Reference
Dr Khalid/Rabia Loop Diuretics
loop diuretics act by inhibiting the Na-K-Cl cotransporter (NKCC) in the thick
ascending limb of the loop of Henle, reducing the absorption of NaCl.

27
CVS-System Wise 1700-by Sush and Team. 2016
Susmita, Asad, Manu, Saima, Zohaib, Savia, Shanu, Mona, Manisha, Sitara, Samreena, Sami and Komal

There are two variants of NKCC; loop diuretics act on NKCC2, which is more
prevalent in the kidneys.
Indications= heart failure: both acute (usually intravenously) and chronic
(usually orally), resistant hypertension, particularly in patients with renal
impairment.
Adverse effects= hypotension, hyponatremia,
hypokalemia,hypocalcemia,hypochloremic alkalosis, hyperglycemia (less
common with thiazides), renal impairment( from dehydration+direct toxic
effect) and gout


Q: 1505. A pt came to the hosp with a complaint of severe chest pain lasting for
>1h. Following ECG test, pt revealed to have ST depression. He was already on
aspirin. What is the most specific tx for this pt?
a. GTN (next)
b. Simvastatin
c. Clopidogrel
d. BBlocker
e. LMWH
OHCM- 811> ANABL
Clincher(s)
A
B
C
D
E
KEY LMWH
Additional Management of STEMI
Information
The 2008 British Thoracic Society oxygen therapy guidelines have now been
widely adopted and oxygen should only be given if the oxygen saturations are
< 94%

Sublingual glyceryl trinitrate and intravenous morphine + metoclopramide
should be given to help relieve the symptoms.

Aspirin 300mg should be given to all patients (unless contraindicated).
Reference
Dr Khalid/Rabia Answer: Key says B but i think A (should be E- most specific)
Non-ST-elevation ACS (NSTE-ACS): patients present with acute chest pain but
without persistent ST-segment elevation. The ECG shows persistent or
transient ST-segment depression or T-wave inversion, flat T waves, pseudo-
normalisation of T waves, or no ECG changes at presentation.

NSTE-ACS is further divided into:
Unstable angina: normal troponin levels.

28
CVS-System Wise 1700-by Sush and Team. 2016
Susmita, Asad, Manu, Saima, Zohaib, Savia, Shanu, Mona, Manisha, Sitara, Samreena, Sami and Komal


Non-ST-elevation MI (NSTEMI): a rise in troponin levels.
The presentation of unstable angina and NSTEMI may be indistinguishable, and
also indistinguishable from acute STEMI.

NSTE-ACS can present in a variety of ways, including:[2]
Prolonged (longer than 20 minutes) anginal pain at rest.
New-onset angina with limitation of daily activities.
Recent destabilisation of previously stable angina, with moderate or severe
limitation of daily activities.
Post-myocardial infarction angina.

In unstable angina (and non-Q wave infarction), the ECG typically shows T-
wave inversion or ST-segment depression, but the ECG may be normal if some
time has elapsed since the last episode of pain.

Cardiac enzymes:
Within the first 6 hours, the sensitivity of troponins is superior to CK-MB for
the detection of myocardial infarction.
Troponin I and T become detectable in serum 3-6 hours after infarction, peak
at 12-24 hours, and remain raised for up to 14 days.
Troponins are therefore usually tested 6 and 12 hours after the onset of pain.
In patients with unstable angina, minor troponin elevations may identify
patients at risk for subsequent cardiac events and death. Elevated troponin
levels indicate an increased risk of mortality in both the short term and long
term. Patients with chest pain and elevated troponin levels should remain in
hospital for further assessment, including an inpatient coronary angiogram.

The National Institute for Health and Care Excellence (NICE) recommends the
Global Registry of Acute Cardiac Events (GRACE) risk score.

Immediate management of a suspected ACS
Arrange urgent hospital admission (phone 999/112/911).
Resuscitation as required.
Pain relief: GTN and/or an intravenous opioid (use an antiemetic with opioids).
Single loading dose of 300 mg aspirin unless the person is allergic.
A resting 12-lead ECG - but don't delay transfer to hospital.
Assess oxygen saturation, using pulse oximetry before hospital admission if
possible. Give oxygen if oxygen saturation (SpO2) is less than 94% with no risk
of hypercapnic respiratory failure; aim for SpO2 of 94-98% (aim for 88-92% for
people with chronic obstructive pulmonary disease).


Q: 1511. A 27yo man presents to the ED with 2d hx of severe headache and
pyrexia (38.9C). CT: petechial hemorrhage in the temporal and inf frontal
lobes. What is the most likely dx?

29
CVS-System Wise 1700-by Sush and Team. 2016
Susmita, Asad, Manu, Saima, Zohaib, Savia, Shanu, Mona, Manisha, Sitara, Samreena, Sami and Komal

a. Brain abscess
b. Meningococcal meningitis
c. Cerebral malaria
d. Herpes simplex encephalitis
e. New variant CID

Clincher(s)
A
B
C
D D
E
KEY
Additional Answer: D
Information Herpes simplex encephalitis (HSE) is recognised worldwide as the most
frequent infectious encephalitis.
In children older than 3 months and in adults: HSE is usually caused by herpes
simplex virus type 1 (HSV-1) and is localised to the temporal and frontal lobes.
In neonates: HSE is usually caused by herpes simplex virus type 2 (HSV-2)
acquired at the time of delivery, and brain involvement is generalised.
Other herpes viruses may cause encephalitis but much less frequently than
HSV. However, cytomegalovirus (CMV) encephalitis should be considered in
those with immunodeficiency.


Reference
Dr Khalid/Rabia
Q: 1512. A 44yo woman with memory loss, poor concentration and inability to
recognize household projects. She has right-handed involuntary writhing
movement. There is strong fam hx of similar complain. What is the single most
likely dx?
a. Picks dementia
b. Wilsonss disease
c. Huntingtons disease
d. HIV associated dementia
e. Fronto-temporal dementia

Clincher(s)
A
B
C
D
E
KEY C
Additional
Information

30
CVS-System Wise 1700-by Sush and Team. 2016
Susmita, Asad, Manu, Saima, Zohaib, Savia, Shanu, Mona, Manisha, Sitara, Samreena, Sami and Komal

Reference
Dr Khalid/Rabia Answer: C
Huntington's disease is associated with cell loss within the basal ganglia and
cortex. It is an autosomal-dominant, progressive neurodegenerative disorder
with a distinct phenotype, including chorea and dystonia, incoordination,
cognitive decline, and behavioural difficulties.[1] Huntington's disease was first
described by George Huntington in 1872. The disease is associated with
increases in the length of a cysteine-adenosine-guanine (CAG) triplet repeat
present in a gene called 'huntingtin' located on chromosome
4p16.3.Huntington's disease is the most common genetic cause of chorea.
Signs and Symptoms:
The mean age at onset of symptoms is 30-50 years.
In some cases symptoms start before the age of 20 years with behavioural
disturbances and learning difficulties at school (juvenile Huntington's disease)
Early signs may be personality change, self-neglect, apathy with clumsiness,
fidgeting with fleeting facial grimaces.
Huntington's disease then leads to progressive chorea, rigidity and dementia.
It is frequently associated with seizures.
Chorea is initially mild but may be severe and cause uncontrollable limb
movements.
As the disease progresses, chorea is gradually replaced by dystonia and
Parkinsonian features.
Dysarthria, dysphagia and abnormal eye movements are common. There may
also be other movement disorders - eg, tics and myoclonus.
Huntington's disease is associated with increasing depression, bradykinesia,
cognitive impairment and aggression as the disease progresses.Behavioural
difficulties include apathy or lack of initiative, dysphoria, irritability, agitation
or anxiety, poor self-care, poor judgment and inflexibility.[1]Late features
include spasticity, clonus, supranuclear gaze palsy and extensor plantar
responses. The rate of cognitive decline is very variable.
Investigations:
MRI and CT scans in moderate-to-severe Huntington's disease show a loss of
striatal volume and increased size of the frontal horns of the lateral ventricles,
but scans are usually unhelpful for diagnosis of early disorder.
If genetic testing is considered then extensive genetic counselling in a
specialist unit is required in view of the implications of an untreatable, familial,
progressive, neurodegenerative disease.
Treatment:
Current drug therapy has no effect on the progression of disability.
Hyperkinesias and psychiatric symptoms may respond well to
pharmacotherapy, but neuropsychological deficits and dementia remain
untreatable.[8]
Patients, their families and carers require a great deal of physical and
emotional support.
Chorea:
Benzodiazepines, valproic acid, dopamine-depleting agents (eg, tetrabenazine)

31
CVS-System Wise 1700-by Sush and Team. 2016
Susmita, Asad, Manu, Saima, Zohaib, Savia, Shanu, Mona, Manisha, Sitara, Samreena, Sami and Komal

and neuroleptics may be useful
Q: 1513. A 54yo man has collapsed suddenly following a headache. He has
hypertension and takes warfarin for prosthetic heart valve. GCS=4 and dilated
left pupil. What is the single most likely dx?
a. Ant circulation stroke
b. Post circulation stroke
c. Intracerebral hemorrhage
d. Intracerebellar haemorrhage (no cerebellar signs)
e. Pontine haemorrhage (pinpoint)

Clincher(s)
A
B
C
D
E
KEY C
Additional
Information
Reference
Dr Khalid/Rabia Answer: C
Intracerebral hemorrhage occurs when a diseased blood vessel within the
brain bursts, allowing blood to leak inside the brain. (The name means within
the cerebrum or brain). The sudden increase in pressure within the brain can
cause damage to the brain cells surrounding the blood. If the amount of blood
increases rapidly, the sudden buildup in pressure can lead to unconsciousness
or death. Intracerebral hemorrhage usually occurs in selected parts of the
brain, including the basal ganglia, cerebellum, brainstem, or cortex.

Q: 1516. A 38yo woman with hemophilia who received several blood transfusions
a few years ago presents with irritability and increasing memory deficit. She is
unable to speak properly. He is on anti-TB tx. What is the single most likely dx?
a. Creutzfeldt Jacob disease
b. Drug toxicity
c. Vascular dementia
d. HIV associated dementia
e. Space occupying lesion

Clincher(s)
A
B
C
D
E
KEY HIV associated dementia (TB in clincher)
Additional

32
CVS-System Wise 1700-by Sush and Team. 2016
Susmita, Asad, Manu, Saima, Zohaib, Savia, Shanu, Mona, Manisha, Sitara, Samreena, Sami and Komal

Information
Reference
Dr Khalid/Rabia Answer: D
Repeated BT ---> patient got HIV inf ( immunocompromised)---> developed TB
--> HIV induced dementia
The use of plasma-derived factor VIII, before the availability of recombinant
products, led to infection with HIV, hepatitis B virus (HBV) and hepatitis C virus
(HCV) in many haemophiliacs. One case of likely transmission of variant
Creutzfeldt-Jakob disease (vCJD) by UK factor VIII concentrates has been
reported in an elderly haemophilic patient in the UK. The recent report of a
blood test that may be used to detect vCJD has raised the possibility of a new
way to identify infected individuals, perhaps even before the onset of clinical
symptoms
Q: 1520. An 82yo woman has been admitted from a nursing home with dense
hemiplegia and homonymous hemianopia. She is dysphasic. What vessel is
most likely to be involved?
a. Ant cerebral artery
b. Mid cerebral artery (there is homonomous hemianopia + dysphasia)
c. Post cerebral artery (there is homo hemianopia but also cortical blindness-
cannot perceive they are blind)
d. Internal carotid artery
e. Post inf cerebellar artery

Clincher(s)
A
B
C
D
E
KEY
Additional
Information
Reference Answer: B
Areas supplied by the middle cerebral artery include:

The bulk of the lateral surface of the hemisphere; except for the superior inch
of the frontal and parietal lobe (anterior cerebral artery), and the inferior part
of the temporal lobe.
Superior division supplies latero inferior frontal lobe (location of Broca's area
i.e. language expression)
Inferior division supplies lateral temporal lobe (location of Wernicke's area i.e.
language comprehension)
Deep branches supply the basal ganglia as well as the internal capsule.
Occlusion of the middle cerebral artery results in Middle cerebral artery
syndrome, potentially showing the following defects:

33
CVS-System Wise 1700-by Sush and Team. 2016
Susmita, Asad, Manu, Saima, Zohaib, Savia, Shanu, Mona, Manisha, Sitara, Samreena, Sami and Komal

Paralysis (-plegia) or weakness (-paresis) of the contralateral face and arm
(faciobrachial)
Sensory loss of the contralateral face and arm.
Damage to the dominant hemisphere (usually the left hemisphere) results in
aphasia i.e. Broca's or Wernicke's
Damage to the non-dominant hemisphere (usually the right hemisphere)
results in contralateral neglect syndrome
Large MCA infarcts often have dviation conjugue, a gaze preference towards
the side of the lesion, especially during the acute period. Contralateral
homonymous hemianopsia is often present.


Dr Khalid/Rabia


672
673
678
697
724
730
740

756
Q:670 A 55yo man returns for routine follow up 6wks after a MI. He gets breathless
when walking
uphill. His ECG shows ST elevation in leads V1, V2, V3 and V4. What is the
single most likely
explanation for the abnormal investigation?
a. Heart block
b. Right ventricular strain
c. Atrial thrombus
d. Left ventricular aneurysm
e. Dresslers syndrome



Clincher(s)
A
B
C
D
E
KEY D
Additional If ST elevation occur after few weeks ,
Information
Reference

34
CVS-System Wise 1700-by Sush and Team. 2016
Susmita, Asad, Manu, Saima, Zohaib, Savia, Shanu, Mona, Manisha, Sitara, Samreena, Sami and Komal

Dr Khalid/Rabia d. Left ventricular aneurysm
S t elevation 6 weeks after mi...with no other major symptoms... Also a
continuous St elevation and a history of mi points towards an aneurysm
persistent ST elevation after few months of acute MI,D/D-1.ventricular
aneurysm 2.underkinetic wall motion disorder. N.B -contrary to sounding
fatal,there is neither predisposition nor any association of cardiac rupture in
ventricular aneurysm.
Ventricular aneurysm occurs at the site of previous STEMI. V1-V4 involvement
indicates previous anteroseptal MI ( caused by LAD obstruction ) . This
localized involvement of leads almost excludes Dressler's syndrome where
pericarditis causes ST elevation in all but aVR leads.
Left ventricular aneurysm
The ischaemic damage sustained may weaken the myocardium resulting in
aneurysm formation. This is typically associated with persistent ST elevation
and left ventricular failure. Thrombus may form within the aneurysm
increasing the risk of stroke. Patients are therefore anticoagulated.

Dressler's syndrome tends to occur around 2-6 weeks following a MI. The
underlying pathophysiology is thought to be an autoimmune reaction against
antigenic proteins formed as the myocardium recovers. It is characterised by a
combination of fever, pleuritic pain, pericardial effusion and a raised ESR. It is
treated with NSAIDs.

Q:672 A 76yo man suddenly collapsed and died. At post mortem exam, a
retroperitoneal hematoma due to ruptured aortic aneurysm was noted. What
is the most likely underlying cause of the aortic aneurysm?

a. Atheroma
b. Cystic medial necrosis
c. Dissecting aneurysm
d. Polyarteritis nodosa
e. Syphilis (involves root of aorta)

Clincher(s)
A
B
C
D
E
KEY
Additional
Information
Reference
Dr Khalid/Rabia a. Atheroma
risk factors include:

35
CVS-System Wise 1700-by Sush and Team. 2016
Susmita, Asad, Manu, Saima, Zohaib, Savia, Shanu, Mona, Manisha, Sitara, Samreena, Sami and Komal

Severe atherosclerotic damage of the aortic wall; however, new

evidence suggests this is not the only factor, and aneurysmal
disease is probably a distinct arterial pathology.
Family history - there are probably strong genetic factors. About
15% of first-degree relatives of a patient with an AAA, mainly men,
will develop an aneurysm.[6]
Tobacco smoking is an important factor.
Male sex.
Increasing age.
Hypertension.
Chronic obstructive pulmonary disease.
Hyperlipidaemia.
In population-based studies, people with diabetes have a lower incidence of
aneurysms than people without diabetes

Q:673 A 33yo male came to the hospital with complaint of occasional left sided chest
pain that lasts <30mins, following exercise, which relieves upon taking rest.

What is the most probable dx?
a. Unstable angina
b. Decubitus angina
c. Stable angina
d. Coronary spasm
e. MI

Clincher(s)
A
B
C
D
E
KEY C
Additional
Information
Reference
Dr Khalid/Rabia c. Stable angina

Stable angina is when the pain is precipitated by predictable factors -
usually exercise.
Unstable angina: angina occurs at any time and should be considered
and managed as a form of acute coronary syndrome.

Decubitus angina occurs when the patient lies down. It is usually a


complication of cardiac failure due to the strain on the heart resulting from the
increased intravascular volume.
Patients usually have severe coronary artery disease.

36
CVS-System Wise 1700-by Sush and Team. 2016
Susmita, Asad, Manu, Saima, Zohaib, Savia, Shanu, Mona, Manisha, Sitara, Samreena, Sami and Komal



A full 12-lead ECG may show some ischaemic changes but a normal ECG does
not rule out a diagnosis of angina. Changes on a resting 12-lead ECG that are
consistent with CAD include:
Pathological Q waves.
Left bundle branch block (LBBB).
ST-segment and T-wave abnormalities (eg flattening or inversion).

when an attack of angina occurs, they should:


Stop what they are doing and rest.
Use glyceryl trinitrate (GTN) spray or tablets as instructed.
Take a second dose of GTN after 5 minutes if the pain has not eased.
Take a third dose of GTN after a further 5 minutes if the pain has still
not eased.
Call 999/112/911 for an ambulance if the pain has not eased after
another 5 minutes (ie 15 minutes after onset of pain), or earlier if the
pain is intensifying or the person is unwell.

Medication
all patients should receive aspirin and a statin in the absence of any
contraindication
sublingual glyceryl trinitrate to abort angina attacks
NICE recommend using either a beta-blocker or a calcium channel
blocker first-line based on 'comorbidities, contraindications and the
person's preference'
if a calcium channel blocker is used as monotherapy a rate-limiting one
such as verapamil or diltiazem should be used. If used in combination
with a beta-blocker then use a long-acting dihydropyridine calcium-
channel blocker (e.g. modified-release nifedipine). Remember that
beta-blockers should not be prescribed concurrently with verapamil
(risk of complete heart block)
if there is a poor response to initial treatment then medication should
be increased to the maximum tolerated dose (e.g. for atenolol 100mg
od)
if a patient is still symptomatic after monotherapy with a beta-blocker
add a calcium channel blocker and vice versa

if a patient is on monotherapy and cannot tolerate the addition of a
calcium channel blocker or a beta-blocker then consider one of the
following drugs: a long-acting nitrate, ivabradine, nicorandil or
ranolazine
if a patient is taking both a beta-blocker and a calcium-channel blocker
then only add a third drug whilst a patient is awaiting assessment for

37
CVS-System Wise 1700-by Sush and Team. 2016
Susmita, Asad, Manu, Saima, Zohaib, Savia, Shanu, Mona, Manisha, Sitara, Samreena, Sami and Komal

PCI or CABG


Q:678 A 62yo man who has recently had flu-like illness has woken to find his speech
altered.
Movement of his eyelids and lips are weak on the right side. Exam: normal.
Which anatomical
site is most likely to be affected?
a. Facial nerve
b. Hypoglossal nerve
c. Oculomotor nerve
d. Trigeminal nerve
e. Glossopharyngeal nerve

a. Facial nerve
bell's palsy


Clincher(s)
A
B
C
D
E
KEY A
Additional
Information
Reference
Dr Khalid/Rabia
Q: 697
A 72yo man presents to the ED with chest pain. The following ECG was taken
What is the most likely dx?

38
CVS-System Wise 1700-by Sush and Team. 2016
Susmita, Asad, Manu, Saima, Zohaib, Savia, Shanu, Mona, Manisha, Sitara, Samreena, Sami and Komal


a. Anterior MI
b. Inferior MI
c. Lateral MI
d. Posterior MI
e. NSTEMI

Clincher(s)
A
B
C
D
E
KEY B
Additional L 2, 3 and AVF has ST elevation
Information
Reference
Dr Khalid/Rabia
Q:724 A pt on HTN drugs develops hyperkalemia. Which anti-HTN is likely to cause it?
a. Ramipril (others will cause hypokalemia)
b. Lorsartan
c. Thiazide
d. Nifedipine
e. Furosemide

Just an 5 effect of ace inhibitors. Diuretics cause hypokalemia

Clincher(s)
A
B
C
D
E
KEY
Additional
Information
Reference
Dr Khalid/Rabia
Q:730 730. A 55yo man presents with HTN. He complains of headache and visual
disturbances. He also reports itching after a hot bath and burning sensation in
finger and toes. His face is flushed red. PE: mild splenomegaly. Inv:
Hgb=20g/dl, WBC=20, plt=500, EPO normal. What is the likely dx?
a. Myelofibrosis (anemia)
b. Polycythemia rubra vera (Hb-high)
c. Essential thrombocythemia (platelet increases)

39
CVS-System Wise 1700-by Sush and Team. 2016
Susmita, Asad, Manu, Saima, Zohaib, Savia, Shanu, Mona, Manisha, Sitara, Samreena, Sami and Komal

d. CML (not so high Hb, - ameia later, WBC high: > 50K
e. CLL (not so high Hb anemia later WBC 10-15 K lympho count
increased)

Increased haemoglobin, flushing, itching, burning sensation, visual disturbance
(amaurosis fugax) all are symptoms of polycythemia rubra vera.


Clincher(s)
A
B
C
D
E
KEY
Additional
Information
Reference
Dr Khalid/Rabia
Q:740 A 25yo man presents with hx of breathlessness. A transthoracic echo reveals a
patent foramen ovale. What diagnostic inv would you do for a patent foramen
ovale?
a. Transesophageal echo (invasive)
b. Bubble echo (type of trans thoracic echo or trans eso phageal)

c. Transthoracic echo (

d. ECG

A, b, c> bubble eco (which is a contrast)
Clincher(s)
A
B
C
D
E
KEY B
Additional All rely on the injection of bubble contrast (a mixture of saline, air and blood)
Information into a peripheral vein combined with procedures to increase pressure in the
right atrium (Valsalva release, sniff, etc.). With TOE and TTE, visualisation of
movement of bubbles from the right atrium (RA) into the left atrium (LA)
within three to five cardiac cycles demonstrates the presence of a right-to-left
intracardiac shunt.

40
CVS-System Wise 1700-by Sush and Team. 2016
Susmita, Asad, Manu, Saima, Zohaib, Savia, Shanu, Mona, Manisha, Sitara, Samreena, Sami and Komal

TCD demonstrates right to left shunting (not necessarily intracardiac) by
detecting the presence of contrast in the middle cerebral artery as hits. TOE
has previously been considered the gold standard investigation as it has a
high sensitivity and specificity in the diagnosis of PFO.3,4 It is, however, an
invasive investigation that requires sedation and causes discomfort to the
patient, and limits the patients ability to perform an adequate Valsalva. TCD is
a non-invasive and sensitive investigation with a high sensitivity in the
detection of PFO,5 but cannot differentiate an intracardiac from a pulmonary
shunt. TTE is also non-invasive and, with the development of second harmonic
imaging, studies have suggested that it may have a sensitivity equivalent to
TOE in the detection of PFO and is more cost-effective.6,7 It is for this reason
that TTE with bubble contrast has become the screening investigation of
choice for a PFO at most centres.

http://bjcardio.co.uk/2011/10/patent-foramen-ovale-diagnosis-indications-
for-closure-and-complications/
Reference
Dr Khalid/Rabia Patent foramen ovale (PFO) is a hole between the left and right atria of the
heart. This hole exists in everyone before birth, but usually closes shortly after
being born. PFO is what the hole is called when it fails to close naturally after a
baby is born.

This hole allows blood to bypass the fetal lungs, which cannot work until they
are exposed to air. When a newborn enters the world and takes its first breath,
the foramen ovale closes, and within a few months it has sealed completely in
about 75 percent of us. When it remains open, it is called a patent foramen
ovale. For the vast majority of the millions of people with a PFO, it is not a
problem, even though blood is leaking from the right atrium to the left.
Problems can arise when that blood contains a blood clot.

Diagnosis of PFO

Patent Foramen Ovale can only be detected by specialized testing. If


suspected, your doctor may order:
Echocardiogram
Transesophageal echo (TEE) an ultrasound test used to visualize the
heart and defect, where an imaging probe with a camera is placed into
the esophagus
Bubble study In some cases during an echo or TEE, the doctor may
inject agitated saline through an IV in your arm with bubbles. During
the test, the doctor can watch the echo to see if bubbles pass from the
right to the left side of the heart to reveal the PFO.

Q:756 A woman presents with hx of falls, becomes pale and clumsy. She is
hypertensive and takes atenolol, bendroflumethiazide and amlodipine. What
inv is needed?
a. 24h ECG

41
CVS-System Wise 1700-by Sush and Team. 2016
Susmita, Asad, Manu, Saima, Zohaib, Savia, Shanu, Mona, Manisha, Sitara, Samreena, Sami and Komal

b. 24 h BP monitoring
c. ECG
d. Echo
e. CT head


Clincher(s)
A
B
C
D
E
KEY B (postural Hypo)
Additional Ace and thiazide: cause postural htn
Information
Reference
Dr Khalid/Rabia
Q: 684. A 56yo man presents to the ED with chest pain. The following ECG was
taken. What is the most likely dx?



a. Anterior MI
b. Inferior MI
c. Lateral MI
d. Posterior MI
e. NSTEMI

a. Anterior MI


Clincher(s)
A
B
C
D
E

42
CVS-System Wise 1700-by Sush and Team. 2016
Susmita, Asad, Manu, Saima, Zohaib, Savia, Shanu, Mona, Manisha, Sitara, Samreena, Sami and Komal

KEY A (v2-4 elevation)
Additional
Information
Reference
Dr Khalid/Rabia

ECG changes Coronary artery


Anterosept V1-V4 Left anterior descending
al

Inferior II, III, aVF Right coronary

Anterolater V4-6, I, aVL Left anterior descending or left


al circumflex

Lateral I, aVL +/- V5-6 Left circumflex

Posterior Tall R waves V1-2 Usually left circumflex, also right


coronary



Q:1181 A 51yo man had a MI a few days ago. He developed breathlessness. Echo was
done and showed a pansystolic murmur. What can be the cause of this
symptom?
a. Ruptured papillary muscle
b. Acute pericarditis
c. Dresslers syndrome
d. Malignant VT
e. Ventricular aneurysm



Clincher(s) MI few days ago,echo pansystolic murmur
A STRUCTURE:
There are five total papillary muscles in the heart, three in the right ventricle

43
CVS-System Wise 1700-by Sush and Team. 2016
Susmita, Asad, Manu, Saima, Zohaib, Savia, Shanu, Mona, Manisha, Sitara, Samreena, Sami and Komal

and two in the left. The anterior, posterior, and septal papillary muscles of the
right ventricle each attach via chordae tendineae to the tricuspid valve. The
anterior and posterior papillary muscles of the left ventricle attach via chordae
tendineae to the mitral valve.
FUNCTION:
The papillary muscles of both the right and left ventricles begin to contract
shortly before ventricular systole and maintain tension throughout.[1] This
prevents regurgitationbackward flow of ventricular blood into the atrial
cavitiesby bracing the atrioventricular valves against prolapsebeing forced
back into the atria by the high pressure in the ventricles.
B Pericarditis presents with fever,pericardial friction rub.cardiac temponade.
C Dresslers syndrome: This develops 210wks after an MI, heart surgery (or
even pacemaker insertion). It is thought that myocardial injury stimulates
formation ofauto antibodies against heart muscle. Symptoms: Recurrent fever
and chest pain pleural or pericardial rub (from serositis). Cardiac tamponade
may occur, so avoid anticoagulants. : Aspirin, NSAIDS or steroids

Has both pericarditis and pleural effusion and pleuritis (3 Ps)

Pansystolics murmur: in REGURGITATION
D
E An aneurysm is a section of defective wall that bulges outward. A ventricular
aneurysm is a defect in the left (or right) ventricle of the heart, usually
produced by transmural infarctio.VA can lead to ventricular
arrhythmia,emboli,ventricular failiure and eventually death.
KEY A
Additional Acute MR leads to rapid pulmonary oedema which is life-threatening
Information and requires emergency valve repair.
Chronic MR is well tolerated but dilatation of the left ventricle
eventually causes heart failure and breathlessness.
Acute MR due to papillary muscle rupture should be considered in
patients presenting with acute pulmonary oedema or shock following
an acute myocardial infarction. However, the murmur may be soft or
inaudible.[1]


Reference
Dr Khalid/Rabia Post MI- MR is the commonest complication> due to papillary muscle rupture
due to MI/ischemia> backflow.

Acute mitral regurgitation
More common with infero-posterior infarction and may be due to ischaemia or
rupture of the papillary muscle. An early-to-mid systolic murmur is typically
heard. Patients are treated with vasodilator therapy but often require
emergency surgical rep

44
CVS-System Wise 1700-by Sush and Team. 2016
Susmita, Asad, Manu, Saima, Zohaib, Savia, Shanu, Mona, Manisha, Sitara, Samreena, Sami and Komal

In acute pericarditis


Q:1184 A 72yo man presents to the ED with chest pain. The following ECG was taken.
What is the most likely dx?










a. Anterior MI
b. Inferior MI
c. Lateral MI
d. Posterior MI
e. NSTEMI

Clincher(s)
A
B ST elevation in leads II,III, AVF
C
D
E
KEY B
Additional ECG Manifestations of Acute Myocardial Ischaemia
Information
ST elevation
New ST elevation at the J-point in two contiguous leads with the cut-off
points: 0.2 mV in men or 0.15 mV in women in leads V2V3 and/or
0.1 mV in other leads.
ST depression and T-wave changes.
New horizontal or down-sloping ST depression >0.05 mV in two
contiguous leads; and/or T inversion 0.1 mVin two contiguous leads

45
CVS-System Wise 1700-by Sush and Team. 2016
Susmita, Asad, Manu, Saima, Zohaib, Savia, Shanu, Mona, Manisha, Sitara, Samreena, Sami and Komal

with prominent R-wave or R/S ratio 1

A study using MRI to diagnose myocardial infarction has shown that more
emphasis on ST segment depression could greatly improve the yield of the ECG
in the diagnosis of myocardial infarction (sensitivity increase from 50% to 8

table: above pat info

46
CVS-System Wise 1700-by Sush and Team. 2016
Susmita, Asad, Manu, Saima, Zohaib, Savia, Shanu, Mona, Manisha, Sitara, Samreena, Sami and Komal


Reference OHCM, patient.info
Dr Khalid/Rabia


Q:1199 A 28yo man presents with a 2h hx of rapid palpitations. He feels a little light
headed but is otherwise well. Exam: pulse=170bpm and regular,
BP=100/68mmHg. He has had 2 similar episodes in the past. What is the most
likely rhythm disturbance?
a. SVT
b. VF
c. VT
d. V-ectopics
e. A-fib
This is paroxysmal tachycardia (rate 150-250); only SVT is regular rhythm,
others are irregular rhythm
Clincher(s) 2h history of palpitation,similar episodes in the past
A Regular
B ventricular fibrillation is a "turbulent, disorganized electrical activity of the
heart in such a way that the recorded electrocardiographic deflections
continuously change in shape, magnitude and direction" . Ventricular
fibrillation is life-threatening
Ventricular fibrillation (v-fib for short) is the most serious cardiac rhythm
disturbance. The lower chambers quiver and the heart can't pump any blood,

causing cardiac arrest.




C


Ventricular tachycardia (V-tach or VT) is a type of tachycardia, or a rapid heart
beat that arises from improper electrical activity of the heart presenting as a
rapid heart rhythm, that starts in the bottom chambers of the heart, called the
ventricles

47
CVS-System Wise 1700-by Sush and Team. 2016
Susmita, Asad, Manu, Saima, Zohaib, Savia, Shanu, Mona, Manisha, Sitara, Samreena, Sami and Komal

D

Atrial
fibrillation (AF or A-fib) is an abnormal heart rhythm characterized by rapid
and irregular beating.[1] Often it starts as brief periods of abnormal beating
which become longer and possibly constant over time.[2] Most episodes have
no symptoms.[3] Occasionally there may be heart palpitations, fainting,
shortness of breath, or chest pain.[4] The disease increases the risk of heart
failure, dementia, and stroke.[3] It is a type of supraventricular tachycardia.

E Irregularly irregular pulse
KEY A
Additional
Information

48
CVS-System Wise 1700-by Sush and Team. 2016
Susmita, Asad, Manu, Saima, Zohaib, Savia, Shanu, Mona, Manisha, Sitara, Samreena, Sami and Komal

Reference Ohcm 803
Dr Khalid/Rabia


Q:1206 A 45yo man is admitted to ED with excruciating pain in the right leg. Exam:
limb is pale and dorsalis pedis and posterior tibial pulses are absent. Pulse=88
bpm, irregular and he has a pansystolic murmur at apex. What is the most
probable dx?
a. Thromboangiitis Obliterans
b. Sciatica
c. DVT
d. Atherosclerosis
e. Embolus

Clincher(s) Pain in leg,absent pulses,pansystolic murmur
A Pain at rest,Raynauds phenomenon,ulceration,gangrene
B Pulses present,along the path of the nereve
C Warm, skin looks red
D Clinically silent
E Ischaemia, occurs from abrupt interruption of blood flow due to emboli. A
cardiac origin is the source of emboli in most of the cases associated with
atrial fibrillation and mitral rugurgition
KEY E
Additional Acute limb ischaemia is most often due to either acute thrombotic occlusion
Information of a previously partially occluded, thrombosed arterial segment, or to
embolus from a distant site

49
CVS-System Wise 1700-by Sush and Team. 2016
Susmita, Asad, Manu, Saima, Zohaib, Savia, Shanu, Mona, Manisha, Sitara, Samreena, Sami and Komal

Reference OHCM 658
Dr Khalid/Rabia


Q:1210 A 51yo man has become increasingly fatigued for the past 10m. PE: no
abnormal findings. Labs: Hgb=9.2, Hct=27.9%, MCV=132fl, plt=242, WBC=7.59.
Which of the following morphologic findings is most likely to be present on
examination of his peripheral blood smear?
a. Hypersegmented neutrophils
b. Nucleated RBC
c. Blasts
d. Hypochromic, microcytic RBC
e. Schistocytes
a. Hypersegmented

Clincher(s) MCV 132fl(normal 76-96fl)
A normal neutrophils only contain three or four nuclear lobes (the "segments"),
hypersegmented neutrophils contain six or more lobes.[
B Normally mature RBC are anucleated.Nucleated RBC are found in fetuses and
in adults its found in in very early stages of cells life. if NRBCs are seen on an
adult's peripheral blood smear, it suggests that there is a very high demand for
the bone marrow to produce RBCs, and immature RBCs are being released into
circulation. Possible pathologic causes include anemia, myelofibrosis,
thalassemia, miliary tuberculosis, cancers involving bone marrow, and in
chronic hypoxemia.[2]
C Blast cells are abnormal, immature, nucleated precursor white cells pushed
out from the marrow into the circulation by processes such as myelofibrosis or
leukaemic infiltration.
D Usually found in iron deficiency anemia
E Schistocytes are often seen in patients with hemolytic anemia. They are
frequently a consequence of mechanical artificial heart valves and hemolytic
uremic syndrome, thrombotic thrombocytopenic purpura, among other causes
KEY A
Additional High MCV (macrocytic anaemia)
Information
1 B12 or folate defi ciency 5 Myelodysplastic syndromes

2 Alcohol excessor liver disease 6 Marrow infi ltration

3 Reticulocytosis 7 Hypothyroidism

4 Cytotoxics, eg hydroxycarbamide 8 Antifolate drugs (eg phenytoin


50
CVS-System Wise 1700-by Sush and Team. 2016
Susmita, Asad, Manu, Saima, Zohaib, Savia, Shanu, Mona, Manisha, Sitara, Samreena, Sami and Komal


Reference Ohcm wikipedia
Dr Khalid/Rabia

Q:1224 A 58yo man suddenly becomes shocked several days after suffering an acute
ant MI. His CXR shows a large globular-shaped heart and clear lung fields.
What is the single most likely
explanation for the abnormal inv?
a. Acute pericarditis
b. Cardiac tamponade
c. Atrial thrombus
d. Left ventricular aneurysm
e. Dressler syndrome

Clincher(s) Globular heart In Xray (cardiac temponade/ complication of MI)
A
B
C
D
E
KEY B

51
CVS-System Wise 1700-by Sush and Team. 2016
Susmita, Asad, Manu, Saima, Zohaib, Savia, Shanu, Mona, Manisha, Sitara, Samreena, Sami and Komal

Additional
Information


Reference ohcm
Dr Khalid/Rabia

Q:1229 A branch of the dominant coronary artery that supplies the inferior portion of
the septum. What is the single most appropriate option?
a. Septal branches
b. Obtuse marginal branches
c. Circumflex artery
d. Left main stem, post descending artery
e. Diagonal branch

Clincher(s)
A
B
C
D
E
KEY D
Additional Left coronary artery is larger in diameter than the right coronary artery and
Information arises from the posterior aortic sinus of ascending aorta. It passes posterior
to the pulmonary trunk, then divides into two branches; the anterior
interventricular / left anterior descending artery and the circumflex artery.

Reference . Within the groove, the anterior interventricular artery passes around the
inferior border of the heart. On the inferior surface of the heart it anastomoses
with the posterior interventricular branch of the right coronary artery.
The anterior interventricular artery supplies the:
left ventricle
right ventricle - small section of outflow tract
interventricular septu

The right coronary artery emerges from the anterior aortic sinus and descends
in the coronary sulcus on the anterior surface of the heart, between the right
atrium and right ventricle. At the inferior border of the heart, the artery
branches to give off the right marginal artery. Continuing into the posterior
part of the coronary sulcus, the right coronary artery branches to give off the

52
CVS-System Wise 1700-by Sush and Team. 2016
Susmita, Asad, Manu, Saima, Zohaib, Savia, Shanu, Mona, Manisha, Sitara, Samreena, Sami and Komal

posterior interventricular artery that runs in the posterior interventricular
groove. Overall, the branches of the right coronary artery supply the right
atrium and almost all of the right ventricle.
The arrangement of the coronary arteries varies considerably in every
individual. For example, in about 15% of people, the left coronary artery gives
rise to both (anterior and posterior) interventricular arteries. In a small
number of people (4%), one single coronary artery arises from ascending aorta
and supplies the entire heart.



Dr Khalid/Rabia


Q:1239 The artery that supplies the ant right ventricular wall. What is the single most
appropriate option?
a. Acute marginal branch
b. Left ant descending artery
c. Coronary sinus
d. Circumflex artery
e. Right coronary artery

Clincher(s)
A the right marginal artery of the heart is a branch of the right coronary artery. It
branches off at the inferior border of the heart and passes to the left towards
the apex. It passes horizontally, branching and tapering along its route. It
supplies the right ventricle.
B the LAD artery and its branches supply most of the interventricular septum;
the anterior, lateral, and apical wall of the left ventricle, most of the right and
left bundle branches, and the anterior papillary muscle of the bicuspid valve
C The coronary sinus is a collection of veins joined together to form a large
vessel that collects blood from the heart muscle (myocardium). It delivers
deoxygenated blood to the right atrium, as do the superior and inferior vena
cavae

53
CVS-System Wise 1700-by Sush and Team. 2016
Susmita, Asad, Manu, Saima, Zohaib, Savia, Shanu, Mona, Manisha, Sitara, Samreena, Sami and Komal

D The LCX supplies the posterolateral left ventricle and the anterolateral
papillary muscle.
It also supplies the sinoatrial nodal artery in 38% of people.
It supplies 15-25% of the left ventricle in right-dominant systems. If the
coronary anatomy is left-dominant, the LCX supplies 40-50% of the left
ventricle

E
KEY A
Additional
Information
Reference Various websites/ wiki/GP note
Dr Khalid/Rabia


Q:1246 After an MI, a man presents with pansystolic murmur which is radiating to the
axilla. What is the dx?
a. Tricuspid regurgitation
b. Mitral regurgitation
c. Aortic stenosis
d. Mitral stenosis

Clincher(s) Pansystolic murmur radiating to axilla
A pansystolic murmur, heard best at lower sternal edge in inspiration
B displaced,hyperdynamic apex; RV heave; soft S1; split S2; loud P2
(pulmonary hypertension)
pansystolic murmur at apex radiating to axilla
C Slow rising pulse with narrow pulse pressure (feel for diminished and delayed
carotid upstrokeparvus et tardus); heaving, non-displaced apex beat; LV
heave; aortic thrill; ejection systolic murmur (heard at the base, left
sternal edge and the aortic area, radiates to the carotids).
D loud S1; opening
snap (pliable valve); rumbling mid-diastolic murmur (heard best in expiration,
with patient on left side
E
KEY B
Additional
Information
Reference OHCM 138,140
Dr Khalid/Rabia D-ARMS
S-MRAS

D (dias)-AR (aro reg=Pul) MS (=TS)
S (sys)-MR (mit reg=TR) AS (=PS)

MR (radiate to axilla)-Tricusp reg (pansyn murmur)

54
CVS-System Wise 1700-by Sush and Team. 2016
Susmita, Asad, Manu, Saima, Zohaib, Savia, Shanu, Mona, Manisha, Sitara, Samreena, Sami and Komal


AS (radiate to carotid); wit PS no radiation

PASS and PAID

PAS (pul and aortic sys)-S
PAI (P and A insuff/reg)-D

And mitral and tricus stenois and regu- will be opposite





Q: 247 A 40 yo manic depressive is noted to have high serum levels of lithium and
profound
hypokalemia. His GP had started him on anti-HTNs. Choose the single most
likely cause?
a. Verapamil
b. Amiodarone
c. Ranitidine
d. Lithium
e. Thiazide


Clincher(s)
A
B Anti arythmic
C
D
E Known side effect of thiaziode diuretics is hypokalemia
KEY The key is E. Thiazide. [Thiazide was prescribed for Hpt and when lithium was
prescribed its level increased due to thiazide and thiazide also caused
hypokalemia resulting the given picture].
Thiazide diuretics can increase the risk of lithium (Eskalith, Lithobid) toxicity by
reducing the kidney's ability to eliminate lithium in the urine.

Additional Thiazide Diuretics Side Effects:
Information A possible increase in blood sugar level.
A possible increase in the level of uric acid. So contra indicated in
gout.
It can cause a low blood level of potassium, sodium, and
magnesium, and a high level of calcium. These effects may cause
weakness, confusion, and rarely, abnormal heart rhythms to
develop.
Other problems, such as:

55
CVS-System Wise 1700-by Sush and Team. 2016
Susmita, Asad, Manu, Saima, Zohaib, Savia, Shanu, Mona, Manisha, Sitara, Samreena, Sami and Komal

o Upset stomach.
o Dizziness on standing (due to too low blood pressure).
o Erection problems (impotence) - often reversible on
stopping treatment.
o Skin sensitivity to sunlight.


Reference Medicine.net
Dr Khalid/Rabia Thiazide Diuretics Side Effects:
A possible increase in blood sugar level.
A possible increase in the level of uric acid. So contra indicated in
gout.
It can cause a low blood level of potassium, sodium, and
magnesium, and a high level of calcium. These effects may cause
weakness, confusion, and rarely, abnormal heart rhythms to
develop.
Other problems, such as:
o Upset stomach.
o Dizziness on standing (due to too low blood pressure).
o Erection problems (impotence) - often reversible on
stopping treatment.
o Skin sensitivity to sunlight.




Q: 298 An 18yo man complains of fatigue and dyspnea, he has left parasternal heave
and systolic thrill
with a harsh pan-systolic murmur at left parasternal edge. What is the most
probable dx?
a. TOF
b. ASD
c. VSD
d. PDA
e. TGA


Clincher(s)
A TOF may be left undiagnosed this late but patients present with severe
cyanosis and other typical features of TOF.

B ASD has a soft systolic ejection murmur in the pulmonic area and diastolic
rumble at left sternal border.

56
CVS-System Wise 1700-by Sush and Team. 2016
Susmita, Asad, Manu, Saima, Zohaib, Savia, Shanu, Mona, Manisha, Sitara, Samreena, Sami and Komal

C
D Continuos machine like murmur
E Transposition of great arteries presents in the infants and not that late in life.

KEY Key = C (VSD)
Points in favour = Age , Left parasternal heave, pan systolic murmur at left
parasternal edge.

Additional
Information
Reference
Dr Khalid/Rabia

VSD :-

Epidemiology[3]
VSDs are the most common congenital heart defect in children,
occurring in 50% of all children with congenital heart disease and in
20% as an isolated lesion.
The incidence of VSDs has increased significantly with advances in
imaging and screening of infants and ranges from 1.56 to 53.2 per
1,000 live births. The ease with which small muscular VSDs can now be
detected has contributed to this increase in incidence.
In the adult population VSDs are the most common congenital heart
defect, excluding bicuspid aortic valve.

Presentation
How haemodynamically significant a VSD is depends on its size, pressure in the
individual ventricles and pulmonary vascular resistance.[3] The presence of a
VSD may not be obvious at birth because of nearly equal pressures in both the
ventricles with little or no shunting of blood. As the pulmonary vascular
resistance drops, the pressure difference between the two ventricles increases
and the shunt becomes significant allowing the defect to become clinically
apparent. An exception to this rule is Down's syndrome where the pulmonary
vascular resistance may not fall and the VSD may not become clinically
apparent, first presenting with pulmonary hypertension. All babies with
Down's syndrome should therefore be screened for congenital heart disease
no later than 6 weeks of age.[8]
The clinical presentation varies with the severity of the lesion:
With a small VSD, the infant or child is asymptomatic with normal
feeding and weight gain and the lesion may be detected when a
murmur is heard at a routine examination.
With a moderate-to-large VSD, although the babies are well at birth,
symptoms generally appear by 5 to 6 weeks of age. The main symptom
is exercise intolerance and since the only exercise babies do is feeding,

57
CVS-System Wise 1700-by Sush and Team. 2016
Susmita, Asad, Manu, Saima, Zohaib, Savia, Shanu, Mona, Manisha, Sitara, Samreena, Sami and Komal

the first impact is on feeding. Feeding tends to slow down and is often
associated with tachypnoea and increased respiratory effort. Babies are
able to feed less, and weight gain and growth are soon affected. Poor
weight gain is a good indicator of heart failure in a baby. Recurrent
respiratory infections may also occur.
With very large VSDs the features are similar but more severe. If
appropriate management is not carried out promptly in infants with
large VSDs excessive pulmonary blood flow may lead to increase in
pulmonary vascular resistance and pulmonary hypertension. These
babies may develop a right to left shunt with cyanosis or Eisenmenger's
syndrome.

Physical signs
Again, these depend on the severity of the lesion with, one exception, the
loudness of the murmur. Murmurs are caused by turbulence of blood flow.
There may be more turbulence with a small hole than with a large defect. The
loudness of the murmur gives no indication of the size of the lesion. Even the
adage 'the louder the sound, the smaller the lesion' is untrue.
With a small VSD the infant is well developed and pink. The precordial
impulse may be greater than usual but is usually normal. If it can be
heard, the physiological splitting of the second sound is normal but
there is a harsh systolic murmur that is best heard at the left sternal
edge, which may obliterate the second sound. The murmur tends to be
throughout systole but, if the defect is in the muscular portion, it may
be shorter as the hole is closed as the muscle contracts.
With a moderate or large VSD there is enhanced apical pulsation as
well as a parasternal heave. A grade 2 to 5/6 systolic murmur is audible
at the lower left sternal border. It may be pansystolic or early systolic. A
prominent third sound with a short early mid-diastolic rumble is
audible at the apex with a moderate-to-large shunt (because of
increased flow through the mitral valve during diastole). S2 is loud and
single in patients with pulmonary hypertension.
Large defects with no shunts or those with Eisenmenger physiology and
right-to-left shunt may have no murmur.

Investigations = ECG, CXR, ECHOCARDIOGRAPHY and Cardiac catheterization.

Management[10]

Medical management
Management in the infant and child depends on symptoms, with small
asymptomatic defects needing no medical management, and unlikely
to need any intervention.
First-line treatment for moderate or large defects affecting feeding and
growth is with diuretics for heart failure and high-energy feeds to

58
CVS-System Wise 1700-by Sush and Team. 2016
Susmita, Asad, Manu, Saima, Zohaib, Savia, Shanu, Mona, Manisha, Sitara, Samreena, Sami and Komal

improve calorie intake.
Angiotensin-converting enzyme inhibitors are used to reduce afterload
which promotes direct systemic flow from the left ventricle, thus
reducing the shunt. Digoxin can also be given for its inotropic effect.
Any patient needing significant medical management should be
referred for surgical assessment.

Surgical management
Surgical repair is required if there is uncontrolled heart failure,
including poor growth. Even very small babies may be considered for
surgery.
Infundibular defects may be considered for closure even if they are
asymptomatic because of their location.
Development of aortic valve prolapse and aortic regurgitation in
perimembranous VSDs may be an indication for surgical closure.
Most defects are closed nowdays by directly placing a patch from the
right ventricular side, usually with the surgeon working through the
tricuspid valve.
Patients with large muscular VSDs which are difficult to see or those
with multiple holes (Swiss cheese septum) presenting as neonates or
infants need initial palliation in the form of pulmonary artery banding
followed many months later by corrective surgery and removal of the
pulmonary artery band.

Catheter closure
Advances in catheter techniques and devices mean that many muscular
and perimembranous VSDs can now be closed percutaneously. This is
in the setting of normal atrioventricular and ventriculoarterial
connections and absence of any atrioventricular or arterial valve
override.
Transcatheter techniques are useful because they avoid
cardiopulmonary bypass. There are, however, recognised complications
for device closure of perimembranous VSDs, including complete heart
block needing permanent pacemaker.[11]
The National Institute for Health and Care Excellence (NICE) has
provided detailed guidance on indications, efficacy and complications
of the procedure.[12]
It is safer to close muscular VSDs using a device but muscular VSDs
which are haemodynamically significant are likely to be seen in only
young infants, making catheterisation difficult and challenging. Hybrid
procedures increasingly being used involve insertion of the device in
the operation theatre after surgical exposure of the defect.[13]



59
CVS-System Wise 1700-by Sush and Team. 2016
Susmita, Asad, Manu, Saima, Zohaib, Savia, Shanu, Mona, Manisha, Sitara, Samreena, Sami and Komal


Q: 306 An MI pt who is already on aspirin no longer smokes and his cholesterol, ECG,
echo and BP are
normal. Choose the best option for him:
a. Give statin
b. Give statin+warfarin
c. Low cholesterol diet
d. Statin+ACEi

Clincher(s)
A
B
C
D
E
KEY D. Statin + ACEi.

Additional ACEi should be given to all MI patients unless contraindicatedeg. Renal
Information failure, hyperkalemia, hyponatremia
Reference Ohcm 114
Dr Khalid/Rabia [Offer all people who have had an acute MI treatment with the following
drugs:

ACE (angiotensin-converting enzyme) inhibitor

dual antiplatelet therapy (aspirin plus a second antiplatelet agent)

beta-blocker

statin. [2007, amended 2013] [NICE guideline].





Q: 322 A 50yo smoker and heavy drinker presents with complaints of racing heart. A
24h ECG comes
out normal. What is your next step in management?
a. ECHO
b. Reassure
c. Stress test


Clincher(s)
A
B

60
CVS-System Wise 1700-by Sush and Team. 2016
Susmita, Asad, Manu, Saima, Zohaib, Savia, Shanu, Mona, Manisha, Sitara, Samreena, Sami and Komal

C
D
E
KEY The key is B. Reassure (SVT)

Additional
Information
Reference
Dr Khalid/Rabia


Q: 325 In 85% of the population this artery is dominant. What is the single most
appropriate option?
a. Left ant descending artery
b. Coronary sinus
c. Circumflex artery
d. Left main stem, post descending artery
e. Right coronary artery


Clincher(s)
A
B
C
D
E
KEY The key is E. Right coronary artery.

Additional
Information
Reference
Dr Khalid/Rabia If the posterior discending artery is supplied by the circumflex artery then it is
left dominant and if posterior descending artery is supplied by the right
coronary artery then it is right dominant. As in 85% of population posterior
descending artery is supplied by right coronary artery it is called the dominant
that is right coronary artery is dominant.


Q: 333 A 60yo pt who has had a MI a week back presents with dyspnea and
pericardial rub. ECG shows
ST elevation. CXR: loss of margin at costo-vertebral angle. What is the single
most likely cause?
a. Cardiac tamponade
b. Mitral regurge
c. Dresslers syndrome
d. Atrial fib

61
CVS-System Wise 1700-by Sush and Team. 2016
Susmita, Asad, Manu, Saima, Zohaib, Savia, Shanu, Mona, Manisha, Sitara, Samreena, Sami and Komal

e. Emboli


Clincher(s)
A
B
C Recurrent pericarditis, pleural effusions, fever and chest pain, anemia and ESR
raised 1-3 weeks post MI. treatment is NSAIDS, steroids if severe
D
E
KEY The key is C. Dresslers syndrome.

Additional
Information
Reference Ohcm 116
Dr Khalid/Rabia There is pericardial rub there is pericarditis and in pericarditis there is
widespread ST elevation. So the condition is Dresslers syndrome.



Q: 344 A 68yo man gets repeated attacks of LOC and TIA. What is the most likely
cause for this?
a. Atrial fib
b. Mitral stenosis
c. Aortic stenosis
d. HOCM
e. Carotid artery stenosis



Clincher(s)
A
B
C
D
E
KEY The key is E. Carotid artery stenosis.
Additional
The carotid artery is the large vertical artery in red. The blood supply to the
Information
carotid artery starts at the arch of the aorta (bottom). The carotid artery
divides into the internal carotid artery and the external carotid artery. The
internal carotid artery supplies the brain. Plaque often builds up at that
division, and causes a narrowing (stenosis). Pieces of plaque can break off and
block the small arteries above in the brain, which causes a stroke. Plaque can
also build up at the origin of the carotid artery at the aorta.

62
CVS-System Wise 1700-by Sush and Team. 2016
Susmita, Asad, Manu, Saima, Zohaib, Savia, Shanu, Mona, Manisha, Sitara, Samreena, Sami and Komal

Signs and symptoms[edit]
The carotid artery is the large artery whose pulse can be felt on both sides of
the neck under the jaw. On the right side it starts from the brachiocephalic
trunk (a branch of the aorta) as the common carotid artery, and on the left
side the common carotid artery comes directly off the aortic arch. At the
throat it forks into the internal and external carotid arteries. The internal
carotid artery supplies the brain, and the external carotid artery supplies the
face. This fork is a common site for atherosclerosis, an inflammatory buildup of
atheromatous plaque that can narrow the lumen of the common or internal
carotid arteries.
The plaque can be stable and asymptomatic, or it can be a source of
embolization. Emboli break off from the plaque and travel through the
circulation to blood vessels in the brain. As the vessel gets smaller, they can
lodge in the vessel wall and restrict blood flow to parts of the brain which that
vessel supplies. This ischemia can either be temporary, yielding a transient
ischemic attack, or permanent resulting in a thromboembolic stroke.
Clinically, risk of stroke from carotid stenosis is evaluated by the presence or
absence of symptoms and the degree of stenosis on imaging.
Transient ischemic attacks (TIAs) are a warning sign, and are often followed by
severe permanent strokes, particularly within the first two days. TIAs by
definition last less than 24 hours and frequently take the form of a weakness
or loss of sensation of a limb or the trunk on one side of the body, or the loss
of sight (amaurosis fugax) in one eye. Less common symptoms are artery
sounds (bruits), or ringing in the ears (tinnitus).

Diagnosis[edit]

63
CVS-System Wise 1700-by Sush and Team. 2016
Susmita, Asad, Manu, Saima, Zohaib, Savia, Shanu, Mona, Manisha, Sitara, Samreena, Sami and Komal


Section of carotid artery with plaque. Blood flows from the common carotid
artery(bottom), and divides into the internal carotid artery (left) andexternal
carotid artery (right). The atherosclerotic plaque is the dark mass on the left.
Carotid stenosis is usually diagnosed by color flow duplex ultrasound scan of
the carotid arteries in the neck. This involves no radiation, no needles and no
contrast agents that may cause allergic reactions. This test has moderate
sensitivity and specificity, and yields many false-positive results.
Typically duplex ultrasound scan is the only investigation required for decision
making in carotid stenosis as it is widely available and rapidly performed.
However, further imaging can be required if the stenosis is not near the
bifurcation of the carotid artery.
One of several different imaging modalities, such as angiogram, computed
tomography angiogram (CTA)[1][2][3] or magnetic resonance imagingangiogram
(MRA) may be useful. Each imaging modality has its advantages and
disadvantages - Magnetic resonance angiography and CT angiography with
contrast is contraindicated in patients with renal insufficiency, catheter
angioigraphy has a 0.5% to 1.0% risk of stroke, MI, arterial injury or
retoperitoneal bleeding. The investigation chosen will depend on the clinical

64
CVS-System Wise 1700-by Sush and Team. 2016
Susmita, Asad, Manu, Saima, Zohaib, Savia, Shanu, Mona, Manisha, Sitara, Samreena, Sami and Komal

question and the imaging expertise, experience and equipment available. [4]

Screening[edit]
The U.S. Preventive Services Task Force (USPSTF) recommends
against screening for carotid artery stenosis in those without symptoms.[5]

Management[edit]
Options include:

Medications alone (an antiplatelet drug (or drugs) and control of risk
factors for atherosclerosis).
Medical management plus carotid endarterectomy or carotid stenting,
which is preferred in patients at high surgical risk and in younger patients.
Control of smoking, hypertension, and hyperlipidemia.
The goal of treatment is to reduce the risk of stroke (cerebrovascular accident).
Intervention (carotid endarterectomy or carotid stenting) can cause stroke;
however, where the risk of stroke from medical management alone is high,
intervention may be beneficial. In selected trial participants with
asymptomatic severe carotid artery stenosis, carotid endarterectomy reduces
the risk of stroke in the next 5 years by 50%, though this represents a
reduction in absolute incidence of all strokes or perioperative death of
approximately 6%. In most centres, carotid endarterectomy is associated with
a 30-day stroke or mortality rate of < 3%; some areas have higher rates.[6]
Clinical guidelines (such as those of National Institute for Clinical
Excellence (NICE)[citation needed]) recommend that all patients with carotid
stenosis be given medication, usually anti-hypertensive drugs, anti-clotting
drugs, anti-platelet drugs (such as aspirin), and especially statins (which were
originally prescribed for their cholesterol-lowering effects but were also found
to reduce inflammation and stabilize plaque).
NICE and other guidelines also recommend that patients
with symptomatic carotid stenosis be given carotid endarterectomy urgently,
since the greatest risk of stroke is within days. Carotid endarterectomy reduces
the risk of stroke or death from carotid emboli by about half.
For people with stenosis but no symptoms, the interventional
recommendations are less clear. Such patients have a historical risk of stroke
of about 1-2% per year. Carotid endarterectomy has a surgical risk of stroke or
death of about 2-4% in most institutions. In the large Asymptomatic Carotid
Surgery Trial (ACST) endarterectomy reduced major stroke and death by about
half, even after surgical death and stroke was taken into account.[7] According

65
CVS-System Wise 1700-by Sush and Team. 2016
Susmita, Asad, Manu, Saima, Zohaib, Savia, Shanu, Mona, Manisha, Sitara, Samreena, Sami and Komal

to the Cochrane Collaboration the absolute benefit of surgery is small.[citation
needed]
For intervention using stents, there is insufficient evidence to support
stenting rather than open surgery, and several trials, including the ACST-2, are
comparing these 2 procedures.
Surgery and stenting[edit]
The largest clinical trial performed, CREST, randomized patients at risk for a
stroke from carotid artery blockage to either open surgery (carotid
endarterectomy) or carotid stent placement with embolic protection. This trial
followed patients for 4 years and found no overall difference in the primary
end point of both treatment arms (myocardial infarctions, any perioperative
strokes or ipsilateral strokes within 4 years, or death during procedure).
Patients assigned to the surgical arm experienced more perioperative
myocardial infarctions compared to the stenting group; however, the
difference was not statistically significant (6.8% vs or 7.2% HR for stenting is
1.1 CI 0.81-1.51 P value 0.51) whereas patients assigned to the carotid stent
arm experienced more periprocedural strokes compared to endarteretomy
(6.4% vs 4.7% HR for stenting 1.5 P-0.03). There was no mortality difference
and no difference for major (disabling) strokes between surgery and stenting.
It was noted that there did seem to exist an age cutoff where below 75 years
old endarterectomy provided more positive outcomes and over 75 stenting
offered a better risk profile. However, it should be noted that the CREST trial
was not designed for subgroup analysis and thus not powered enough to draw
any statistically significant conclusions.[8] A later study published in 2013
evaluated how these perioperative complications affect long-term survival.
This study showed that experiencing a stroke within the first year conferred a
two-fold lower survival rate (Hazard Ratio(HR) 6.6 [CI 3.7-12]) than those who
experienced a perioperative myocardial infarction at two years post
intervention (HR 3.6 [CI 2-6.8]). This difference in mortality, however,
converges and becomes negligible at 5 years (HR 2.7 [CI 1.7-4.3] vs HR 2.8 [CI
1.8-4.3]).[9] A 2010 study found benefits (reduced strokes) from carotid
endarterectomy in those without symptoms who are under 75.[10]

Reference wikipedia
Dr Khalid/Rabia


Q: 347 A 57yo woman who is suffering from HTN, presented to the hospital with
complaints of
recurrent falls when trying to get out of bed or getting up from sitting. She is
on some anti-HTN
therapy with no other med prbs. What is the cause of her fall?

66
CVS-System Wise 1700-by Sush and Team. 2016
Susmita, Asad, Manu, Saima, Zohaib, Savia, Shanu, Mona, Manisha, Sitara, Samreena, Sami and Komal

a. CCB
b. Vestibrobasiliar insufficiency
c. Thiazide
d. Hypoglycemia
e. Infection


Clincher(s)
A Peripheral edema, flushing,
B
C
D
E
KEY The key is C. Thiazide. It causes postural hypotension by volume depletion.
Additional Side effects of thiazide diuretics are dose related and include:
Information
dizziness and lightheadedness,
blurred vision,
loss of appetite,
itching,
stomach upset,
headache, and
weakness.


Reference Medicine.net
Dr Khalid/Rabia


Q: 350 A 28yo shipyard worker was admitted for pain in calf while at work which has
been increasing
over the last 3m. There is no hx of HTN or DM but he is a smoker. Exam: loss of
posterior tibial
and dorsalis pedis pulsation along with a non-healing ulcer at the base of the
right 1st MTP joint.
What is the most probably dx?
a. Thromboangitis obliterans
b. Sciatica
c. DVT
d. Bakers cyst
e. Embolus

67
CVS-System Wise 1700-by Sush and Team. 2016
Susmita, Asad, Manu, Saima, Zohaib, Savia, Shanu, Mona, Manisha, Sitara, Samreena, Sami and Komal

Clincher(s)
A
B
C
D
E
KEY The key is A. Thromboangitis obliterans.
Additional . i) young age ii) smoker iii) pain in cuff iv) loss of posterior tibial and dorsalis
Information pedis pulsation v) non-healing ulcer at the base of the right 1st MTP joint all are
suggestive of Buergers disease.

Reference
Dr Khalid/Rabia


Q: 766 A 52yo man with hx of ant MI 3 wks ago developed sudden onset of dyspnea.
Exam: BP=100/60mmHg, pulse=100bpm, SaO2=88%, chest=bilateral crackles.
Which inv would you do to find the underlying cause?
a. CXR
b. Echo
c. D-dimer
d. Radionucleotide lung scan
e. Troponin

Clincher(s) Hx of MI, bilateral crackles, BP=100/60mmHg, pulse=100bpm, SaO2=88%
A Will show the features of pulmonary oedema
B
C Conditions in which there may be a raised D-dimer include:
pulmonary embolism (PE)
deep vein thrombosis (DVT)
disseminated intravascular coagulation (DIC)
postoperatively
D
E It is for MI
KEY: B The key is A. CXR. It is wrong key. Correct key should be B. Echo. [The
question does not want to know the features of pulmonary oedema rather it is
clearly mentioned which investigation would you do to find out the
UNDERLYING CAUSE. As in this patient sudden papillary muscle rupture or
VSD may be the likely cause (ventricular aneurism may take a more slower
course) so echo seems to be more logical approach].
Additional Cardiac temponade- becks, and diastolic falls to 0
Information
Could be MR: to see through echo
Reference Dr. Khalid, GPnotebook
Dr Khalid/Rabia

68
CVS-System Wise 1700-by Sush and Team. 2016
Susmita, Asad, Manu, Saima, Zohaib, Savia, Shanu, Mona, Manisha, Sitara, Samreena, Sami and Komal



Q: 775 A middle aged male is feeling unwell after a recent MI. The recent ECG shows
prolonged QRS complex and tented T wave. Na+=136mmol/l, K+=6.2mmol/l,
urea=5mmol/l. What is the most appropriate management?
a. Calcium gluconate
b. Calcium resonium
c. Calcium with vit D supplement
d. Vit D
e. Calcium

Clincher(s)
A
B
C
D
E
KEY: A Calcium gluconate.
ECG change of prolonged QRS complex and tented T wave are suggestive of
hyperkalaemia which is further supported by K+ level of 6.2 mmol/l. So to
protect the heart from arrhythmia calcium gluconate should be given
Additional
Information


Reference
Dr Khalid/Rabia OHCM pg:849 hyperkalaemia emergency; ohcm pg: 688 - hyperkalemia

69
CVS-System Wise 1700-by Sush and Team. 2016
Susmita, Asad, Manu, Saima, Zohaib, Savia, Shanu, Mona, Manisha, Sitara, Samreena, Sami and Komal

Q: 787 A 74yo lady called an ambulance for an acute chest pain. She has a hx of DM
and HTN, and is a heavy smoker. Paramedics mentioned that she was
overweight and recently immobile because of a hip pain. She collapsed and
died in the ambulance. What is the most likely cause of death?
a. Pulmonary embolism
b. MI
c. Stroke
d. Cardiac arrhythmia

Clincher(s) recently immobile because of a hip pain, DM & HTN,
A
B Sudden onset of chest pain which is central, crushing, retrosternal, radiating t
left arm or neck. Lasting >20min. associated with nausea, vomiting, sweating
C


D
KEY: A Pulmonary embolism.
Immobilisation for hip pain is the clincher of pulmonary embolism here

70
CVS-System Wise 1700-by Sush and Team. 2016
Susmita, Asad, Manu, Saima, Zohaib, Savia, Shanu, Mona, Manisha, Sitara, Samreena, Sami and Komal

Additional
Information


Reference Samsons notes, GPnotebook,
Dr Khalid/Rabia



Q: 805 A 75yo man on digoxin develops weakness in the right upper and lower limbs

71
CVS-System Wise 1700-by Sush and Team. 2016
Susmita, Asad, Manu, Saima, Zohaib, Savia, Shanu, Mona, Manisha, Sitara, Samreena, Sami and Komal

which resolves within a few hours. What is the most definitive inv for this
condition?
a. Carotid Doppler
b. Angiography
c. CT head
d. Digoxin level
NEED to be checked: to post on FB
Carotid artery stenosis
Clincher(s)
A
B
C
D
E
KEY: B? Dr. khalid >> Angiography. [Most definitive angiography; initial or most
appropriate dopplar].

72
CVS-System Wise 1700-by Sush and Team. 2016
Susmita, Asad, Manu, Saima, Zohaib, Savia, Shanu, Mona, Manisha, Sitara, Samreena, Sami and Komal


Additional Carotid is essential and angiography
Information
Reference OHCM pg:658
Dr Khalid/Rabia Dr. Rabia >> KEY is B its a wrong key. (probably correct)

Correct Key is A.
This is a case of TIA since focal neurological symptoms vanished within few
hours of onset.
Also supported by use of digoxin which means patient already had some
arrhythmias or heart failure or there can be digoxin induced heart failure due
to which the chance of thromboembolism were increased and TIA occurred.
One of the major complication of these emboli is Carotid artery stenosis which
leads to decreased perfusion of the cerebral cortex and presentation with
stroke or TIA.
Carotid Doppler is the most definitive investigation for this condition.


Q: 817 A 48yo man has continuous ant chest pain which is worse on inspiration and
has temp of 37.5C since 4wks after a MI. His ESR=45mm/h. What is the single
most likely explanation for the abnormal inv?
a. Acute pericarditis

73
CVS-System Wise 1700-by Sush and Team. 2016
Susmita, Asad, Manu, Saima, Zohaib, Savia, Shanu, Mona, Manisha, Sitara, Samreena, Sami and Komal

b. Cardiac tamponade
c. Atrial thrombus
d. Left ventricular aneurysm
e. Dressler syndrome
Chest pain worse on ispiration but hx of pericarditis after 2 days. Here it is 4
weeks

Clincher(s) continuous ant chest pain worse on inspiration, temp of 37.5C, 4wks after a
MI, ESR=45mm/h (ESR Normal range: <20mm/h.)
A


C
D


E E
KEY: E Dressler syndrome
dressler occurs after > 2 weeks @ least. before that it can be pericarditis.
Dressler & LV Aneurysm occurs after 2 weeks. wherever there is pleurisy after
2 weeks of MI, most probably Dressler. Autoimmune kind of disease. only
steroids will help.

74
CVS-System Wise 1700-by Sush and Team. 2016
Susmita, Asad, Manu, Saima, Zohaib, Savia, Shanu, Mona, Manisha, Sitara, Samreena, Sami and Komal


Additional
Information
Reference OHCM pg: 712, samsons notes
Dr Rabia Answer: E
Dresslers syndrome was first described in 1956. It is characterised by pleuritic
chest pain, low-grade fever and pericarditis (autopsy shows localised fibrinous
pericarditis), which may be accompanied by pericardial effusion. It tends to
follow a benign clinical course. It is thought to be immune-mediated (antiheart
antibodies may be present). The reported incidence has been declining in
recent years.[1]
Risk factors
If a person has had a previous episode, it is more likely to recur. It seems more
likely to occur after a large infarct.

Presentation
It usually presents two to five weeks after the initial episode, with pain and
fever that may suggest further infarction.
The pain is the main symptom, often in the left shoulder, often pleuritic, and
worse on lying down.
There may be malaise, fever and dyspnoea.
Rarely, it may cause cardiac tamponade or acute pneumonitis.
A pericardial friction rub may be heard. The typical sound of pericarditis is
described as like the sound of boots walking over fresh snow.
Differential diagnosis
The pain may initially suggest a further episode of angina or myocardial
infarction. Pleuritic chest pain may also suggest pneumonia or pulmonary
embolism.

Investigations
FBC will show leukocytosis, sometimes with eosinophilia and an elevated ESR.
Serology may show heart autoantibodies.
ECG may show ST elevation in most leads without reciprocal ST depression,
typical of pericardial effusion.
Echocardiography shows pericardial effusion.
MRI scan may show an effusion and, more recently, has been shown to reveal
pericardial involvement.[8]
CXR shows pleural effusions in 83%, parenchymal opacities in 74%, and an
enlarged cardiac silhouette in 49%.

Management
Aspirin may be given in large doses.

75
CVS-System Wise 1700-by Sush and Team. 2016
Susmita, Asad, Manu, Saima, Zohaib, Savia, Shanu, Mona, Manisha, Sitara, Samreena, Sami and Komal

Other non-steroidal anti-inflammatory drugs (NSAIDs) or corticosteroids may
be used, especially if there are severe and recurrent symptoms.
Steroids are particularly valuable where severe symptoms have required
pericardiocentesis, and when infection has been excluded.
In resistant or recurrent cases, colchicine may be useful.
If there is significant pericardial effusion then pericardiocentesis, involving
aspiration of the fluid, may be required to relieve the constriction on the heart.


Q: 836 A 70yo woman with longstanding anxiety is seen in the OPD. She complains of
her heart skipping a beat quite often. This particularly occurs when she is
trying to get to sleep. The palpitations are never sustained. What is the most
likely rhythm disturbance?
a. SVT
b. VF
c. VT
d. V-ectopics
e. A-fib

Clincher(s) longstanding anxiety, heart skipping a beat quite often, occurs when she is
trying to get to sleep, palpitations are never sustained
A
B
C
D
E
KEY: D V-ectopics.
From the given options the most likely answer is V-ectopics. If it occurs in a
normal heart though symptomatic it is benign in nature but if it is secondary to
heart disease like MI it may precipitate to more life threatening arrhythmia like
ventricular fibrillation


Additional
Information
Reference Dr. Khalid & samsons notes
Dr Khalid/Rabia


Q: 876 A 60yo is on tx for IHD, HTN and hyperlipidemia. During the night he complains
of wheeze and SOB. Which of the following meds is responsible for that?
a. Amlodipine
b. Atenolol

76
CVS-System Wise 1700-by Sush and Team. 2016
Susmita, Asad, Manu, Saima, Zohaib, Savia, Shanu, Mona, Manisha, Sitara, Samreena, Sami and Komal

c. Ramipril
d. Simvastatin
e. Bendroflumethiazide

Clincher(s) Wheeze and SOB
A CCB - major effect is to cause the relaxation of the vascular smooth muscle in
small arterioles resulting in reduced peripheral vascular resistance. They also
promote the dilatation of coronary arteries


B Beta (-)
C ACEi


D HMG CoA reductase inhibitors (statins)
E Thiazide diuretic do not cause SOB
KEY: B Atenolol.
Asthma can be precipitated by beta blockers
wheeze and SOB which is a very common side effect of Atenolol, a beta
blocker which are contraindicated in asthma as it causes severe
bronchospasm leading to wheeze and SOB

SOB is less common with calcium channel blockers, ACE inhibitors and
anti hyperlipidemics.

77
CVS-System Wise 1700-by Sush and Team. 2016
Susmita, Asad, Manu, Saima, Zohaib, Savia, Shanu, Mona, Manisha, Sitara, Samreena, Sami and Komal

Additional
Information


Reference Dr. khalid and Rabia
Dr Rabia Key : B

BETA BLOCKERS

Beta-blockers are medicines that are used to treat a variety of conditions.
Their full correct name is beta-adrenoceptor blocking medicines. There are
several types of beta-blocker. They include acebutolol, atenolol, bisoprolol,
carvedilol, celiprolol, labetalol, metoprolol, nadolol, nebivolol, oxprenolol,
pindolol, sotalol, propranolol and timolol.

Beta-blockers work by blocking the transmission of certain nerve impulses. The
ends of some nerves release a chemical (neurotransmitter) called
noradrenaline when the nerve is stimulated. This chemical then stimulates
beta-adrenergic receptors. These receptors are tiny structures which occur on
cells in various parts of the body including the heart, brain, and blood vessels.
When these receptors are stimulated, they cause various effects. For example,
nerve impulses to the heart can stimulate beta-adrenergic receptors on heart
cells. This causes an increase in the force and rate of the heartbeat.

The beta-adrenergic receptors are also stimulated by adrenaline (epinephrine),
a hormone which circulates in the bloodstream. Adrenaline is made in the
adrenal gland.

The beta-blocker medicine 'sits' on beta-adrenergic receptors and stops
(blocks) the receptor from being stimulated. So, for example, if beta-
adrenergic receptors in the heart are blocked, the force and rate of the
heartbeat are reduced.

Indications :
prevent angina pains
reduce risk of further MI
control certain arrythmias

78
CVS-System Wise 1700-by Sush and Team. 2016
Susmita, Asad, Manu, Saima, Zohaib, Savia, Shanu, Mona, Manisha, Sitara, Samreena, Sami and Komal

treat heart failure
lower bkood pressure
glucoma
anxiety
overactive thyroid gland
migraine

Common side effects include :
Blurred vision
cold hands or feet
confusion
difficult or labored breathing
dizziness, faintness, or lightheadedness
shortness of breath
sweating
tightness in chest
unusual tiredness or weakness
wheezing

Contraindications :
asthma
uncontrolled heart failure
bradycardia
hypotension
certain problems with rythm of heart
(source: patient.co.uk and mayo clinic)


Q: 894 A 50yo man presented with increased breathlessness at rest (think of another
diruretic). He is currently on furosemide, digoxin and isosorbide mononitrate.
What drug is going to help him?
a. Ramipril
b. Bendroflumethiazide
c. Atenolol
d. Amlodipine
e. Diltiazem

Clincher(s) increased breathlessness at rest, on furosemide already
A Ramipril - ACE inhibitor to treat high blood pressure decreasing the risk of
heart attack and stroke
B
C Atenolol - beta blocker to treat angina, hypertension and prevent heart attack
D & E Amlodipine & Diltiazem - calcium channel blocker used to treat hypertension,
angina and other conditions caused by coronary artery disease
KEY: B Dr. Khalid>> Bendroflumethiazide.
As patient is still in heart failure and pulmonary edema with the usage of loop

79
CVS-System Wise 1700-by Sush and Team. 2016
Susmita, Asad, Manu, Saima, Zohaib, Savia, Shanu, Mona, Manisha, Sitara, Samreena, Sami and Komal

diuretic...a thiazide diuretic could be added with loop when response is
inadequate ( Refernce;patient.info) this will reduce the volume overload and
improve breathing
Additional
Information
Reference
Dr Khalid/Rabia



Q: 923 A 56yo man who is hypertensive recently underwent a change in meds. 2days
later he developed wheezing. Which drug can cause this?
a. Atenolol
b. Ramipril
c. Bendroflumethiazide
d. Verapamil
e. Furosemide
CVs 108-109 SEffects
Clincher(s) wheezing
A
B
C
D
E
KEY: A Atenolol
Beta blockers cause bronchoconstriction leading to wheeze and SOB. it is
contraindicated in asthma.


Additional
Information
Reference GPnotebook
Dr Khalid/Rabia

80
CVS-System Wise 1700-by Sush and Team. 2016
Susmita, Asad, Manu, Saima, Zohaib, Savia, Shanu, Mona, Manisha, Sitara, Samreena, Sami and Komal









CCBs (GPNotebook)
nifedipine treatment of hypertension, angina and Raynaud's phenomenon.

An common side effect is swelling of the ankles.


Nimodipine has a neuroprotective activity
verapamil treatment of supraventricular arrhythmias, angina, hypertension and
in the treatment of paroxysmal tachyarrhythmias
Side effects include:
constipation - most common side effect
nausea
vomiting
flushing
headache
dizziness
fatigue
ankle oedema
diltiazem treatment of angina.

Q:926 . A child was brought in to ED by his parents for taking his grand-dads meds.
There is an extra
systole in the ECG. Which drug was taken?
a. Digoxin
b. Amitriptyline
c. Atenolol
d. Ramipril
e. Bendroflumethiazide

Clincher(s) Estra systole in ecg
A In digoxin toxicity, the finding of frequent premature ventricular beats (PVCs)
is the most common and the earliest dysrhythmia. Sinus bradycardia is also
very common. In addition, depressed conduction is a predominant feature of
digoxin toxicity.
B Amitriptyline (TCA- and Ramitrimil)
Sinus tachycardia is the most common ECG finding
Widening of the PR, QRS, and QTc intervals; nonspecific ST-segment and T-
wave changes
C Atenolol
bradycardia (slow heartbeat), severe hypotension with shock, acute heart

81
CVS-System Wise 1700-by Sush and Team. 2016
Susmita, Asad, Manu, Saima, Zohaib, Savia, Shanu, Mona, Manisha, Sitara, Samreena, Sami and Komal

failure, hypoglycemia and bronchospastic reactions.
D Ramipril
hypotension, tachycardia, hyperkalaemia, and acute renal failure.

E Bendroflumethiazide- bendroflumethiazide is a thiazide diuretic. ADVERSE
EFFECTS ;hyperGLUC
KEY a. Digoxin- Other ECG changes that suggest digoxin toxicity include bigeminal
and trigeminal rhythms, ventricular bigeminy, and bidirectional ventricular
tachycardia
Features
generally unwell, lethargy, nausea & vomiting, anorexia, confusion,
yellow-green vision
arrhythmias (e.g. AV block, bradycardia)
Management
Digibind, correct arrhythmias, monitor potassium
Additional
Information
Reference
Dr Khalid/Rabia DR KHALIDNOTE - Both digoxin and amitryptiline can cause extrasystole


Q:928 A HTN pt on bendroflumethiazide 2.5mg/d has come for his routine checkup.
Exam:
BP=145/85mmHg. Lab: K+=5.9 (high), Na+=137. What is the most appropriate
management for this pt?
a. Stop meds
b. Continue same dose
c. Increase the dose
d. Decrease the dose
e. Repeat the blood test

Two doses available 2.5 , 5 mg
Clincher(s) , thiazide diuretic ,K=(3.5-5mmol/l) raised , NA=135-145mmol/l normal
A
B
C if blood pressure is not adequately controlled by a low dose of thiazide, an
additional antihypertensive agent should be considered rather than increasing
the dose.
Patient.info
D
E
KEY Repeat the blood test
Repeat the test as thiazides cause hypokalemia

(imp) Common adverse effects of thuazide HyperGLUC
dehydration

82
CVS-System Wise 1700-by Sush and Team. 2016
Susmita, Asad, Manu, Saima, Zohaib, Savia, Shanu, Mona, Manisha, Sitara, Samreena, Sami and Komal

postural hypotension
hyponatraemia, hypokalaemia, hypercalcaemia
gout
impaired glucose tolerance
impotence


Additional
Information
Reference
Dr Khalid/Rabia .DR KHALID - Repeat the blood test. [Bendroflumethiazide causes
hyponatremia and hypokalemia. But the findings are opposite which is
probably error of test. Hence blood test should be repeated to confirm the
level of potassium and sodium].


Q:937 A 57yo man having HTN on oral anti-HTN. However, he is finding it difficult to
mobilize as he feels dizzy whenever he tries to get up. What is the most
appropriate inv for him?
a. Ambulatory BP
b. ECG
c. MRI
d. CXR
e. CT

Clincher(s) DIZZY- postural hypotension (known side effect of anti HTN)
A
B
C
D
E
KEY a. Ambulatory BP

Additional
Information Who should be referred for ambulatory blood pressure monitoring?
The National Institute for Health and Care Excellence (NICE)
recommends that if a clinic blood pressure is 140/90 mmHg or higher,
ABPM should be offered to confirm the diagnosis of hypertension. If a
person is unable to tolerate ABPM, home blood pressure monitoring
(HBPM) is a suitable alternative to confirm the diagnosis of
hypertension.[5]
Poorly controlled hypertension - eg, suspected drug resistance.
Patients who have developed target organ damage despite control of
blood pressure.
Patients who develop hypertension during pregnancy.

83
CVS-System Wise 1700-by Sush and Team. 2016
Susmita, Asad, Manu, Saima, Zohaib, Savia, Shanu, Mona, Manisha, Sitara, Samreena, Sami and Komal

High-risk patients - eg, those with diabetes mellitus, those with
cerebrovascular disease, and kidney transplant recipients.
Suspicion of white coat hypertension - high blood pressure readings in
clinic which are normal at home.
Suspicion of reversed white coat hypertension, ie blood pressure
readings are normal in clinic but raised in the patient's own
environment.
Postural hypotension.
Elderly patients with.sys hypertension
Reference
Dr Khalid/Rabia


Q:962 A 53yo lady presents with hot flash and night sweats. Her LMP was last year.
She had MI recently. What is the most appropriate management for her?
a. Raloxifene
b. Estrogen
c . COCP
e-clonidine
d-primrose oil


Clincher(s) Menopause- 53 yr lady , hot flashes , night sweats , LMP last yr ,Recent MI
A Raloxifene- is a selective estrogen receptor modulator (SERM). It works in
osteoporosis by decreasing bone breakdown and thinning that may occur in
women after menopause. It works to decrease the risk of invasive breast
cancer by blocking estrogen in breast and uterine tissue.
B Estrogen contra indicated if you get angina chest pain, or if you have had a
heart attack (post MI) and usually for patients who had hysterectomy
http://patient.info/medicine/oestrogen-hrt-estradiol-estriol-estrone
C COCP- indicated for contraception .
Contra ind if had a heart attack or stroke or had angina or circulation problems
in your legs (peripheral arterial disease)
D Primrose oil- can be given for hot flushes . (not medical)
E Clonidine centrally acting alpha-agonist hypotensive agents. indicated for
cardiac sym & vasomotor symp
KEY Ans : Correct Answer is Clonidine
.Clonidine has cardiac uses like antihypertensive and diagnosis of
pheochromocytoma (by reducing nor epinephrine) And non cardiac uses like
post menopausal vasomotor symptoms ,opiod withdrawal , diabetic diarrhea ,
smoking cessation, and analgesia

Additional HRT contraindicated in pts with recent MIs or ischemic episodes.
Information
Contraindications:

84
CVS-System Wise 1700-by Sush and Team. 2016
Susmita, Asad, Manu, Saima, Zohaib, Savia, Shanu, Mona, Manisha, Sitara, Samreena, Sami and Komal

pregnancy
undiagnosed abnormal vaginal bleeding
active thromboembolic disorder or acute-phase myocardial infarction
suspected or active breast or endometrial cancer
active liver disease with abnormal liver function tests
porphyria cutanea tarda

Reference http://www.drugs.com/cdi/raloxifene.html
Dr Khalid/Rabia


Q:982 A 50yo female had swelling in her ankles. She is a known alcoholic. Now she
presented with breathlessness and palpitations. What is the most likely cause
of her condition?
a. VT
b. SVT
c. A-flut
d. A-FIB
e. V-ectopics

Clincher(s) 50 yr female ,swelling in ankles, alcoholic , breathlessnees , palpitations
A
B
C
D
E
KEY D- AFIB
Most Common Arryhtmias : Afib , Aflutter sometimes SVts.
Increased alcohol promotes alcoholic cardiomyopathy, heart failure and
arrhythmias.
S/S Alcoholic Cardiomyopathy
shortness of breath
swelling of the legs
rapid and irregular heartbeat
rapid and irregular pulse
fatigue, weakness, dizziness, fainting
an enlarged liver
cough that produces a frothy, pink mucus
Additional Ans D (with alcoholism most common cause of A fib)
Information
Add on info: in younger patients: Holiday Heart Syndrome : Holiday heart
syndrome is an irregular heartbeat pattern presented in individuals who are
otherwise healthy. Coined in 1978.the term is defined as arrhythmias
sometimes following excessive alcohol consumption; usually temporary"
Holiday heart syndrome can be the result of stress, dehydration, and drinking
alcohol. It is sometimes associated with binge drinking common during the

85
CVS-System Wise 1700-by Sush and Team. 2016
Susmita, Asad, Manu, Saima, Zohaib, Savia, Shanu, Mona, Manisha, Sitara, Samreena, Sami and Komal

holiday season. Cocain will also give. Afib will occur.

Reference
Dr Khalid/Rabia Dr Khalid - Ankle swelling suggests alcoholic cardiomyopathy and alcoholism is
also a well known cause of atrial fibrillation


Q: 1053 A 42yo lady had corrective surgery for cyanotic congenital heart disease at the
age of 3y, after a palliative operation during infancy. There is a parasternal
impulse and an early diastolic murmur. What is the most probable dx?
a. Aortic regurgitation
b. Ischemic mitral regurgitation
c. Aortic stenosis
d. Pulmonary stenosis
e. Pulmonary regurgitation

Clincher(s)
A Blowing diastolic murmur(mid diastolic rumble aka Austin flint murmur ) &
mid systolic apical murmur
B Holo systolic murmur (pansys) rad to axilla
C syst murmur rad to carotids
D harsh crescendo-decrescendo ejection murmur is audible and is heard best at
the left parasternal 2nd (valvular stenosis) or 4th (infundibular stenosis)
intercostal space with the diaphragm of the stethoscope when the patient
leans forward. Unlike the aortic stenosis murmur, a PS murmur does not
radiate, and the crescendo component lengthens as stenosis progresses.
E Early diastolic murmur in pulmonary regurgitation,
KEY Dx is pulmonary regurgitation.
42 yo lady has parasternal impulse and early diastolic murmur. Received
corrective surgery for cyanotic congenital heart disease (pulmonary
hypertension, Eisenmenger syndrome)
.
Causes of pulmonary hypertension also cause pulmonary regurgitation.
{Graham steell murmur if mitral stenosis+ pulmonary hypertension
Additional D-ARMS (diastolic- aortic regur & mitral stenosis)
Information S-MRAS (systolic mitral regurg & aortic stenosis)

PASS and PAID
p A stonis is systolic
p A insuffucency is dystolic

Eisermanger synd-Eisenmenger syndrome refers to any untreated congenital
cardiac defect with intracardiac communication that leads to
pulmonary hypertension, reversal of flow, and cyanosis.[1, 2, 3] The previous left-
to-right shunt is converted into a right-to-left shunt secondary to elevated
pulmonary artery pressures and associated pulmonary vascular disease

86
CVS-System Wise 1700-by Sush and Team. 2016
Susmita, Asad, Manu, Saima, Zohaib, Savia, Shanu, Mona, Manisha, Sitara, Samreena, Sami and Komal

Reference


Dr Khalid/Rabia

Q: 1058 A 65yo lady presents with a 6h hx of facial droop and weakness in the left side
of her body. What single agent will she be prescribed for her whole life?
a. Clopidogrel
b. Altepase
c. Aspirin
d. Labetalol


Clincher(s) 65 yr lady , 6hours facial droop , weakness on left TIA
A
B
C
D
E
KEY Rx is clopidogrel.
65 yo lady with facial droop and weakness on left side is suggestive of a stroke
/TIA : both prophylaxis is clopidegrel
Additional
Information
Reference
Dr Khalid/Rabia

87
CVS-System Wise 1700-by Sush and Team. 2016
Susmita, Asad, Manu, Saima, Zohaib, Savia, Shanu, Mona, Manisha, Sitara, Samreena, Sami and Komal


TOPICS to do from HERE


Q:1066 What is the most likely dx based on this ECG?
a. Normal
b. VT
c. Sinus Tachycardia
d. WPW syndrome
e. A-fib


Clincher(s) Ecg not given in the quest - taken from google


A Normal -
B Three or more consecutive pvcs , wide qrs complexes , in regular rapid thythm
Rx- cardioversion and anti-arrhythmics ( amiodarone)
C HR GREATER THAN 72 BPM
D SVT- (AVRT) atri-ventricular rentry tachycardia
a retrograde p wave seen after a normal QRS.
a preexcitation delta wave is seen characteristically in WPW. Delta wave
slurring slow rise of initial portion of the QRS
E IRREGULAR pulse , no p wave , tachycardia
KEY Key: C
No other apparent abnormaliTY

Additional SVT NORMAL QRS ( afib , wpw synd , sinus tachyc)
Information VT- WIDE QRS COMPLEX

88
CVS-System Wise 1700-by Sush and Team. 2016
Susmita, Asad, Manu, Saima, Zohaib, Savia, Shanu, Mona, Manisha, Sitara, Samreena, Sami and Komal

Reference


Dr Khalid/Rabia WPW syndrome:
Is an SVT
Due to reentry phenomena
If there is accessory pathy, it forms a reentry path
Delta way caused
Just before QRS complex
A-V: anterograde
V-A: retrograde


Q:1363 In lyme disease, which complication is most likely to lead to
collapse?
a. Dilated CM
b. AV block
c. Mild encephalitis
d. Meningitis
e. Myocarditis


Clincher(s) cardiac complication of lyme disease.
A Dilated cardiomyopathy is associated with viral infections,
autoimmune diseases, per/postpartum thyrotoxicosis. It does not
causes sudden collapse. Hypertrophic cardiomyopathy lead to
sudden death.

89
CVS-System Wise 1700-by Sush and Team. 2016
Susmita, Asad, Manu, Saima, Zohaib, Savia, Shanu, Mona, Manisha, Sitara, Samreena, Sami and Komal

B AV block is complication of lyme disease which only lead to
collapse in patients. (infection damages the AV and hampers the
conductive pathway)
C mild encephalitis has no association with lyme disease.
D meningitis is a complication of lyme disease.
E acute myocarditis is a complication of lyme disease.
KEY B. AV block (all others are complications)
Additional
Information

Reference OHCM PG 430


Dr Khalid/Rabia Hypertropic cardiomyopathy leads to sudden collapse and then
sudden death


90
CVS-System Wise 1700-by Sush and Team. 2016
Susmita, Asad, Manu, Saima, Zohaib, Savia, Shanu, Mona, Manisha, Sitara, Samreena, Sami and Komal

Q: 1372 A 50yo man came to the hosp a few months after he had a MI.
Exam: everything normal, S1 and S2 were heard on auscultation,
but there is a new pan-sytolic murmur. What is the most
appropriate inv of choice?
a. ECG
b. 24h ECG
c. Echo
d. CXR
e. CT


Clincher(s) H/o MI and pansystolic murmur.
A no role
B no role
C best/appropriate investigation of choice for diagnosing the cause of
pan systolic murmur.
D no role
E no role
KEY c- Echo.
Additional Most common post MI complication is cardiac septal wall rupture
Information which lead to Mitral regurgitation. Pan systolic murmur is a
clinical feature of this valvular insufficiency.
Reference
Dr Khalid/Rabia For every murmur you need to do echo.



Q: 1381 A man after MI presented with sudden breathlessness and
dyspnea. Exam: scattered pansystolic murmur all over the
precordium. What is the next inv that will lead to dx?

a. ECG
b. Echo
c. CT
d. Blood culture

e. CXR

91
CVS-System Wise 1700-by Sush and Team. 2016
Susmita, Asad, Manu, Saima, Zohaib, Savia, Shanu, Mona, Manisha, Sitara, Samreena, Sami and Komal

Clincher(s) signs of cardiac failure. most likely differential is valular
insufficiency.
A does not give any clue.
B best for diagnosing.
C
D
E
KEY B- ECHO
Additional
Information
Reference
Dr Khalid/Rabia



Q: 1394 A man is brought to the ED after he was stabbed in the back. Chest
is clear bilaterally with muffled heart sounds. BP=60/0mmHg,
pulse=120bpm, JVP is raised. What is the most probable dx?

a. Pulmonary embolism

b. Cardiac tamponade

c. Pericardial effusion

d. Hemothorax

e. Pneumothorax


Clincher(s) chest stab on back, muffled heart sounds, BP 60/0, raised JVP
A
B typical picture of cardiac tamponade
C
D

92
CVS-System Wise 1700-by Sush and Team. 2016
Susmita, Asad, Manu, Saima, Zohaib, Savia, Shanu, Mona, Manisha, Sitara, Samreena, Sami and Komal

E
KEY B- cardiac tamponade
Additional
Information

Reference ohcm pg 814


Dr Khalid/Rabia In hemopnumothorax: pic like cardiac temponade (no pumping
action of blood), but BP will be lower

tension thorax (one way valve): also raised raised JVP, BP and pulse
is equivocal



Q: 1396 A 38yo man presented to ED with severe pain in upper abdomen.
He has already taken course of triple therapy and now had elective
endoscopy 2d ago. He is in shock. What is the most
probable dx?
a. Ca esophagus
b.Barr et s esophagus
c. Mediastinitis
d. Ca stomach


Clincher(s) h/o GERD, abdominal pain after upper GI endoscopy
A

93
CVS-System Wise 1700-by Sush and Team. 2016
Susmita, Asad, Manu, Saima, Zohaib, Savia, Shanu, Mona, Manisha, Sitara, Samreena, Sami and Komal

B
C
D Oesophageal rupture leading to mediastinitis.
E
KEY
Additional Asking for complication of endoscopy
Information

Always suspect mediastinitis in a pt with shock and fever after a hx


of - Recent cardiothoracic surgery or instrumentation.

- Upper GI endoscopy.

- Bronchoscopy.

- Recent dental or oropharyngeal infection.

- Upper respiratory tract infection

- Ingestion of a foreign body (particularly button batteries by young


children,

which may cause oesophageal rupture).

other signs may include edema of the neck and face , and crunching
sound.

Reference
Dr Khalid/Rabia


Q: 1412 A 68yo man on tx for an irregular heart beat comes to the ED. He
has palpitations for the last 3h. Exam: pulse=regular, 154bpm.
Carotid sinus massage settled his pulse down to 80bpm. What is
the most likely rhythm disturbance?
a. SVT
b. V-fib
c. VT
d. V-ectopics
e. A-fib

94
CVS-System Wise 1700-by Sush and Team. 2016
Susmita, Asad, Manu, Saima, Zohaib, Savia, Shanu, Mona, Manisha, Sitara, Samreena, Sami and Komal

Clincher(s) H/o irregular heart beat, tachycardia. (Only presentation of SVT)
A
B irregular pulse and it is associated with heart disease.
C sinus tachycardia but occur within 48 hour of MI.
D post-MI.
E irregular pulse and it is associated with heart disease.
KEY A- SVT
Additional During an attack, tachycardia may be the only finding if the patient
Information is otherwise healthy and there is no cardiac dysfunction. During an
episode of SVT the pulse rate is 140-250 beats per minute (bpm).In
haemodynamically stable regular narrow QRS-complex tachycardia,
vagal manoeuvres - eg, Valsalva, carotid massage, facial immersion
in cold water.

Reference
Dr Khalid/Rabia To differentiate VT from SVT:

VT- hx of MI (within 48 hrs of MI- clincher)

Carotid sinus massage: block AV node> conduction lost, if pulse
goes down: always think of SVT
(regular pulse points to SVT)

In ventricular ectopic: will continue whether we block AV or not




Q: 1417 A 76yo man is in the CCU 2d after an acute MI. He tells you that he
had an episode of rapid pounding in the chest lasting for about
2mins. He remains conscious throughout. What is the
most likely rhythm?
a. SVT
b. VF
c. VT
d. V-ectopics
e. A-fib

95
CVS-System Wise 1700-by Sush and Team. 2016
Susmita, Asad, Manu, Saima, Zohaib, Savia, Shanu, Mona, Manisha, Sitara, Samreena, Sami and Komal

Clincher(s) occur within 48hour of acute MI.
A weak association
B late complication
C
D late complication of anterior wall MI.
E old age and sever heart disease.
KEY C- VT
Additional Complication:
Information Early: VT
Late: Vfib, Ven ectopic, Afib

Ventricular tachycardia (VT) is broad-complex tachycardia
originating from a ventricular ectopic focus. It has the potential to
precipitate ventricular fibrillation and hence requires urgent
treatment.


There are two main types of VT:

monomorphic VT: most commonly caused by myocardial


infarction

polymorphic VT: A subtype of polymorphic VT is torsades de


pointes which is precipitated by prolongation of the QT
interval.

Management:

If the patient has adverse signs(systolic BP< 90mm hg, chest pain,
heart failure or rate> 150 beats/min) then immediate
cardioversion is indicated. In the absence of such signs anti
arrhythmic drugs may be used. if the medical cardioversion
does not work, then electrical cardioversion can be offered.

Drug therapy:

amiodarone: ideally administered through central line.

lidocaine: use with caution in severe left ventricular impairement.

procainamide.

Verapamil should not be used in VT.

96
CVS-System Wise 1700-by Sush and Team. 2016
Susmita, Asad, Manu, Saima, Zohaib, Savia, Shanu, Mona, Manisha, Sitara, Samreena, Sami and Komal

Reference
Dr Khalid/Rabia


97
CVS-System Wise 1700-by Sush and Team. 2016
Susmita, Asad, Manu, Saima, Zohaib, Savia, Shanu, Mona, Manisha, Sitara, Samreena, Sami and Komal

Q: 1446 A 54yo man comes with sudden onset of palpitations and
breathlessness. His HR=164bpm. What is the single most
appropriate tx in the acute phase?
a. Adenosine
b. Metaprolol
c. Verapamil
d. Amiodarone


Clincher(s)
A Adenosine. sudden onset palpitations and breathlessness is SVT.
For those not hemodynamically compromised after Vagal
Maneuvers first appropriate treatment in acute phase is
Adenosine.

For actue phase (after carotid massage) we give adonesine and


then verapamil,

If unstable: Electrical cario

Ecg will be given usually.

B
C
D
E
KEY A- adenosine. (SVT as there is not hx of MI, ht disease)

98
CVS-System Wise 1700-by Sush and Team. 2016
Susmita, Asad, Manu, Saima, Zohaib, Savia, Shanu, Mona, Manisha, Sitara, Samreena, Sami and Komal

Additional SVT is usually paroxysmal and episodes may occur regularly or very
Information infrequently (sometimes years apart). Episodes may only last for a
few minutes or may last for up to several months.

Episodes are characterised by the sudden onset of a narrow


complex tachycardia, typically an atrioventricular nodal reentry
tachycardia (AVNRT). Other causes include atrioventricular re-entry
tachycardias (AVRT) and junctional tachycardias.

Risk Factors= SVT may occur at any age but often occurs in younger
people in the absence of heart disease.

Risk factors include: Previous Myocardial infarction, mitral valve


prolapse, rheumatic heart diseases, pericarditis, pneumonia,
chronic lung disease, chronic alcohol intoxication, digoxin toxicity

Presentation: Palpitations and dizziness are the most common


symptoms reported.

Other symptoms including fatigue, light-headedness, chest


discomfort, dyspnoea, polyuria and syncope.

Investigations:

ECG= P wave absent or inverted after QRS.

Management= Vagal manoeuvres (breath-holding, valsalva


manoeuvre, carotid massage) are

1st line treatments if haemodynamically stable. IV adenosine is the


drug of choice.

If adenosine fails, use verapamil 5 mg IV over 2min, or over 3min if


elderly.

If haemodynamically unstable= DC (direct current) cardioversion.

Reference
Dr Khalid/Rabia


99
CVS-System Wise 1700-by Sush and Team. 2016
Susmita, Asad, Manu, Saima, Zohaib, Savia, Shanu, Mona, Manisha, Sitara, Samreena, Sami and Komal

Q: 1448 The artery that runs in the ant inter-ventricular groove. What is
the single most appropriate option?
a. Acute marginal branch
b. Left ant descending artery
c. Coronary sinus
d. Circumflex artery
e. Right coronary artery


Clincher(s) Straightforward anatomy question
A branch of right coronary artery and follows the acute margin of
heart.
B Left anterior descending artery run in the anterior inter-
ventricular groove.
C left posterior atrioventricular groove.
D follows the course of coronary sinus.
E right atrioventricular groove.
KEY B- left anterior descending artery.
Additional
Information

Reference

100
CVS-System Wise 1700-by Sush and Team. 2016
Susmita, Asad, Manu, Saima, Zohaib, Savia, Shanu, Mona, Manisha, Sitara, Samreena, Sami and Komal

Dr Khalid/Rabia Rt coronary artry to SA node: so called dominant artery
To the muscles is ant wall ht; left desecinding artery (left CA)

Rt coronary to atrium and AS margin aand lowere border of
heart/LV







Q:1252 A 59yo pt has been dx with HTN. His BP has been >160/90 mmHg on 3
separate occasions. His biochemical profile is as follows: Na+=145 mmol/l,
K+=6.2mmol/l,
creatinine=112 umol/l, urea=5.7mmol/l. What is the most appropriate anti-
HTN drug for him?
a. Amlodipine
b. Bendroflumethiazide
c. Ramipril
d. Lorsartan
e. Propranolol

Clincher(s) Treated for HTN ,hyperkalaemia,high BP
A I think amlodipine is the drug of choice here as the kidneys are not functioning
due to high creat and potassium levels except for pheripheral oedema it has
no other side effect
B PT has hyperkalaemia so if thiazides diuretics are given it will lower BP AND
ALSO high K conc hence the drug of choice in such patients ( is the explanation
given by RABIA.

Thiazide will lower the K level
C ACE inhibitors are contraindicated in kidney diseases as the creatinine and
potassium levels are high
D CANT be given in increased k levels and creatinine levels
E Is beta blocker so pat kidneys are not functioning increase chances of heart
failure so not indicated
KEY B (skype decision)
Additional From pass medicine;
Information
Amlodipine;YES
THIAZIDES: NO AS KIDNEYS ARE NOT FUNCTIONING
ENALAPRIL : NO AS CREAININE AND K LEVELS ARE HIGH
PROPRANOLOL: NO AS ITS BETA BLOCKER
SPIROOLACTONE ;NO IF K LEVELS ARE HIGH

101
CVS-System Wise 1700-by Sush and Team. 2016
Susmita, Asad, Manu, Saima, Zohaib, Savia, Shanu, Mona, Manisha, Sitara, Samreena, Sami and Komal

AMLODIPINE; is used with or without other medications to treat high blood


pressure. Lowering high blood pressure helps prevent strokes, heart attacks,
and kidney problems. Amlodipine belongs to a class of drugs known as calcium
channel blockers. It works by relaxing blood vessels so blood can flow more
easily.

Amlodipine is also used to prevent certain types of chest pain (angina). It may
help to increase your ability to exercise and decrease the frequency of angina
attacks. It should not be used to treat attacks of chest pain when they occur.
Use other medications (such as sublingual nitroglycerin) to relieve attacks of
chest pain as directed by your doctor

Side Effects

Dizziness, lightheadedness, swelling ankles/feet, or flushing may occur. If any


of these effects persist or worsen, tell your doctor or pharmacist promptly.

To lower your risk of dizziness and lightheadedness, get up slowly when rising
from a sitting or lying position.

Remember that your doctor has prescribed this medication because he or she
has judged that the benefit to you is greater than the risk of side effects. Many
people using this medication do not have serious side effects.

Tell your doctor right away if any of these unlikely but serious side effects
occur: fast/irregular/pounding heartbeat, fainting.

Although this medication is effective in preventing chest pain (angina), some


people who already have severe heart disease may rarely develop worsening
chest pain or a heart attack after starting this medication or increasing the
dose. Get medical help right away if you experience: worsening chest pain,
symptoms of a heart attack (such as chest/jaw/left arm pain, shortness of
breath, unusual sweating).

A very serious allergic reaction to this drug is rare. However, get medical help
right away if you notice any symptoms of a serious allergic reaction, including:
rash, itching/swelling (especially of the face/tongue/throat), severe dizziness,
trouble breathing.

This is not a complete list of possible side effects. If you notice other effects
not listed above, contact your doctor or pharmacist


Reference PATIENT .CO.UK

102
CVS-System Wise 1700-by Sush and Team. 2016
Susmita, Asad, Manu, Saima, Zohaib, Savia, Shanu, Mona, Manisha, Sitara, Samreena, Sami and Komal

Dr Khalid/Rabia
DR RABIAS NOTES
the patient has hyperkalemia so as in order to correct that thiazide should be given
as thiazide cause hypokalemia. so it B bendroflumethiazide although recent
guidelines now recommend other thiazide like diuretics such as Indapamide and
chlortalidone but it's an old question so bendroflumethiazide is the answer.
Thiazide Diuretics Thiazide diuretics work by inhibiting sodium absorption at the
beginning of the distal convoluted tubule (DCT). Potassium is lost as a result of more
sodium reaching the collecting ducts. Thiazide diuretics have a role in the treatment
of mild heart failure although loop diuretics are better for reducing overload. The
main use of bendroflumethiazide was in the management of hypertension but
recent NICE guidelines now recommend other thiazide-like diuretics such as
indapamide and chlortalidone.
Common adverse effects
dehydration
postural hypotension
hyponatraemia, hypokalaemia, hypercalcaemia
gout
impaired glucose tolerance
impotence

Rare adverse effects
thrombocytopenia
agranulocytosis
photosensitivity rash
pancreatitis

Q:1254 A 74yo man presents with sudden onset of with right sided weakness and
slurred speech. He also has loss of sensation over the right side of the body
and visual field defects. CT shows ischemic stroke. What is the most
appropriate management?
a. Alteplase
b. Streptokinase
c. Nimodipine
d. Aspirin
e. Labetalol

Clincher(s) CT SCAN showing ischemic stroke changes
A To be given if the duration is presented in question according to guidelines
alteplase is given if the patient is presented in less than 4.5 hours
B Same thing again as above
But if its acute MI it is the drug of choice
C Drug of choice for htn no significant clinical indication in stroke patients
D Is indicated if its not the haemorrhagic stroke
E Is a beta blocker not indicated
KEY D ASPIRIN (if less than 4.5 hrs: alteplase given)

103
CVS-System Wise 1700-by Sush and Team. 2016
Susmita, Asad, Manu, Saima, Zohaib, Savia, Shanu, Mona, Manisha, Sitara, Samreena, Sami and Komal

Additional ACUTE MI; STREPTOKINASE ,Tpa
Information ACUTE ISCHAEMIC STROKE: Tpa
Acute pulmonary embolism: SK,UK,Tpa that is alteplase
ACUTE DVT: SK
CLOTTED AV FISTULAS AND SHUNTS: UK
UK IS UROKINASE
SK IS STREPTOKINASE
Reference Aspirin. The CT scan shows ischemic stroke and time when patient presented is
not given if it would have less than 4.5hrs we would have given alteplase.
STROKE MANAGEMENT The Royal College of Physicians (RCP) published
guidelines on the diagnosis and management of patients following a stroke in
2004. NICE also issued stroke guidelines in 2008, although they modified their
guidance with respect to antiplatelet therapy in 2010.
Selected points relating to the management of acute stroke include:
blood glucose, hydration, oxygen saturation and temperature should be
maintained within normal limits
blood pressure should not be lowered in the acute phase unless there are
complications e.g. Hypertensive encephalopathy*
aspirin 300mg orally or rectally should be given as soon as possible if a
haemorrhagic stroke has been excluded
with regards to atrial fibrillation, the RCP state: 'anticoagulants should not be
started until brain imaging has excluded haemorrhage, and usually not until 14
days have passed from the onset of an ischaemic stroke'
if the cholesterol is > 3.5 mmol/l patients should be commenced on a statin.
Many physicians will delay treatment until after at least 48 hours due to the
risk of haemorrhagic transformation

Thrombolysis
Thrombolysis should only be given if:
it is administered within 4.5 hours of onset of stroke symptoms (unless as
part of a clinical trial)
haemorrhage has been definitively excluded (i.e. Imaging has been
performed)

Alteplase is currently recommended by NICE.
Secondary prevention
NICE also published a technology appraisal in 2010 on the use of clopidogrel
and dipyridamole
Recommendations from NICE include:
clopidogrel is now recommended by NICE ahead of combination use of
aspirin plus modified release (MR) dipyridamole in people who have had an
ischaemic stroke
aspirin plus MR dipyridamole is now recommended after an ischaemic stroke
only if clopidogrel is contraindicated or not tolerated, but treatment is no
longer limited to 2 years' duration

104
CVS-System Wise 1700-by Sush and Team. 2016
Susmita, Asad, Manu, Saima, Zohaib, Savia, Shanu, Mona, Manisha, Sitara, Samreena, Sami and Komal


MR dipyridamole alone is recommended after an ischaemic stroke only if
aspirin or clopidogrel are contraindicated or not tolerated, again with no limit
on duration of treatment

With regards to carotid artery endarterectomy:
recommend if patient has suffered stroke or TIA in the carotid territory and
are not severely disabled
should only be considered if carotid stenosis > 70% according ECST** criteria
or > 50% according to NASCET*** criteria


Dr Khalid/Rabia PATINET.CO.UKAND RABIA


Q:1255 . The artery that runs along the left AV groove. What is the single most
appropriate option?
a. Left internal mammary artery
b. Left anterior descending artery
c. Circumflex artery
d. Left main stem (LMS) post descending artery
e. Diagonal branch

Clincher(s) LEFT AV GROOVE
A Internal mammary artery arises from cervical part of subclavian artery at the
lateral edge of scalenes anterior muscle it then traverse first rib to enter
thorax
B Is in interventricular groove
C Is in av groove
D Posterior interventricular artery or posterior descending artery is an artery
running in the posterior interventricular sulcus to the apex of heart where it
meets anterior inter ventricular artery and supplies posterior 1/3rd of
interventricular septum and remaining 2/3rd is supplied by anterior
interventricular artery which is a branch of left anterior descending artery
which is abranch of left coronary artery
E ?
KEY Answer: C. Circumflex Artery. Left Anterior Descending artery descends into
interventricular groove.

105
CVS-System Wise 1700-by Sush and Team. 2016
Susmita, Asad, Manu, Saima, Zohaib, Savia, Shanu, Mona, Manisha, Sitara, Samreena, Sami and Komal

Additional
Information



Reference


Dr Khalid/Rabia


Q:1257 A pt, 50yo smoker and heavy drinker, presents with complaints of racing heart.
A 24h EKG comes out normal. What is your next step in management?
a. Echo
b. Reassure
c. Stress test

Clincher(s) EKG is normal no significant past history of any heart attack
A Is not the first investigation to determine eitiology and treatment in heart
patients with pain

106
CVS-System Wise 1700-by Sush and Team. 2016
Susmita, Asad, Manu, Saima, Zohaib, Savia, Shanu, Mona, Manisha, Sitara, Samreena, Sami and Komal

B As ekg is normal and no heart pain and no past history as well so just reassure
C Is the treatment ofchoice if the patient in question complains of heart pain or
past history of any cardiac events
D
E
KEY Answer: B.Reassure. Everything is normal in this patient so reassure the
patient. If there would have been chest pain or past h/o chest pain then stress
test could be a viable option.
Additional
Information Echocardiography (Echo) is a key diagnostic tool in the diagnosis of heart
failure and can determine the aetiology and help plan treatment for patients.
Echo is also indicated for heart murmurs, atrial fibrillation and hypertension.
12-lead electrocardiogram (ECG) is also available to aid in the diagnosis,
however normal ECG does not exclude serious cardiac disease.
Ambulatory ECG is available to assist in evaluating arrhythmias.
Ambulatory Blood Pressure monitoring is also available to assist in the
assessment and management of patients with known or suspected disorders
of blood pressure regulation
Reference https://www.inhealthgroup.com/sites/default/files/contract_services/london_
nhs_diagnostic_service/clinical_library/gp_guide_cardiovascular_050813_103
5.pdf
above link shows details of management
Dr Khalid/Rabia Nhs


Q:1273 A 34yo primigravida who is 16wk GA comes for routine antenatal check up.
Her
BP=160/100mmHg. She has a hx of repeated childhood UTI. What is the most
likely cause of her
high BP?
a. Essential HTN
b. Chronic pyelonephritis
c. Acute pyelonephritis
d. Pre-eclampsia
e. Chronic UTI

Clincher(s) HIGH BP,REPEATED UTI IN CHILD HOOD Shows chronic state
A Is also called idiopathic hypertension that by definition has no identifiable
cause
B Repeated uti leads to chronic pyelonephritis
C Its not acute as patient has repeated UTI shows chronic state
D Its increased BP FROM 20 weeks of pregnancy or soon after delivery with
proteinuria so excluded
E Its the repeated infections of urinary tract even though patient is on right
treatment

107
CVS-System Wise 1700-by Sush and Team. 2016
Susmita, Asad, Manu, Saima, Zohaib, Savia, Shanu, Mona, Manisha, Sitara, Samreena, Sami and Komal

KEY B.
Recurrent episodes of UTI point towards chronic pyelonephritis.
Pre eclampsia has HTN with proteinuria/oedema after 20 weeks of gestation.
Risk Factors:
any structural renal tract anomalies, obstruction or calculi
Children with vesicoureteral reflux
Intrarenal reflux in neonates
Genetic predisposition
Any factors predisposing to recurrent urinary infection - eg, neurogenic
bladder

Presentation:
Fever
Malaise
Loin pain
Nausea
Vomiting
Dysuria
Hypertension
Failure to thrive

Additional So repeated UTI leads to renal tract obstruction and calculi leading to
Information pyelonephritis
Reference Rabia
Dr Khalid/Rabia

Q:1307 A 46yo man with tachycardia has the following ECG. What is the most likely
dx?
a. SVT
b. VT
c. Mobitz I heart block
d. Atrial fibrillation
e. WPW syndrome

Clincher(s)
A
B
C
D
E
KEY
Additional No answer is given and no ecg to interpret sorry
Information
Reference
Dr Khalid/Rabia

108
CVS-System Wise 1700-by Sush and Team. 2016
Susmita, Asad, Manu, Saima, Zohaib, Savia, Shanu, Mona, Manisha, Sitara, Samreena, Sami and Komal


Q:1333 A new born bay is borught with pansystolic murmur at sternal border but the
baby is not cyanosed. What is the dx?
a. VSD
b. ASD
c. TOF
d. PDA

Clincher(s) Non cyanotic ,pansystolic murmer
A Is a non cyanotic ,PANSYSTOLIC MURMER
B It causes ejection systolic murmer and is non cyanotic
C Cyanotic heart disease
It INCLUDES pulmonary stenosis
Right ventricular hypertrophy
VSD
Over riding of aorta
And clinically we have ejection systolic murmer
Fixed split second heart sound
Sounds like lub dalub lub dalub
D Is cyanotic causes constant machinery like murmer
E
KEY A


Non cyanotic heart disorders are:
VSD
PDA
PULMONARY STENOSIS
ASD
COARCTATION OF AORTA
AORTIC STENOSIS
Cyanotic heart diseses are:
TOF
Transposition of great vessels
Pansystolic murmer is caused by MR,TR,and VSD



Additional Pansystolic murmers are seen in the following :
Information Holosystolic (pansystolic)
mitral/tricuspid regurgitation (high-pitched and 'blowing' in character)
VSD ('harsh' in character)

VSD:
A ventricular septal defect (VSD) is a defect in the ventricular septum, the wall
dividing the left and right ventricles of the heart.

109
CVS-System Wise 1700-by Sush and Team. 2016
Susmita, Asad, Manu, Saima, Zohaib, Savia, Shanu, Mona, Manisha, Sitara, Samreena, Sami and Komal

The ventricular septum consists of an inferior muscular and superior
membranous portion and is extensively innervated with conducting
cardiomyocytes.
During ventricular contraction, or systole, some of the blood from the left
ventricle leaks into the right ventricle, passes through the lungs and reenters
the left ventricle via the pulmonary veins and left atrium. This has two net
effects. First, the circuitous refluxing of blood causes volume overload on the
left ventricle. Second, because the left ventricle normally has a much higher
systolic pressure (~120 mmHg) than the right ventricle (~20 mmHg), the
leakage of
blood into the right ventricle therefore elevates right ventricular pressure and
volume, causing pulmonary hypertension with its associated symptoms.
In serious cases, the pulmonary arterial pressure can reach levels that equal
the systemic pressure. This reverses the left to right shunt, so that blood then
flows from the right ventricle into the left ventricle, resulting in cyanosis, as
blood is by-passing the lungs for oxygenation.[7]
This effect is more noticeable in patients with larger defects, who may present
with breathlessness, poor feeding and failure to thrive in infancy. Patients with
smaller defects may be asymptomatic. Four different septal defects exist, with
perimembranous most common, outlet, atrioventricular, and muscular less
commonly
Inv :
Auscultation
Echo

Reference Rabia .nice
Dr Khalid/Rabia


Q: 119 A 4yo girl is found to have bounding pulse and continuous machinery murmur.
What is the most probable dx?
a. TOF
b. ASD
c. VSD
d. PDA
e. CoA
Ans.


Clincher(s)
A
B ASD ejection systolic and fixed splitting

C VSD has a pansystolic murmur

D Continuous Machinery murmur points toward / clincher for Patent ductus

110
CVS-System Wise 1700-by Sush and Team. 2016
Susmita, Asad, Manu, Saima, Zohaib, Savia, Shanu, Mona, Manisha, Sitara, Samreena, Sami and Komal

arteriosus PDA

E
KEY The key is D. PDA.
Additional PDA PRESENTATION: Usually asymptomatic. Acyanotic disease. A large-shunt
Information PDA may cause lower respiratory tract infection as well as feeding difficulties
and poor growth during infancy, with failure to thrive because of heart failure.

ECHO IS DIAGNOSTIC

MANAGEMENT: Indomethacin can be used but not useful in term infants. In
preterms indomethacin or ibuprofen may be used.

Closure is indicated if the patient is symptomatic at any stage of life or if
asymptomatic but with great left heart load.

Surgery is used where non surgical method can not be used. In asymptomatic
infant we wait till 1 yr for spontaneous closure of PDA if that does not occur it
can be closed by surgery at any time.

Most common complication is infective endocarditis.

Reference
Dr Khalid/Rabia


Q: 120 A 12yo child with episodes of sudden bluish discoloration and brief loss of
consciousness. Exam: clubbing, central cyanosis, systolic thrill with systolic
ejection murmur in 2nd left ICS. What is the most probable dx?
a. TOF
b. ASD
c. VSD
d. PDA
e. CoA




Clincher(s) Cyanosis, systolic ejection murmur
A TOF presents with ejection systolic murmur

ASD, VSD, PDA are all acyanotic congenital heart diseases. TOFF is the most
common cyanotic congenital heart disease that survives to adulthood.
B
C
D

111
CVS-System Wise 1700-by Sush and Team. 2016
Susmita, Asad, Manu, Saima, Zohaib, Savia, Shanu, Mona, Manisha, Sitara, Samreena, Sami and Komal

E
KEY Ans. The key A. TOF.

Additional Q in plab are in relation to cyanosis and other findings
Information

CONGENITAL HEART DISEASE
CLASSIFICATION
1. Cyanotic
2. Non-cyanotic

CYANOTIC: (R>L shunt) T (Dx: Echo Rx: Ionotropic drugs, surgery) Paediatric

1. Transposition of great vessel
(aorta and pulmonary trunks are in opposite places- usually
accompanied with VSD/ DA- so surgery needed in 1st 6 months of life)

2. Truncus arteriosus
(Aorta and PA still connected=TA one structure)

3. Tetralogy of Fallot (TOF most common cyanotic to survive to adult)
1.pulmonary stenosis (severity depends on this/initially this murmur),
2. right ventricular hypertrophy, 3. VSD, 4. over- riding aorta over VSD.
(acute emergency- Tets spell- squatting in child helps- typical of TOF)
Sign:
Child- Difficulty feeding, failure to thrive, clubbing, squat
Adult: unoperated: asymptomatic, cyanosed (not extreme)
Adult: operated: exertional dyspnea, palpitation, RV failure, syncope,
sudden death
TOF presents with ejection systolic murmur

Inv: ECG: RV hypertrophy with RBBB
x-ray N or shows boot shaped heart
Echo; degree of stenosis
Mx: Give o2, knee chest, morphine-to relax pul outflow, Long term B
blocker, Surgery by 1yrs
Eisenmengers syndrome/complex
4. Pulmonary atresia
(no Pul Valve to PA e VSD)

5. Tricuspid atresia

(Tricuscid not developed/valve blocked- has ASD & VSD)

progressive cyanosis
poor feeding

112
CVS-System Wise 1700-by Sush and Team. 2016
Susmita, Asad, Manu, Saima, Zohaib, Savia, Shanu, Mona, Manisha, Sitara, Samreena, Sami and Komal

tachypnea over the first 2 weeks of life
holosystolic murmur due to the VSD

6. TAPVR (Total Anomalous Pulmonary Venous Return)

(PVs enter into RA- has ASD)



NON CYANOTIC (L>R shunt)
1.-ASD
(Atrial septal defect)
sign: pulmonary ejection sys murmur
pul HTN can cause tricuspid or pul regurg
Eisenmengers syndrome/complex reversal of L>R shunt- causing
cyanosis, ht failure &chest inf, cause P HTN> emergency surgery
Inv: ECG: RBBB,LAD (left axis dev) with prolonged PR interval or RAD
Diagnostic: ECHO
Rx: child: closure before 10 years; Adult: Transcatheter closure

2.VSD (25% CHD)
Clinchers: non-cyanosed new born baby, harsh pansystolic murmur at
sternal border
UnRx or large holes-cause pul HTN, Eisenmengers syndrome/complex
Small vsd- ecg. Cxr normal; large: ven hypertrophy/cardiomegaly,
Rx. Medical, then surg


3.-Patent ductus arteriosus
(Continuous Machinery murmur)

4.-coarctation of aorta
cong narrowing of desc aorta; > boys,
Sign:
Radiofemoral delay (femoral later than radial), weak femoral pulse
>BP
systolic murmur (best over left scapula)/ scapular bruit
Complication: HF; infective endo
Inv: Ct or MRI aortogram; CXR- rib notching
Rx: Surgery/balloon dilatation+- stenting

5.-aortic stenosis



Reference OHCM- 150-151

113
CVS-System Wise 1700-by Sush and Team. 2016
Susmita, Asad, Manu, Saima, Zohaib, Savia, Shanu, Mona, Manisha, Sitara, Samreena, Sami and Komal

Dr Khalid/Rabia TOF Typical features (same as above):
1 Ventricular septal defect (VSD)
2 Pulmonary stenosis (most imp feature)
3 Right ventricular hypertrophy
4 The aorta overriding the VSD

During a hypoxic spell, the child becomes restless and agitated and may cry
inconsolably. Toddlers may squat, which is typical of TOF. Clubbing, difficulty
of feeding, failure to thrive all are features.

Chest Xray Shows BOOT SHAPED HEART. Echo is also done.
In acute stage give O2, place the child in knee chest position, give morphine.
Surgery is required within 1st yr of life otherwise mortality is 95%.



Q: 137 A 67yo man presents with palpitations. ECG shows an irregular rhythm and
HR=140bpm. He is otherwise stable, BP=124/80 mmHg. What is the most
appropriate management?
a. Bisoprolol
b. ACEi
c. Ramipril
d. Digoxin




Clincher(s)
A The patient has Atrial fibrillation. Irregularly irregular pulse and tachycardia.
In the given question since the patient is above the age of 65 so rate control
is done. For which either a beta blocker or calcium channel blocker is used.
(see below)

B
C
D
E
KEY Ans. The key is A. Bisoprolol.

Additional
Information
Reference
Dr Khalid/Rabia Agents used to control rate in patients with atrial fibrillation
beta-blockers
calcium channel blockers
digoxin (not considered first-line anymore as they are less effective at

114
CVS-System Wise 1700-by Sush and Team. 2016
Susmita, Asad, Manu, Saima, Zohaib, Savia, Shanu, Mona, Manisha, Sitara, Samreena, Sami and Komal

controlling the heart rate during exercise. However, they are the
preferred choice if the patient has coexistent heart failure)

Agents used to maintain sinus rhythm in patients with a history of atrial
fibrillation
sotalol
amiodarone
flecainide
others (less commonly used in UK): disopyramide, dofetilide,
procainamide, propafenone, quinidine

TREATMENT CHOICE:




Q: 140 A 67yo man had successful thrombolysis for an inf MI 1 month ago and was
discharged after 5 days. He is now re admitted with pulmonary edema. What is
the most probable dx?
a. Aortic regurgitation
b. Ischemic mitral regurgitation
c. Mitral valve prolapse
d. Pulmonary stenosis
e. Rheumatic mitral valve stenosis


115
CVS-System Wise 1700-by Sush and Team. 2016
Susmita, Asad, Manu, Saima, Zohaib, Savia, Shanu, Mona, Manisha, Sitara, Samreena, Sami and Komal



Clincher(s) Hx of inf MI thrombolysis (1 m ago). Now Pul oedema.
A
B MI leads to papillary muscle rupture which causes mitral regurgitation leading
to pulmonary edema

[Causes of Ischaemic mitral regurgitation: left ventricular remodeling and
dysfunction, annular dilation/dysfunction, and mechanical dyssynchrony].
C
D
E
KEY Ans. The key is B. Ischaemic mitral regurgitation
Additional
Information
Reference
Dr Khalid/Rabia Mitral regurgitation: May be mild (minor papillary muscle dysfunction) or
severe (chordal or papillary muscle rupture or ischaemia). Presentation:
Pulmonary oedema.

Treat LVF and consider valve replacement.

Mitral regurgitation

Causes: Functional (LV dilatation); rheumatic fever; infective endocarditis,
annular calcification in elderly, ruptured chordae tendinae or papillary muscles
(acute TR after MI), cardiomyopathy (chronic MR), congenital (eg ASD, AV
canal,. Remodelling Post MI,

Symptoms: Fatigue; pul cong oedema and also dyspnoea (With acute MR-
emergency due to flash/acute pulmonary oedema> Pul ht failure)

Signs: AF, hyperdynamic apex, RV heave, soft S1, split S2, loud P2 (Pul HTN)
Pansystolic murmur in apex radiating to axilla. (Holosystolic murmur in apex)

Tests: ECG: Af +- pmitrale, LVH
CXR- Big LA and LV, mitral vlave calcification, Pul oedema

Echo, Dopplar echo

Cardiac catherterisation- to confirm diagnosis

Mx: to control rate if fast AF and anticoag if Af, hx of embolism, prosthetic
valve, , MS. Diruetics, Surgery if getting worse.

116
CVS-System Wise 1700-by Sush and Team. 2016
Susmita, Asad, Manu, Saima, Zohaib, Savia, Shanu, Mona, Manisha, Sitara, Samreena, Sami and Komal


Complications OF MI:
Cardiac arrest
Unstable angina
Bradycardias or heart block
cardiogenic shock
Tachyarrhythmias:
Consider implantable cardiac defibrillator
Right ventricular failure (RVF)/infarction
Pericarditis
DVT & PE:
Systemic embolism:
Cardiac tamponade
Mitral regurgitation
Ventricular septal defect
Late malignant ventricular arrhythmias
Dresslers syndrome
Left ventricular aneurysm




Q: 141 A 60yo lady who had stroke 3 years ago now reports having increased dyspnea
on exertion and atrial fibrillation. CXR: straight left border on the cardiac
silhouette. What is the most probable dx?
a. Aortic regurgitation
b. Ischemic mitral regurgitation
c. Mitral valve prolapse
d. Pulmonary stenosis
e. Rheumatic mitral valve stenosis


Clincher(s)
A
B Key
C
D
E [Ref FB] Atrial fibrillation and straight left heart shadow means atrial
hypertrophy which can only occur in mitral stenosis. Stroke might be due to
the emboli formed because of the undelying rheumatic heart disease. Stroke
as an early complication and AF which had led to heart failure as a later
complication in chronic rheumatic heaet disease..stroke occur due embolism
of atrial fibrillation


The recurrency of RHD occur for a long time. so persistent valvular disease and

117
CVS-System Wise 1700-by Sush and Team. 2016
Susmita, Asad, Manu, Saima, Zohaib, Savia, Shanu, Mona, Manisha, Sitara, Samreena, Sami and Komal

carditis. 60% of carditis patients develop carditis due to rheumatic fever. So
Pencillin prohylaxis. OHCM-136

RHD prophylaxis with penicillin v
no carditis- 5 yrs
carditis -10
carditis with vulvular- 40yr or-lifelong

i) Dyspnoea on exertion ii) Straight left border of the cardiac silhouette. Iii)
Atrial fibrillation is a common association.

KEY R MS

Additional Ms> Thrombus in LA>stroke
Information
Stroke is a clincher with AF, which is common with mitral stenosis, which can
be due to prolonged stagnation of blood in the left atrium causing recurrent
emboli. MS and AF can be asymptomatic unless the patient has a complication
such as stroke or systematic embolus or symptomatic arrhythmias. Silhouette
sign is a characteristic for large atrium which enlarge gradually do you to MS.
MV is the commonest valve that is affected by RF.

Mitral Stenosis:

Presentation: dyspnea on exertion; orthopnoea, PND, fatigue; palpitations;
chest pain; systemic emboli (due to > LA which also cause AF which increases
coag risk) ; haemoptysis (due to Pul oed); chronic bronchitis-like picture

CAUSES: Rheumatic (most common), congenital, mucopolysaccharidoses,
endocardial fibroelastosis, malignant carcinoid (rare), prosthetic valve.

SIGNS: Malar flush on cheeks (due to low cardiac output> co2 and
vasoldilation); low-volume pulse; AF common; tapping, non-displaced, apex
beat (palpable S1).

On auscultation: loud S1; opening snap (pliable valve); (low pitched) rumbling
mid-diastolic murmur (heard best at apex in expiration, with patient on left
side)

ECG show P-mitrale. (due to enlargement of LA); No p wave in AF

ECHO is diagnostic.

CXR: left atrial enlargement: straightening of left heart border occur in mitral
stenosis plus double bubble sign of right side.
(double shadow in right cardiac silhouette- double bubble)

118
CVS-System Wise 1700-by Sush and Team. 2016
Susmita, Asad, Manu, Saima, Zohaib, Savia, Shanu, Mona, Manisha, Sitara, Samreena, Sami and Komal


TREATMENT: for AF: rate control (crucial), anticoag with warfarin, Loop
diuretics for <preload and pul congestion,

If Med Rx failure: balloon valvuloplasty (if pliable, non-calcified valve), open
mitral valvotomy or valve replacement (risk factor for clots). (if MR along with
MS)
Oral penicillin as prophylaxis against recurrent rheumatic fever.

Complications: Pulmonary hypertension, emboli, pressure from large LA on
local structures, eg hoarseness (recurrent laryngeal nerve), dysphagia
(oesophagus), bronchial obstruction; infective endocarditis


Reference OHCM- 138
Dr Khalid/Rabia

Q: 163 A 28yo man with complains of headache and nose bleeds also has pain in the
lower limbs on exertion. Exam: radio-femoral delay, cold legs with weak pulse
and mild systolic murmur with normal S1S2. What is the most probable dx?
a. TOF
b. ASD
c. VSD
d. PDA
e. CoA


Clincher(s) radio-femoral delay
A
B
C
D
E headache and nosebleeds - >hypertension, pain in lower limb on exertion -> as
reduced blood supply to leg due to coarctation, radio-femoral delay, cold legs
with week pulse, mid-systolic murmur are all features of coarctation of aorta.

KEY E. The key is coarctation of aorta
Additional
Information
Reference
Dr Khalid/Rabia Radio femoral delay is a clincher for coarctation of aorta.

Coarctation of the aorta Congenital narrowing of the descending aorta. More
common in boys.

Associations: Bicuspid aortic valve; Turners syndrome. Signs: Radiofemoral

119
CVS-System Wise 1700-by Sush and Team. 2016
Susmita, Asad, Manu, Saima, Zohaib, Savia, Shanu, Mona, Manisha, Sitara, Samreena, Sami and Komal

delay (femoral pulse later than radial); weak femoral pulse; BP; scapular
bruit; systolic murmur (best heard over the left scapula).

Complications: Heart failure; infective endocarditis. Coarctation of the aorta is
associated with berry aneurysms which if ruptured cause Sub arachnoid
hemorrhage.

Tests: CT or MRI-aortogram, CXR shows rib notching.

Cardiac catheterisation
To confirm the diagnosis when this is not clear at ultrasound.
To determine the gradient across the coarctation (with a gradient in
excess of 20 mm Hg considered to be significant).
To assess other abnormalities and the overall haemodynamic picture
when considering therapeutic options in more detail.
Therapeutically using balloon angioplasty with or without stent
implantation.

Treatment: Surgery or balloon dilatation stenting.


Q: 166 A 75yo lady who had mitral valve replacement 13 yrs ago has developed
recurrent breathlessness. Her husband has noticed prominent pulsation in her
neck. She complains of abdominal pain and ankle swelling. What is the most
probable dx?
a. Aortic regurgitation
b. Mitral regurgitation
c. Mitral stenosis
d. Tricuspid regurgitation
e. Pulmonary stenosis




Clincher(s) Hx of Valve replacement. prominent pulsation in her neck. abdominal pain
and ankle swelling.
A Aortic regurgitation, mitral stenosis and regurgitation do not involve
abdominal pain and ankle swelling. And the JVP will not be raised.

B
C
D Ms>valve replacement> complication > stenosis again> PH>LVF>RHF (here
give cause of RHF)

Prominent nck pulse...ankle oedema, ...pointing towards right heart


failure...options left are....tricuspid regurg or pulm stenosis..

120
CVS-System Wise 1700-by Sush and Team. 2016
Susmita, Asad, Manu, Saima, Zohaib, Savia, Shanu, Mona, Manisha, Sitara, Samreena, Sami and Komal

Pul stenosis is usually congenital so....ans is tricuspid regurg...in which most
prominent sign is...'giant v wave' in the jugular venous pulse.


E Pulmonary stenosis is mostly congenital. But it can be caused by rheumatic
fever and in this case the patient could have developed rheumatic fever given
his history of valve replacement. But other signs dont match.

KEY Ans. The key is D. Tricuspid regurgitation.
Additional
Information
Reference
Dr Khalid/Rabia
Tricuspid regurgitation

Causes: Functional (RV dilatation; eg due to pulmonary hypertension induced
by LV failure); rheumatic fever; infective endocarditis (IV drug abuser);
carcinoid syndrome; congenital (eg ASD, AV canal, Ebsteins anomaly, ie
downward displacement of the tricuspid valve drugs (eg ergot derived
dopamine agonists,fenfluramine).

Symptoms: Fatigue; hepatic pain on exertion; ascites; oedema and also
dyspnoea and orthopnoea if the cause is LV dysfunction.

Signs: Giant v waves and prominent y descent in JVP, RV heave; pansystolic
murmur, heard best at lower sternal edge in inspiration; pulsatile
hepatomegaly; jaundice; ascites.

Management: Treat underlying cause. Drugs: diuretics, digoxin, ACE-i. Valve
replacement (~10% 30-day mortality).




Q: 225 A 78yo lady on warfarin for atrial fibrillation lives in a care home. She presents
with hx of progressive confusion for three days. She was also noticed to have
bruises on her arms. INR = 7. What is the most probable dx?
a. Alzheimers
b. Delirium
c. Subdural hemorrhage
d. Vascular dementia
e. Picks dementia



Clincher(s) INR high, hx of fall, warfarin, progressive confusion

121
CVS-System Wise 1700-by Sush and Team. 2016
Susmita, Asad, Manu, Saima, Zohaib, Savia, Shanu, Mona, Manisha, Sitara, Samreena, Sami and Komal

A
B
C Age 78 yrs, living in a care home where unnoticed trivial injury is possible (like
fall), warfarin and high INR is potential risk factor of subdural haemorrhage
suggested by bruises on arms also.

Usually INR between 1-2 in A-fib patients, for warfarin should be 2-3.5.

INR- international normalized ratio- lab parmeter- ratio of PT (coag factors) etc
values of of standard with patient. Should be around 1-2. Beyond this it is
abnormal. INR higher (more thinner blood) (when excess of Warfarin) = chance
of bleeding is high, wProtocol for Rx for warfarin and INR. For valve diseases-
aim should be 2-3.5 with warfarin. But should not go below 2.

D its not vascular dementia bcs , vascular dementia presented with previous h/o
recurrent stroke which is appered gradually in stepladder patern.this pts no
h/o stoke or such weakness

E
KEY Ans. The key is C. Subdural haemorrhage.
Additional In this case Warfarin should be stopped and vit k should be given until INR<5
Information then we can restart warfarin


Reference
Dr Khalid/Rabia Subdural Haemorrhage: Consider this very treatable condition in all whose
conscious level fluctuates, and also in those having an evolving stroke,
especially if on anticoagulants.

Causes: Bleeding is from bridging veins between cortex and venous sinuses
(vulnerable to deceleration injury), resulting in accumulating haematoma
between dura and arachnoid. Most subdurals are from trauma but the trauma
is often forgotten as it was so minor or so long ago (up to 9 months). Elderly
are particularly susceptible. Other risk factors: falls (epileptics, alcoholics);
anticoagulation

Symptoms Fluctuating level of consciousness (seen in 35%) insidious physical
or intellectual slowing, sleepiness, headache, personality change, and
unsteadiness.

Signs: Raised ICP; (vomiting, headache, fundoscopy- papillooedema) seizures.
Localizing neurological symptoms (eg unequal pupils, hemiparesis) occur late
and often long after the injury

Imaging: CT/MRI shows clot midline shift (but beware bilateral isodense
clots). Look for crescent-shaped collection of blood over 1 hemisphere. The

122
CVS-System Wise 1700-by Sush and Team. 2016
Susmita, Asad, Manu, Saima, Zohaib, Savia, Shanu, Mona, Manisha, Sitara, Samreena, Sami and Komal

sickleshape different iates subdural blood from extradural haemorrhage.

Treatment Irrigation/evacuation, eg via burr twist drill and burr hole
craniostomy, can be considered 1st-line; craniotomy is 2nd-line if the clot has
organized. Address causes of the trauma (eg falls due cataract or arrhythmia;
abuse).



Q: 3 A 44yo woman has lost weight over 12 months. She has also noticed episodes
where her heart beats rapidly and strongly. She has a regular pulse rate of
90bpm. Her ECG shows sinus rhythm. What is the most appropriate
investigation to be done?

a. Thyroid antibodies
b. TFT
c. ECG
d. Echocardiogram
e. Plasma glucose

Clincher(s) Weight loss, heart beats rapid and strong
A
B Correct answer
C
D
E
KEY B Weight loss + palpitations = THYROTOXICOSIS ! TFT investigation
Additional
Information

123
CVS-System Wise 1700-by Sush and Team. 2016
Susmita, Asad, Manu, Saima, Zohaib, Savia, Shanu, Mona, Manisha, Sitara, Samreena, Sami and Komal

124
CVS-System Wise 1700-by Sush and Team. 2016
Susmita, Asad, Manu, Saima, Zohaib, Savia, Shanu, Mona, Manisha, Sitara, Samreena, Sami and Komal


125

CVS-System Wise 1700-by Sush and Team. 2016
Susmita, Asad, Manu, Saima, Zohaib, Savia, Shanu, Mona, Manisha, Sitara, Samreena, Sami and Komal

Reference Rapid Medicine, OHCM 210
Dr Khalid/Rabia Q. 1. What is the key?
Q. 2. What is the diagnosis?
Q. 3. What is the significance of episodes of rapid strong heart beat?
Ans. 1. The key is B. TFT.
Ans. 2. Thyrotoxicosis [weight loss over 12 months, episodes of rapid strong
heart beet (thyrotoxicosis induced paroxysmal atrial fibrillation) points
towards the diagnosis of thyrotoxicosis].
Ans. 3. Episodes of rapid strong heart beat indicates thyrotoxicosis induced
paroxysmal atrial fibrillation.



Q: 28 A 65yo woman presented with transient arm and leg weakness as well as a
sudden loss of vision in the left eye. Her symptoms resolved within the next
couple of hours. What is the most appropriate next investigation?

a. CT brain
b. Echo
c. Doppler USG
d. Arteriography
e. 24h ECG


Clincher(s) Transient arm & leg weakness, sudden loss of vision left eye, symptom
resolved within few hours
A To see existing infarcts
B Rarely shows cardiac cause
C Correct answer
D
E
KEY C Doppler USG help evaluate blood flow
Additional This could be TIA due to carotid artery narrowing.
Information








126
CVS-System Wise 1700-by Sush and Team. 2016
Susmita, Asad, Manu, Saima, Zohaib, Savia, Shanu, Mona, Manisha, Sitara, Samreena, Sami and Komal

127
CVS-System Wise 1700-by Sush and Team. 2016
Susmita, Asad, Manu, Saima, Zohaib, Savia, Shanu, Mona, Manisha, Sitara, Samreena, Sami and Komal

128
CVS-System Wise 1700-by Sush and Team. 2016
Susmita, Asad, Manu, Saima, Zohaib, Savia, Shanu, Mona, Manisha, Sitara, Samreena, Sami and Komal



Reference OHCM 480

129
CVS-System Wise 1700-by Sush and Team. 2016
Susmita, Asad, Manu, Saima, Zohaib, Savia, Shanu, Mona, Manisha, Sitara, Samreena, Sami and Komal

Dr Khalid/Rabia
RCP recommendations
Patients with suspected TIA who are at high risk of stroke (eg, an
ABCD2 score of 4 or above) should receive:
o Aspirin or clopidogrel (each as a 300 mg loading dose
and then 75 mg daily) and a statin started immediately.
o NB: clopidogrel is not licensed for the management of
TIA and therefore the National Institute for Health and
Care Excellence (NICE) and the British National
Formulary (BNF) recommend aspirin plus modified-
release dipyridamole.
o Specialist assessment and investigation within 24 hours
of onset of symptoms.
People with crescendo TIA (two or more TIAs in a week), atrial
fibrillation or those on anticoagulants should be treated as being at
high risk of stroke even if they may have an ABCD2 score of 3 or
below.
Patients with suspected TIA who are at low risk of stroke (eg, an
ABCD2 score of 3 or below) should receive:
o Aspirin or clopidogrel (each as a 300 mg loading dose
and then 75 mg daily) and a statin.
o NB: clopidogrel is not licensed for the management of
TIA and therefore NICE and the BNF recommend aspirin
plus modified-release dipyridamole.
Patients with TIA in atrial fibrillation should be anticoagulated in
the TIA clinic once intracranial bleeding has been excluded and if
there are no other contra-indications.




Q: 43 A man brought into the ED after being stabbed in the chest. Chest is bilaterally
clear with muffled heart sounds. BP is 60/nil. Pulse is 120bpm. JVP raised.
What is the most likely dx?

a. Pulmonary embolism
b. Cardiac tamponade
c. Pericardial effusion
d. Hemothorax
e. Pneumothorax


Clincher(s)
A Cough, dyspnea, sharp chest pain
B Correct answer
C Cause of this are the causes of pericarditis. Common cause of pericarditis =

130
CVS-System Wise 1700-by Sush and Team. 2016
Susmita, Asad, Manu, Saima, Zohaib, Savia, Shanu, Mona, Manisha, Sitara, Samreena, Sami and Komal

Virus (coxsackie viruses), MI, TB, RhF etc
D dull percussion note on the affected side
E Symptoms are asymptomatic
KEY C Cardiac Tamponade ! BECKS TRIAD
Additional Cardiac tamponade is a form of cardiogenic shock which is caused by fluid
Information accumulation in the pericardial space.
The fluid which is either blood or a large pericardial effusion, increases the
pericardial pressure and inhibits venous return.
Reduced venous return results in reduced cardiac output, hypotension and
shock.

Cardiac tamponade is an emergency.
Pericardial aspiration may be both therapeutic and diagnostic.
One should ensure that pericardial fluid is sent for microbiological and
cytological investigation.
The underlying condition, such as ventricular rupture or malignant pericarditis,
should be treated but it is clear that the prognosis is often poor and a palliative
approach may be appropriate.

Acute: Cardiac tamponade treated by emergency pericardiocentesis.

Possible clinical features include:
hypotension and shock
pulsus paradoxus
inaudible or distant heart sounds
distension of neck veins
Kussmaul's sign
confusion

Patient's with acute cardiac tamponade classically present with Beck's triad.


Kussmaul's sign is a rise in JVP on inspiration.
Kussmaul's sign is seen in conditions in which right ventricular filling is limited
by:
pericardial fluid
non-compliant pericardium or myocardium
The classical differential diagnosis is:
constrictive pericarditis
restrictive cardiomyopathy
cardiac tamponade

131
CVS-System Wise 1700-by Sush and Team. 2016
Susmita, Asad, Manu, Saima, Zohaib, Savia, Shanu, Mona, Manisha, Sitara, Samreena, Sami and Komal







Reference http://www.gpnotebook.co.uk/simplepage.cfm?ID=1315635222&linkID=2598
4&cook=yes

OHCM 814 & 148
Dr Khalid/Rabia Beck's triad: low bp, muffled HS, raised jvp .. Cardiac tamponade
Features
dyspnoea
raised JVP, with an absent Y descent - this is due to the limited right
ventricular filling

132
CVS-System Wise 1700-by Sush and Team. 2016
Susmita, Asad, Manu, Saima, Zohaib, Savia, Shanu, Mona, Manisha, Sitara, Samreena, Sami and Komal

tachycardia
hypotension
muffled heart sounds
pulsus paradoxus
Kussmaul's sign (much debate about this)
ECG: electrical alternans



Q: 87 A man with prosthetic heart valve underwent hemicolectomy and after some
days complains of left hypochondriac pain, fever and has a systolic murmur.
What is the next investigation to ascertain the cause of HF?

a. CT
b. Blood culture
c. ECG
d. MRI
e. Radioactive thyroid scan


Clincher(s) Fever & systolic murmur ! after some days
A
B Correct answer
C
D
E
KEY B Blood culture done prompt if fever above symptoms last >1 week.
Additional Infective endocarditis is a serious condition characterised by colonisation or
Information invasion of the heart valves or the mural endocardium by a microbiological
agent. Vegetations form containing fibrin, platelets and micro-organisms which
seed into the circulation causing persistent fever, non-specific systemic
symptoms, murmurs and emboli. Normal, abnormal or prosthetic valves may
be involved. The most common presentation is a sub-acute bacterial
endocarditis.

The classical diagnostic triad is:
persistent fever
emboli
new or changing murmurs.

133
CVS-System Wise 1700-by Sush and Team. 2016
Susmita, Asad, Manu, Saima, Zohaib, Savia, Shanu, Mona, Manisha, Sitara, Samreena, Sami and Komal

134
CVS-System Wise 1700-by Sush and Team. 2016
Susmita, Asad, Manu, Saima, Zohaib, Savia, Shanu, Mona, Manisha, Sitara, Samreena, Sami and Komal

135
CVS-System Wise 1700-by Sush and Team. 2016
Susmita, Asad, Manu, Saima, Zohaib, Savia, Shanu, Mona, Manisha, Sitara, Samreena, Sami and Komal


136
CVS-System Wise 1700-by Sush and Team. 2016
Susmita, Asad, Manu, Saima, Zohaib, Savia, Shanu, Mona, Manisha, Sitara, Samreena, Sami and Komal




Reference OHCM & Rapid Medicine
Dr Khalid/Rabia


Q: 106 A 45yo lady has 10m hx of SOB. She is found to have irregularly irregular pulse

137
CVS-System Wise 1700-by Sush and Team. 2016
Susmita, Asad, Manu, Saima, Zohaib, Savia, Shanu, Mona, Manisha, Sitara, Samreena, Sami and Komal

and loud P2 with fixed splitting and ejection systolic murmur in left 2nd ICS.
What is the probable dx?

a. TOF (Tetralogy of Fallot)
b. ASD (Atrial Septal Defect)
c. VSD (Ventricular Septal Defect)
d. PDA (Patent Ductus Arteriosus)
e. CoA


Clincher(s) Fixed splitting
A TOF features " VSD, PS, RVH, Aorta overriding VSD
B Correct answer
C Pansytolic murmur ! MR TR or VSD
D Machinery murmur = Continuous murmurs are often maximal in late systole
E
KEY B fixed splitting second sound
Additional Atrial septal defects (ASDs) are the most likely congenital heart defect to be
Information found in adulthood. They carry a significant mortality, with 50% of patients
being dead at 50 years. Two types of ASDs are recognised, ostium secundum
and ostium primum. Ostium secundum are the most common.

Features
ejection systolic murmur, fixed splitting of S2
embolism may pass from venous system to left side of heart causing a stroke

Ostium secundum (70% of ASDs)
associated with Holt-Oram syndrome (tri-phalangeal thumbs)
ECG: RBBB with RAD

Ostium primum
present earlier than ostium secundum defects
associated with abnormal AV valves
ECG: RBBB with LAD, prolonged PR interval

138
CVS-System Wise 1700-by Sush and Team. 2016
Susmita, Asad, Manu, Saima, Zohaib, Savia, Shanu, Mona, Manisha, Sitara, Samreena, Sami and Komal

139
CVS-System Wise 1700-by Sush and Team. 2016
Susmita, Asad, Manu, Saima, Zohaib, Savia, Shanu, Mona, Manisha, Sitara, Samreena, Sami and Komal

140
CVS-System Wise 1700-by Sush and Team. 2016
Susmita, Asad, Manu, Saima, Zohaib, Savia, Shanu, Mona, Manisha, Sitara, Samreena, Sami and Komal


Reference http://www.ncbi.nlm.nih.gov/books/NBK341/

http://www.gpnotebook.co.uk/simplepage.cfm?ID=-
348848096&linkID=3165&cook=yes

OHCM
Dr Khalid/Rabia Ans. 1. The key is B. Atrial septal defect.
Ans. 2. Diagnosis is ASD with atrial fibrillation. [i) atrial fibrillation = irregularly
irregular pulse. ii) ASD = SOB, fixed splitting with loud P2, ESM in pulmonary
area]. This pictures are of atrial septal defect itself though similar findings we
get in pulmonary hypertension. One should not misdiagnose SOB, ESM in
pulmonary area and loud P2 as pulmonary hypertension in the given case.


Q: 108 A 76yo is treated with HTN. He suffers from pain and redness at the MTP
(metatarsophalangeal) joint of his right big toe. Which of the following anti-
HTN cause this symptoms?

a. Losartan
b. Bendroflumethiazide
c. Ramipril
d. Bisoprolol
e. Verapamil


Clincher(s) Pain & redness at MTP at Right big toe ! gout
A Angiotensin 2 receptor antagonist
B Correct answer (Thiazide Diuretics contraindicated in GOUT)
C
D
E

141
CVS-System Wise 1700-by Sush and Team. 2016
Susmita, Asad, Manu, Saima, Zohaib, Savia, Shanu, Mona, Manisha, Sitara, Samreena, Sami and Komal

KEY B Bendroflumethiazide
Additional Gout is a form of microcrystal synovitis caused by the deposition of
Information monosodium urate monohydrate in the synovium. It is caused by chronic
hyperuricaemia (uric acid > 0.45 mmol/l)

Drug causes
thiazides, furosemide
alcohol
cytotoxic agents
pyrazinamide

Thiazide diuretics work by inhibiting sodium absorption at the beginning of the
distal convoluted tubule (DCT). Potassium is lost as a result of more sodium
reaching the collecting ducts. Thiazide diuretics have a role in the treatment of
mild heart failure although loop diuretics are better for reducing overload. The
main use of bendroflumethiazide was in the management of hypertension but
recent NICE guidelines now recommend other thiazide-like diuretics such as
indapamide and chlortalidone.

Common adverse effects
dehydration
postural hypotension
hyponatraemia, hypokalaemia, hypercalcaemia
gout
impaired glucose tolerance
impotence

Rare adverse effects
thrombocytopaenia
agranulocytosis
photosensitivity rash
pancreatitis

142
CVS-System Wise 1700-by Sush and Team. 2016
Susmita, Asad, Manu, Saima, Zohaib, Savia, Shanu, Mona, Manisha, Sitara, Samreena, Sami and Komal

143
CVS-System Wise 1700-by Sush and Team. 2016
Susmita, Asad, Manu, Saima, Zohaib, Savia, Shanu, Mona, Manisha, Sitara, Samreena, Sami and Komal









Reference OHCM 304 & 550 , Pass Medicine , Rapid Medicine
Dr Khalid/Rabia

Q: 113 A 72yo man has been advised to have antibiotic prophylaxis for some years
now before dental tx. He has never experienced chest pain. Three weeks ago,

144
CVS-System Wise 1700-by Sush and Team. 2016
Susmita, Asad, Manu, Saima, Zohaib, Savia, Shanu, Mona, Manisha, Sitara, Samreena, Sami and Komal

he noticed breathlessness on exertion and for one week he had orthopnea. His
pulse is normal. What is the most probable dx?

a. Aortic regurgitation
b. Ischemic mitral regurgitation
c. Mitral valve prolapse
d. Pulmonary stenosis
e. Mitral valve stenosis


Clincher(s) Dyspnoea on exertion, orthopnoea, pulse normal
A Can be dx also
B
C
D
E Correct answer
KEY E
Additional Aortic Insufficiency
Information











Mitral Stenosis

145
CVS-System Wise 1700-by Sush and Team. 2016
Susmita, Asad, Manu, Saima, Zohaib, Savia, Shanu, Mona, Manisha, Sitara, Samreena, Sami and Komal

146
CVS-System Wise 1700-by Sush and Team. 2016
Susmita, Asad, Manu, Saima, Zohaib, Savia, Shanu, Mona, Manisha, Sitara, Samreena, Sami and Komal



147
CVS-System Wise 1700-by Sush and Team. 2016
Susmita, Asad, Manu, Saima, Zohaib, Savia, Shanu, Mona, Manisha, Sitara, Samreena, Sami and Komal

Reference Aortic Insufficiency

https://www.nlm.nih.gov/medlineplus/ency/article/000179.htm

Mitral Stenosis

https://www.nlm.nih.gov/medlineplus/ency/article/000175.htm

OHCM 144
Dr Khalid/Rabia The patient has mitral stenosis or Aortic regurgitation. he is given prophylaxis
for infective endocarditis. According to OHCM, such prophylaxis has no benefit
and should not be given.

RISK FACTORS for IE: aortic or mitral valve disease; tricuspid valves in IV drug
users; coarctation; patent ductus arteriosus; VSD; prosthetic valves

Mitral Stenosis: Presentation: dyspnoea; fatigue; palpitations; chest pain;
systemic emboli; haemoptysis; chronic bronchitis-like picture
CAUSES: Rheumatic, congenital, mucopolysaccharidoses, endocardial
fibroelastosis, malignant carcinoid, prosthetic valve.
SIGNS: Malar flush on cheeks (due to cardiac output); low-volume pulse; AF
common; tapping, non-displaced, apex beat (palpable S1). On auscultation:
loud S1; opening snap (pliable valve); rumbling mid-diastolic murmur (heard
best in expiration, with patient on left side
ECG shows P-mitrale ECHO is diagnostic. CXR: left atrial enlargement (double
shadow in right cardiac silhouette)
TREATMENT: balloon valvuloplasty (if pliable, non-calcified valve), open mitral
valvotomy or valve replacement.
Complications: Pulmonary hypertension, emboli, pressure from large LA on
local structures, eg hoarseness (recurrent laryngeal nerve), dysphagia
(oesophagus), bronchial obstruction; infective endocarditis

Aortic Regurgitation: Causes:
Acute: Infective endocarditis, ascending aortic dissection, chest trauma.
SYMPTOMS: Exertional dyspnoea, orthopnoea, and paroxysmal nocturnal
dyspnoea. palpitations, angina, syncope, CCF

Signs: Collapsing (water-hammer) pulse (p40); wide pulse pressure; displaced,
hyperdynamic apex beat; high-pitched early diastolic murmur (heard best in
expiration, with patient sitting forward).





Q: 353 A 47yo man with hx of IHD complains of chest pain with SOB on exertion over

148
CVS-System Wise 1700-by Sush and Team. 2016
Susmita, Asad, Manu, Saima, Zohaib, Savia, Shanu, Mona, Manisha, Sitara, Samreena, Sami and Komal

the past few days. ECG normal, Echo= increased EF and decreased septal wall
thickness. What is the most likely dx?
a. Dilated CM
b. Constrictive pericarditis
c. Amyloidosis
d. Subacute endocarditis



Clincher(s) h/o IHD
A Echocardiogram shows left ventricular dilatation with normal or thinned walls
and reduced ejection fraction.
B Constrictive pericarditis The heart is encased in a rigid pericardium.
Causes: Often unknown (UK); elsewhere TB, or after any pericarditis.
Clinical features: These are mainly of right heart failure with JVP (with
prominent
x and y descents, p41); Kussmauls sign (JVP rising paradoxically with
inspiration);
soft, diff use apex beat; quiet heart sounds; S3; diastolic pericardial knock,
hepatosplenomegaly,
ascites, and oedema.
Tests: CXR: small heart } pericardial calcifi cation (if none, CT/MRI helps
distinguish
from other cardiomyopathies). Echo; cardiac catheterization.
Management: Surgical excision.
C On echocardiography the heart shows restrictive filling pattern, with normal to
mildly reduced systolic function
D echocardiography (ultrasound study of the heart) may visualize vegetations
(growths) on heart valves.
E -
KEY Dilated Cardiomyopathy
Additional Dilated cardiomyopathy A dilated, fl abby heart of unknown cause.
Information Associations:
alcohol, BP, haemochromatosis, viral infection, autoimmune, peri- or
postpartum,
thyrotoxicosis, congenital (X-linked). Prevalence: 0.2%. Presentation: Fatigue,
dyspnoea, pulmonary oedema, RVF, emboli, AF, VT. Signs: Pulse, BP,
JVP, displaced,
diff use apex, S3 gallop, mitral or tricuspid regurgitation (MR/TR), pleural eff
usion,
oedema, jaundice, hepatomegaly, ascites. Tests: Blood: plasma BNP is sensitive
and
specifi c in diagnosing heart failure. Na+ indicates a poor prognosis. CXR:
cardiomegaly,

149
CVS-System Wise 1700-by Sush and Team. 2016
Susmita, Asad, Manu, Saima, Zohaib, Savia, Shanu, Mona, Manisha, Sitara, Samreena, Sami and Komal

pulmonary oedema. ECG: tachycardia, non-specifi c T wave changes, poor R
wave
progression. Echo: globally dilated hypokinetic heart and low ejection fraction.
Look
for MR, TR, LV mural thrombus. : Bed rest, diuretics, digoxin, ACE-i, anticoag
ul ation, biventricular
pacing, ICDs, cardiac transplantation. Mortality: Variable, eg 40% in 2yrs
Reference

Dr Khalid/Rabia


Q: 363 A 66yo man has the following ECG. What is the most appropriate next step in
management?

a. Metoprolol
b. Digoxin
c. Carotid sinus massage
d. Adenosine
e. Amiodarone



Clincher(s)
A It's AF due to absent P wave with an irregular R-R interval, So the drug of
choice for AF is a beta blocker, if HF Present we give digoxin. Here my answer
will be A.

B
C
D
E
KEY A
Additional Treatment in emergency situations ultimately involves electrical pacing.
Information Pharmacological management of suspected beta-blocker overdose might be
treated with glucagon, calcium channel blocker overdose treated with calcium
chloride and digitalis toxicity treated with the digoxin immune Fab.[8]

Third-degree AV block can be treated by use of a dual-chamber artificial


pacemaker.[9] This type of device typically listens for a pulse from the SA node

150
CVS-System Wise 1700-by Sush and Team. 2016
Susmita, Asad, Manu, Saima, Zohaib, Savia, Shanu, Mona, Manisha, Sitara, Samreena, Sami and Komal

via lead in the right atrium and sends a pulse via a lead to the right ventricle at
an appropriate delay, driving both the right and left ventricles. Pacemakers in
this role are usually programmed to enforce a minimum heart rate and to
record instances of atrial flutter and atrial fibrillation, two common secondary
conditions that can accompany third-degree AV block. Since pacemaker
correction of third-degree block requires full time pacing of the ventricles, a
potential side effect is pacemaker syndrome, and may necessitate use of a
biventricular pacemaker, which has an additional 3rd lead placed in a vein in
the left ventricle, providing a more coordinated pacing of both ventricles.

The 2005 Joint European Resuscitation and Resuscitation Council (UK)


guidelines[10] state that atropine is the first line treatment especially if there
were any adverse signs, namely: 1) heart rate < 40 bpm, 2) systolic blood
pressure < 100 mm Hg, 3) signs of heart failure, and 4) ventricular arrhythmias
requiring suppression. If these fail to respond to atropine or there is a
potential risk of asystole, transvenous pacing is indicated. The risk factors for
asystole include 1) previous asystole, 2) complete heart block with wide
complexes, and 3) ventricular pause for > 3 seconds. Mobitz Type 2 AV block is
another indication for pacing.

As with other forms of heart block, secondary prevention may also include
medicines to control blood pressure and atrial fibrillation, as well as lifestyle
and dietary changes to reduce risk factors associated with heart attack and
stroke.


Reference Wikipedia

Dr Khalid/Rabia


Q: 381 ECG of an 80yo pt of ICH shows saw-tooth like waves, QRS complex of 80ms
duration, ventricular rate=150/min and regular R-R interval. What is the most
porbable dx?
a. Atrial fib
b. Atrial flutter
c. SVT
d. Mobitz type1 second degree heart block
e. Sinus tachycardia


Clincher(s)
A ECG shows absent P waves, irregular QRS complexes
B Ans. The key is B. Atrial flutter. [Saw-tooth like waves, normal QRS complex of
80 ms (normal range 70-100 ms), ventricular rate of 150/min and regular R-R
interval are suggestive of atrial flutter].

151
CVS-System Wise 1700-by Sush and Team. 2016
Susmita, Asad, Manu, Saima, Zohaib, Savia, Shanu, Mona, Manisha, Sitara, Samreena, Sami and Komal

C Supraventricular tachycardia (SVT): P wave absent or inverted after QRS.
D
E Sinus tachycardia: normal P wave followed by normal QRS.
KEY Atrial flutter
Additional Atrial fl utter ECG: continuous atrial depolarization (eg ~300/min, but very
Information variable) produces a sawtooth baseline } 2 : 1 AV block (as if SVT at, eg
150bpm). Carotid sinus massage and IV adenosine transiently block the AV
node and may unmask fl utter waves.
Treatment: Cardioversion may be indicated (anticoagulate before, see p125).
Anti-AF
drugs may not workbut consider amiodarone to restore sinus rhythm, and
amiodarone
or sotalol to maintain it. Aim to control rate as above; if the IV route is
needed, a -blocker is preferred (eg p819). Rarely, cavotricuspid isthmus
ablation.
Reference
Dr Khalid/Rabia


Q:402 A 61yo man has been referred to the OPD with frequent episodes of
breathlessness and chest pain a/w palpitations. He has a regular pulse
rate=60bpm. ECG=sinus rhythm. What is the most appropriate inv to be done?
a. Cardiac enzymes
b. CXR
c. ECG
d. Echo
e. 24h ECG



Clincher(s)
A
B
C
D
E If you have palpitation and do an ECG and its normal, next is 24-hr ECG to
detect any arrhythmia that may not be presented now. Palpitations are an
indication for 24 hr

KEY 24hr ECG
Additional
Information
Reference
Dr Khalid/Rabia

152
CVS-System Wise 1700-by Sush and Team. 2016
Susmita, Asad, Manu, Saima, Zohaib, Savia, Shanu, Mona, Manisha, Sitara, Samreena, Sami and Komal

Q: 406 A woman had an MI. She was breathless and is put on oxygen mask and GTN,
her chest pain has improved. Her HR=40bpm. ECG shows ST elevation in leads
I, II, III. What is your next step?
a. LMWH
b. Streptokinase
c. Angiography
d. Continue current management
e. None



Clincher(s)
A
B
C
D
E
KEY Streptokinase
Additional OHCM
Information
Pre-hospital Arrange emergency ambulance. Aspirin 300mg chewed 31 (if no
absolute
CI) and GTN sublingual. Analgesia, eg morphine 510mg IV + metoclopramide
10mg IV (not IM because of risk of bleeding with thrombolysis).
In hospital O2, IVI, morphine, aspirin p808810.
Then the key question for subsequent management of ACS is whether there is
STsegment
elevation (includes new onset LBBB or a true posterior MI). See fi g 1.
ST-segment elevation
Primary angioplasty or thrombolysis, if no contraindication p808.
-blocker, eg atenolol 5mg IV unless contraindicated, eg asthma.
ACE-inhibitor: Consider starting ACE-i (eg lisinopril 2.5mg) in all normotensive
patients
(systolic 120mm/Hg) within 24h of acute MI, especially if there is clinical
evidence of heart failure or echo evidence of LV dysfunction.
Consider clopidogrel 300mg loading followed by 75mg/day for 30 days. 32
ACS without ST-segment elevation 33
-blocker, eg atenolol 5mg IV and nitrates IV unless contraindicated.
Antithrombotic fondaparinux if low bleeding risk and no angiography
planned for
24h, otherwise consider low molecular weight heparin (eg enoxaparin
1mg/kg/12h
SC for 28 days)
Assess risk, eg GRACE SCORE.
High-risk patients (persistent or recurrent ischaemia, ST , diabetes,
troponin):

153
CVS-System Wise 1700-by Sush and Team. 2016
Susmita, Asad, Manu, Saima, Zohaib, Savia, Shanu, Mona, Manisha, Sitara, Samreena, Sami and Komal

GPIIb/IIIa antagonist (eg tirofi ban), 34 or bivalirudin, and angiography within
96h.
Clopidogrel, in addition to aspirin, should be considered for up to 12 months.
30
Low-risk (eg no further pain, fl at or inverted T-waves, or normal ECG, and
negative
troponin): clopidogrel if risk 1 . 53%/yr. Discharge if a repeat troponin is
negative.
Treat medically and arrange further investigation if recurrent ischaemia
Reference
Dr Khalid/Rabia

Q: 409 A 29yo lady came to the ED with complaints of palpitations that have been
there for the past 4 days and also feeling warmer than usual. Exam:
HR=154bpm, irregular rhythm. What is the tx for her condition?
a. Amiadarone
b. Beta blockers
c. Adenosine
d. Verapamil
e. Flecainide



Clincher(s)
A
B
C
D
E
KEY B 4 days hx with thyrotoxicosis
Additional (AT>Aflutter>Fibrillation) SVT (all Tachy above ventricle): Vagal manoeuvres
Information (breath-holding, valsalva manoeuvre, carotid massage) are 1st line treatments
if haemodynamically stable. IV adenosine is the drug of choice.
If adenosine fails, use verapamil 5mg IV over 2min, or over 3min if elderly
( not if already on -blocker). If no response, give further dose of 5mg IV
after 510min. Alternatives: atenolol 2.5mg IV at 1mg/min repeated at 5min
intervals to a maximum of 10mg or sotalol 2060mg (if eGFR >60) IV over
10min. If no good, use DC cardioversion.
Reference ohcm
Dr Khalid/Rabia

Q: 412 A 45yo man who is diabetic and HTN but poorly compliant has chronic SOB,
develops severe SOB and chest pain. Pain is sharp, increased by breathing and
relieved by sitting forward. What is the single most appropriate dx?
a. MI
b. Pericarditis

154
CVS-System Wise 1700-by Sush and Team. 2016
Susmita, Asad, Manu, Saima, Zohaib, Savia, Shanu, Mona, Manisha, Sitara, Samreena, Sami and Komal

c. Lung cancer
d. Good pastures syndrome
e. Progressive massive fibrosis



Clincher(s)
A
B
C
D
E
KEY Pericarditis
Additional Acute pericarditis This is infl ammation of the pericardium. It may be idiopathic
Information or secondary to:
Viruses (Coxsackie, fl u, EpsteinBarr, mumps, varicella, HIV)
Bacteria (pneumonia, rheumatic fever, TB, staphs, streps, MAI in HIV, p410)
Fungi
Myocardial infarction, Dresslers (p712)
Drugs: procainamide, hydralazine, penicillin, cromolyn sodium, isoniazid
Others: uraemia, rheumatoid arthritis, SLE, myxoedema, trauma, surgery,
malignancy (and antineoplastic agents), radiotherapy, sarcoidosis.
Clinical features: Central chest pain worse on inspiration or lying fl at } relief
by sitting forward. A pericardial friction rub (p44) may be heard. Look for
evidence of a pericardial eff usion or cardiac tamponade (see below). Fever
may occur.
Tests: ECG classically shows concave (saddle-shaped) ST segment elevation,
but may be normal or non-specifi c (10%); see fi g 1. Blood tests: FBC, ESR,
U&E, cardiac enzymes (NB: troponin may be raised), viral serology, blood
cultures, and, if indicated, autoantibodies
(p555), fungal precipitins, thyroid function tests. Cardiomegaly on CXR may
indicate a pericardial eff usion. Echo (if suspected pericardial eff usion).
Treatment: Analgesia, eg ibuprofen 400mg/8h PO with food. Treat the cause.
Consider colchicine before steroids/immunosuppressants if relapse or
continuing symptoms occur. 1540% do recur. Steroids may increase the risk
of recurrence.
Reference ohcm
Dr Khalid/Rabia
Q: 418 A 50yo lady presents to ED with sudden severe chest pain radiating to both
shoulder and accompanying SOB. Exam: cold peripheries and paraparesis.
What is the single most appropriate dx?
a. MI
b. Aortic dissection
c. Pulmonary embolism
d. Good pastures syndrome

155
CVS-System Wise 1700-by Sush and Team. 2016
Susmita, Asad, Manu, Saima, Zohaib, Savia, Shanu, Mona, Manisha, Sitara, Samreena, Sami and Komal

e. Motor neuron disease



Clincher(s)
A
B About 96% of individuals with aortic dissection present with severe pain that
had a sudden onset. It may be described as tearing, stabbing, or sharp in
character. 17% of individuals will feel the pain migrate as the dissection
extends down the aorta. The location of pain is associated with the location of
the dissection. Anterior chest pain is associated with dissections involving the
ascending aorta, while interscapular (back) pain is associated with descending
aortic dissections. If the pain is pleuritic in nature, it may suggest acute
pericarditis caused by bleeding into the pericardial sac. This is a particularly
dangerous eventuality, suggesting that acute pericardial tamponade may be
imminent. Pericardial tamponade is the most common cause of death from
aortic dissection.[medical citation needed]

While the pain may be confused with the pain of a myocardial infarction (heart
attack), aortic dissection is usually not associated with the other signs that
suggest myocardial infarction, including heart failure and ECG changes.

Individuals with aortic dissection who do not present with pain have chronic
dissection.

Less common symptoms that may be seen in the setting of aortic dissection
include congestive heart failure (7%), fainting (9%), stroke (6%), ischemic
peripheral neuropathy, paraplegia, and cardiac arrest.[1] If the individual had a
fainting episode, about half the time it is due to bleeding into the pericardium
leading to pericardial tamponade.

Neurologic complications of aortic dissection (i.e., stroke and paralysis) are due
to involvement of one or more arteries supplying portions of the central
nervous system.

If the aortic dissection involves the abdominal aorta, compromise of the


branches of the abdominal aorta is possible. In abdominal aortic dissections,
compromise of one or both renal arteries occurs in 58% of cases, while
mesenteric ischemia (ischemia of the large intestines) occurs 35% of the time


C
D
E
KEY Aortic dissection
Additional (FB) B-Dissecting aortic aneurysm. pt history of chest pain & interscapular back pain indicate
dissecting aneurysm in the decending thoracic aorta can causes interference with the blood
Information

156
CVS-System Wise 1700-by Sush and Team. 2016
Susmita, Asad, Manu, Saima, Zohaib, Savia, Shanu, Mona, Manisha, Sitara, Samreena, Sami and Komal

supply to the anterior spinal artery and causes the infarction of the ant aspect of the spinal artery
that is anterior spinal artery syndrome and paraparesis. Treatment- Type A: Immediately IV
labetalol for control of HTN to reduces the extension of dissection then for surgical Mx but Type-
B for only conservative Mx.

FB: Aortic dissection sudden onset decreased blood supply to limbs and hypoxuc injury to brain
FB: Aortic dissection with limb ischemia secondary to cord ischemia


Reference
Dr Khalid/Rabia


Q: 424 A 28yo man presents with rapid pounding in the chest. He is completely
conscious throughout. The ECG was taken (SVT). What is the 1st med to be
used to manage this condition?
a. Amiodarone
b. Adenosine
c. Lidocaine
d. Verapamil
e. Metoprolol



Clincher(s)
A
B
C
D
E
KEY Adenosine
Additional
Information
Reference
Dr Khalid/Rabia












157
CVS-System Wise 1700-by Sush and Team. 2016
Susmita, Asad, Manu, Saima, Zohaib, Savia, Shanu, Mona, Manisha, Sitara, Samreena, Sami and Komal


158

Potrebbero piacerti anche